+ All Categories
Home > Documents > The In nite Actuary Exam 1/P Online Seminar A.1 ... · The In nite Actuary Exam 1/P Online Seminar...

The In nite Actuary Exam 1/P Online Seminar A.1 ... · The In nite Actuary Exam 1/P Online Seminar...

Date post: 03-Mar-2021
Category:
Upload: others
View: 1 times
Download: 0 times
Share this document with a friend
163
The Infinite Actuary Exam 1/P Online Seminar A.1 Fundamentals of Probability Solutions Last updated February 23, 2013 1. Let A be those buying auto insurance and H those who buy homeowners insurance. Then P[A] = 2P[H ] 0.7 = P[A H ] = P[A] + P[H ] - P[AH ] 0.7 = 2P[H ] + P[H ] - 0.2 P[H ]=0.3 P[HA 0 ] = P[H ] - P[AH ]=0.3 - 0.2= 0.1 The Venn diagram is 0.4 A 0.2 H 0.1 2. Let F be those who buy fire insurance and T those who buy theft insurance. We want P[F T ] - P[F T ] = (P[F ] + P[T ] - P[FT ]) - P[FT ] =0.4+0.3 - 0.2 - 0.2 =0.3 The Venn diagram is 0.2 F 0.2 T 0.1 3.(A B) 0 = A 0 B 0 , so we have 0.3 = P[AB 0 ] and 0.4 = P[A 0 B 0 ]. Adding gives us 0.7 = P[B 0 ], and thus P[B]=1 - 0.7= 0.3 0.3 A B 0.4 4. Let L denote the event that the trip requires lab work, and S the event that it results in a referral to a specialist. We are given: P[(L S ) 0 ]=0.35 P[L S ]=1 - 0.35 = 0.65 P[S ]=0.30 P[L]=0.40 c 2013 The Infinite Actuary, LLC p. 1 A.1 Solutions
Transcript
Page 1: The In nite Actuary Exam 1/P Online Seminar A.1 ... · The In nite Actuary Exam 1/P Online Seminar A.1 Fundamentals of Probability Solutions Last updated February 23, 2013 1. Let

The Infinite Actuary Exam 1/P Online SeminarA.1 Fundamentals of Probability SolutionsLast updated February 23, 2013

1. Let A be those buying auto insurance and H those who buy homeowners insurance. Then

P[A] = 2P[H]

0.7 = P[A ∪H] = P[A] + P[H]− P[AH]

0.7 = 2P[H] + P[H]− 0.2

P[H] = 0.3

P[HA′] = P[H]− P[AH] = 0.3− 0.2 = 0.1

The Venn diagram is

0.4

A

0.2

H

0.1

2. Let F be those who buy fire insurance and T those who buy theft insurance. We want

P[F ∪ T ]− P[F ∩ T ] = (P[F ] + P[T ]− P[FT ])− P[FT ]

= 0.4 + 0.3− 0.2− 0.2

= 0.3

The Venn diagram is

0.2

F

0.2

T

0.1

3. (A∪B)′ = A′B′, so we have 0.3 = P[AB′] and 0.4 = P[A′B′]. Adding gives us 0.7 = P[B′], and

thus P[B] = 1− 0.7 = 0.3

0.3

A B

0.4

4. Let L denote the event that the trip requires lab work, and S the event that it results in areferral to a specialist. We are given:

P[(L ∪ S)′] = 0.35 P[L ∪ S] = 1− 0.35 = 0.65

P[S] = 0.30 P[L] = 0.40

c©2013 The Infinite Actuary, LLC p. 1 A.1 Solutions

Page 2: The In nite Actuary Exam 1/P Online Seminar A.1 ... · The In nite Actuary Exam 1/P Online Seminar A.1 Fundamentals of Probability Solutions Last updated February 23, 2013 1. Let

and we want P[L ∩ S]

P[L ∪ S] = P[L] + P[S]− P[L ∩ S]

0.65 = 0.40 + 0.30− P[L ∩ S]

P[L ∩ S] = 0.05

5. We are given 0.7 = P[A∪B] = P[A]+P[B]−P[AB] and 0.9 = P[A∪B′] = P[A]+P[B′]−P[AB′].Summing gives us 1.6 = 2P [A] + P [B] + P [B′] − (P[AB] + P[AB′]). But P[B] + P[B′] = 1 and

P[AB] + P[AB′] = P[A], so the right hand side is P[A] + 1. Subtracting 1 gives us P[A] = 0.6 .Or, 0.7 = P[A ∪B] so 1− 0.7 = 0.3 = P[A′B′], and P[A ∪B′] = 0.9 so 1− 0.9 = 0.1 = P[A′B].

Adding those gives us 0.4 = P[A′] and P[A] = 1− 0.4 = 0.6 This method is easier to follow if you

draw the Venn diagram:

A B

0.1

0.3

6. Let T denote the event that a patient visits a physical therapist, and C the event that a patientvisits a chiropractor. We are given:

P[T ∩ C] = 0.22

P[(T ∪ C)′] = 0.12 P[T ∪ C] = 1− 0.12 = 0.88

P[C] = 0.14 + P[T ]

and we want P[T ]

P[T ∪ C] = P[C] + P[T ]− P[T ∩ C]

0.88 = 0.14 + P[T ] + P[T ]− 0.22

P[T ] = 0.48

7. A′ ∪B′ means that it is either not in A or not in B, which includes everything except AB (i.e.,

A′ ∪B′ = (A ∩B)′), so P[A′ ∪B′] = 1− P[AB] = 1− 0.2 = 0.8

8. Let S be the event that a student took the SAT and A the event that a student took the ACT.We know P[AS] = 0.3 and P[S] = 0.75, so P[A′S] = 0.75−0.3 = 0.45 and P[AS ′] = 0.4−0.3 = 0.1.

Combining these gives us 0.45 + 0.10 = 0.55

9.

G B

S

.08

.14− .08

= 0.06

.04.02

.19− .14 = .05

.12 .11

c©2013 The Infinite Actuary, LLC p. 2 A.1 Solutions

Page 3: The In nite Actuary Exam 1/P Online Seminar A.1 ... · The In nite Actuary Exam 1/P Online Seminar A.1 Fundamentals of Probability Solutions Last updated February 23, 2013 1. Let

We want the probability that someone watches none of these sports, so we want

1− 0.12− 0.06− 0.11− 0.02− 0.08− 0.04− 0.05 = 0.52

10. There are 3,000 young policy holders. 1,320 are male, leaving 1,680 young female policyholders. There are 1,400 young married policyholders, 600 of whom are male, so there are 800young female married policyholders, leaving 1,680−800 = 880 young female single policyholders.

11. Using the fact that R and H are mutually exclusive, and then given properties (i), (iv), (v)and (ii) gives the following Venn diagram:

H

R

A

0.35− x

x

0.29

0.11

y

0.17

1 = 0.35− x + 0.11 + 0.29 + x + y + 0.17

1 = 0.92 + y, 0.08 = y

2(x + y) = 0.46− x by given (iii)

2x + 0.16 = 0.46− x

x = 0.10

12. P[K] = 3P[T ]

P[Q] =1

4(P[K] + P[T ])

=1

4(3P[T ] + P[T ]) = P[T ]

1 = P[K] + P[Q] + P[T ]

= 3P[T ] + P[T ] + P[T ]

P[T ] = 0.2

P[K ∪Q] = P[K] + P[Q] = 4P[T ] = 0.80

13. The Venn diagram we get is

C N

A

4

3

12

3

25 7

H18

Subtracting the pieces we have solved for from 100 gives

100− 25− 3− 7− 2− 4− 1− 3− 18 = 37

14. Let G denote those who read the Globe, H those who read the Herald, and P those who read

c©2013 The Infinite Actuary, LLC p. 3 A.1 Solutions

Page 4: The In nite Actuary Exam 1/P Online Seminar A.1 ... · The In nite Actuary Exam 1/P Online Seminar A.1 Fundamentals of Probability Solutions Last updated February 23, 2013 1. Let

the Phoenix.G H

P

0.03

0.06

yx

.4− .03− x− y

.25− .09− x .29− .09− y

0.25

We want x + y, and using that everything sums to 1 gives

1 = (0.25− 0.09− x) + x + 0.06 + 0.03 + (0.29− 0.09− y) + y

+ 0.25 + (0.4− 0.3− x− y)

= 1.07− x− y

x + y = 0.07

15. In general, all we know about probabilities is that they can range from 0 to 1. To getrestrictions on P[A ∩B] we want an equation that combines P[A ∩B], along with the things thatwe know, namely P[A] and P[B], and gives us a probability. In particular, we can say that

0 ≤ P[A ∪B] ≤ 1

P[A ∪B] = P[A] + P[B]− P[AB]

0 ≤ 0.7 + 0.6− P[AB] ≤ 1

−1.3 ≤ −P[AB] ≤ −0.3

1.3 ≥ P[AB] ≥ 0.3

That gives us a useful lower bound of P[AB] ≥ 0.3Note that this bound can be obtained if P[AB] = 0.3, P[AB′] = 0.4, P[A′B] = 0.3 and

P[A′B′] = 0.

16. The problem with the approach we used in the previous problem is that it gave the upperbound of P[AB] ≤ 1.3, which we already knew since it is a probability and hence no more than 1.But we also know that AB ⊂ A and AB ⊂ B so P[AB] ≤ P[A] and P[AB] ≤ P[B]. That givesus P[AB] ≤ 0.6 and P[AB] ≤ 0.7. Taking the most restrictive of those gives us the maximum

possible value of P[AB] of 0.6This upper bound can be realized if B ⊂ A.

17. Let C, T and S denote the event of being a car, train, and spaceship respectively. We are toldP[C] = 2P[T ], so P[T ] = (0.5)P[C]. Likewise, P[S] = 2(P[C] + P[T ]) = 2(1 + 0.5)P[C] = 3P[C].Since the total probability is 1, and these events are disjoint, we have 1 = P[T ] + P[C] + P[S] =

4.5P[C], so P[C] = 2/9

18. You could do it by explicitly listing all the possibilities. Alternatively, there are 5 numbers inthe set that are divisible by 5, 12 that are even, and 2 (10 and 20) that are both, so P[even] = 12/25,P[divisible by 5] = 5/25 and P[both] = 2/25, so P[even or divisible by 5, but not both] = (12 +

5− 2 · 2)/25 = 13/25

c©2013 The Infinite Actuary, LLC p. 4 A.1 Solutions

Page 5: The In nite Actuary Exam 1/P Online Seminar A.1 ... · The In nite Actuary Exam 1/P Online Seminar A.1 Fundamentals of Probability Solutions Last updated February 23, 2013 1. Let

19. Let A denote those who own an automobile and H those who own a house. Then P[A] = 0.6,

P[H] = 0.3, and P[AH] = 0.2. We want P[A ∪H]− P[AH] = (0.6 + 0.3− 0.2)− 0.2 = 0.5

20.

A

B

C D

P[A] + P[B] + P[C] + P[D] = P[A ∪B ∪ C ∪D] + P[AD] + P[AC] + P[BC] + P[BD]

0.7 + 0.6 = 0.9 + P[AD] + P[AC] + P[BC] + P[BD]

0.4 = P[AD] + P[AC] + P[BC] + P[BD]

P[exactly 1 of the 4 events] = 0.9− 0.4 = 0.5

21.

L

S

F G

x xy

3y 2y

2x 2x + y2x + 5y

From (iii), we know that L and S are disjoint, hence they are non-overlapping sets in our Venndiagram. Likewise, F and G are also disjoint.

From (v) and (vi), let x be the number who take French and Latin, so 2x is the number whotake Spanish and French and x is the number who take German and Latin.

Let y be the number who take Latin only. (iv) tells us that the number who take German onlyis 2y, and (vii) tells us that the total number who take German and Spanish equals the numberwho take Latin only, which is 2x + y from our diagram.

We now see that 3x + 3y take German, so from (ii), we also have 3x + 3y who take French.3x take French plus another language, so 3y take French only. The number who take Spanish is2(x + 2y + 2x + y) = 6x + 6y, so 6x + 6y − 2x− (2x + y) = 2x + 5y take Spanish only.

Finally, twice as many take German as Latin, so 2(2x+y) = (3x+ 3y) and so x = y. There are20x who take at least one language, and 7x who take more than 1, so P[more than 1 language] =

7x/(20x) = 0.35

c©2013 The Infinite Actuary, LLC p. 5 A.1 Solutions

Page 6: The In nite Actuary Exam 1/P Online Seminar A.1 ... · The In nite Actuary Exam 1/P Online Seminar A.1 Fundamentals of Probability Solutions Last updated February 23, 2013 1. Let

22.

T

E

Fire F lood

2z

2x

z

x

.05

y

0.1

y

From (i), we can draw T and E as disjoint, but need to keepsome overlap between Fire and Flood. Since no one has 3 coverages, that overlap is outside ofboth T and E, hence the basic shape of our Venn diagram.

Let x be T only, so 2x is fire only by (iii). Let y be flood and T , so flood and E is also y (by iv).Let z be fire and E, so 2z is fire and T by (v). Finally, 0.35 = P[E] so P[E only] = 0.35− y − z

We can then get the system of equations

0.53 = 2x + x + 0.1 + 0.35− y − z, 0.08 = 3x− y − z

0.35 = 2z + 2x + z + 0.05, 0.30 = 3z + 2x

1 = 3x + 2z + y + 0.5, 0.50 = 3x + y + 2z

from (ii), (vii), and the fact that all the probabilities sum to 1. Solving gives x = 0.09

23. One approach is to draw a Venn diagram for each combination and compare. It turns out thatthe (i) and (ii) are the same as the original expression, so the answer is A and the Venn diagram

for those is

A B

C

Expression (iii) is different since it also includes everything outside of A ∪B ∪ C.

24. P[A ∪ B] − P[A ∩ B] = (P[A] + P[B] − P[AB]) − P[AB] = 1.6 − 2P[AB] so it is maximizedwhen P[AB] is minimized. Since 1 ≥ P[A ∪ B] = P[A] + P[B] − P[AB] = 1.6 − P[AB], we have

P[AB] ≥ 0.6 and so the minimum of our quantity is 1.6− 2 · 0.6 = 0.4

25. One approach to problems like this is to let one die be red, one blue (so we can tell themapart), list all 36 cases, and count.

Using our probability rules, there are 11 ways to get at least one 6, and 3 ways to roll an 8without a 6 (red 3 and blue 5, or both 4, or red 5 and blue 3). Since those are disjoint, we can

sum the probabilities for a final answer of (11/36) + (3/36) = 14/36 .

c©2013 The Infinite Actuary, LLC p. 6 A.1 Solutions

Page 7: The In nite Actuary Exam 1/P Online Seminar A.1 ... · The In nite Actuary Exam 1/P Online Seminar A.1 Fundamentals of Probability Solutions Last updated February 23, 2013 1. Let

26.

A B

C

0.1

0.3− 0.1

= 0.2

0.10.2

0.1

0.1 0.1

From the Venn diagram, we see the answer is 0.9

27.

A B

C

0.1

0.3− 0.1

= 0.2

0.10.2

0.1

0.1 0.1

A′B′C means everything that is in C but in neither A nor B, so it is just the shaded regionwith probability 0.1.

c©2013 The Infinite Actuary, LLC p. 7 A.1 Solutions

Page 8: The In nite Actuary Exam 1/P Online Seminar A.1 ... · The In nite Actuary Exam 1/P Online Seminar A.1 Fundamentals of Probability Solutions Last updated February 23, 2013 1. Let

The Infinite Actuary Exam 1/P Online SeminarA.2 Conditional Probability SolutionsLast updated February 6, 2013

1. Note that there are 6 ways to get a 7 (1 and 6, 2 and 5, 3 and 4, 4 and 3, 5 and 2, or 6 and 1)and 5 ways to get an 8 (2 and 6, 3 and 5, 4 and 4, 5 and 3, or 6 and 2). There are 6 · 6 = 36 totalcombinations for 2 dice, which gives

P[X = 8 | X = 7 or 8] =P[X = 8]

P[X = 7 or 8]

=5/36

(5 + 6)/36=

5

11

2. Let H denote those with health coverage and D those with dental. Then

P[HD] = P[H] · P[D | H] = 0.5 · 0.6 = 0.3

P[H ′D] = 0.15

P[D] = P[HD] + P[H ′D] = 0.30 + 0.15 = 0.45

3.

A0.5

Ac

B

0.4

Bc

0.1

0.3

0.4

0.2

P[B | A] =P[AB]

P[A]=

0.1

0.5= 0.2

P[A | B] = 0.05 + P[B | A]

P[AB]

P[B]=

0.1

P[B]= 0.05 + 0.20 = 0.25

P[B] = 0.4

4. Drawing a grid gives

Insure1 car

Insure morethan 1 car 70%

Sports Car20%

No SC

10.5%

9.5% 20.5%

The probability of insuring more than one car and a sports car is 0.7 · 0.15 = 0.105, andsince 20% insure a sports car, the probability of insuring exactly one car that is a sports car is0.20 − 0.105 = 0.095, leaving 0.3 − 0.095 = 0.205 as the chance of insuring exactly one car thatis not a sports car.

5. Let C denote those purchasing collision coverage, and D those with disability coverage. Then

0.15 = P[CD] = P[C] · P[D] by independence

0.15 = 2P[D] · P[D]

c©2013 The Infinite Actuary, LLC p. 1 A.2 Solutions

Page 9: The In nite Actuary Exam 1/P Online Seminar A.1 ... · The In nite Actuary Exam 1/P Online Seminar A.1 Fundamentals of Probability Solutions Last updated February 23, 2013 1. Let

P[D] = 0.274, P[C] = 0.548

1− P[C ∪D] = 1− (P[C] + P[D]− P[CD]) = 0.33

6.

At least 1 parentwith heart disease 312

Neither parentwith heart disease 625

HD210

No HD727

102

108

210

517

108

625= 0.173

7.

A only0.65− 0.15

= 0.5

B only0.35

Both0.15

Renew0.4

0.6

0.8

0.5 · 0.4 + 0.35 · 0.6 + 0.15 · 0.8 = 0.53

8.

regular0.85

irregular0.15

low0.22

high0.14

normal0.64

0.05

0.09

0.080.02

0.560.20

9. Let b denote the number of blue balls in the second urn. Then

P[both are same color] =6

10· b

16 + b+

4

10· 16

16 + b

0.44 =6b + 64

10(16 + b)

b = 4

10. Let O, F , and E denote orthodontic work, fillings, and extractions respectively. Then we aregiven

1

2= P[O]

2

3= P[O ∪ F ] = P[O] + P[F ]− P[OF ]

2

3= P[O] + P[F ]− P[O]P[F ] =

1

2+

(1− 1

2

)P[F ]

c©2013 The Infinite Actuary, LLC p. 2 A.2 Solutions

Page 10: The In nite Actuary Exam 1/P Online Seminar A.1 ... · The In nite Actuary Exam 1/P Online Seminar A.1 Fundamentals of Probability Solutions Last updated February 23, 2013 1. Let

1

3= P[F ]

3

4= P[O ∪ E] = P[O] + P[E]− P[OE]

3

4=

1

2+

(1− 1

2

)P[E]

1

2= P[E]

P[E ∪ F ] = P[E] + P[F ]− P[EF ]

=1

2+

1

3− 1

8=

17

24

11. The probability of rolling two 4s and the first roll being a 4 is the same as the probability ofrolling two 4s (the second condition is redundant), so

P[roll two 4s | 1st roll is a 4] =P[roll two 4s]

P[1st roll is a 4]

=

1

3· 1

6· 1

6+

1

3· 6

6· 6

6+

1

3· 2

6· 2

61

3· 1

6+

1

3· 6

6+

1

3· 2

6

=41

54= 0.76

12. A conditional probability is still just a probability, so all of our basic laws of probability stillhold. P[A ∪ B | C] = P[A | C] + P[B | C] − P[AB | C] ≤ P[A | C] + P[B | C] = 0.10, so theinequality in E is backwards. But A′B′ is the complement of A∪B, so P[A′B′ | C] = 1−P[A∪B |C] ≥ 1− 0.10 = 0.90 and the answer is B

Note that C is false since the inequality is backwards, and A and D need not hold because wecan’t just multiply probabilities without knowing things about the independence of A and B.

13. Since C and D are disjoint, P[C ∪D] = P[C] + P[D].Since A and B are complements, we have

P[C] = P[AC] + P[BC]

= P[C | A] · P[A] + P[C | B] · P[B]

=1

2· 1

4+

1

8· 3

4=

7

32P[D] = P[AD] + P[BD]

= P[D | A] · P[A] + P[D | B] · P[B]

=1

4· 1

4+

1

8· 3

4=

5

32

P[C ∪D] =12

32=

3

8

c©2013 The Infinite Actuary, LLC p. 3 A.2 Solutions

Page 11: The In nite Actuary Exam 1/P Online Seminar A.1 ... · The In nite Actuary Exam 1/P Online Seminar A.1 Fundamentals of Probability Solutions Last updated February 23, 2013 1. Let

14. Let H, L, and N denote heavy, light, and non-smokers respectively. Then

P[H | D] =P[HD]

P[D]

=P[HD]

P[HD] + P[LD] + P[ND]

=0.2P[D | H]

0.2P[D | H] + 0.3P[D | L] + 0.5P[D | N ]

=0.2 · 2 · P[D | L]

0.2 · 2 · P[D | L] + 0.3 · P[D | L] + 0.5 · 0.5 · P[D | L]

= 0.42

15. Let T (for teen), Y (for young), M , and O denote our 4 age ranges, and let A be the eventthat the driver has an accident. We want

P[T | A] =P[T ∩ A]

P[A]

=P[T ] · P[A | T ]

P[T ] · P[A | T ] + P[Y ] · P[A | Y ] + P[M ] · P[A |M ] + P[O] · P[A | O]

=0.08 · 0.06

0.08 · 0.06 + 0.15 · 0.03 + 0.49 · 0.02 + 0.28 · 0.04

= 0.158

16. Let U denote someone who is ultra-preferred.

P[U | died] =P[U and died]

P[died]

=0.10 · 0.001

0.50 · 0.010 + 0.40 · 0.005 + 0.10 · 0.001

=0.0001

0.0071= 0.0141

17. Let E denote those with ER charges and O those with OR charges. Then

E

Ec0.25

O

0.4

Oc

0.6

0.3

0.1

0.45

0.15

P[EcOc] = P[Ec] · P[Oc]

0.15 = 0.25P[Oc]

c©2013 The Infinite Actuary, LLC p. 4 A.2 Solutions

Page 12: The In nite Actuary Exam 1/P Online Seminar A.1 ... · The In nite Actuary Exam 1/P Online Seminar A.1 Fundamentals of Probability Solutions Last updated February 23, 2013 1. Let

0.6 = P[Oc]

P[O] = 1− 0.6 = 0.4

18. Let C, Sr, and St denote critical, serious, and stable patients, respectively. Let L denote apatient who lived. Then we want

P[Sr | L] =P[Sr, L]

P[L]

=0.30 · 0.90

0.10 · 0.60 + 0.30 · 0.90 + .60 · 0.99= 0.292

19. Let T, Y,M and S denote teen, young, midlife, and senior drivers respectively, and C theevent that a driver has a collision. Then

P[Y | C] =P[Y,C]

P[C]

=P[Y C]

P[TC] + P[Y C] + P[MC] + P[SC]

=0.16 · 0.08

0.08 · 0.15 + 0.16 · 0.08 + 0.45 · 0.04 + 0.31 · 0.05

= 0.2196

20.P[N ≥ 1 | N ≤ 4] =

P[N = 1, 2, 3 or 4]

P[N = 0, 1, 2, 3 or 4]

=

1

2 · 3+

1

3 · 4+

1

4 · 5+

1

5 · 61

1 · 2+

1

2 · 3+

1

3 · 4+

1

4 · 5+

1

5 · 6

=2

5

21.P[have disease | positive test] =

P[have disease and positive test]

P[positive test]

=0.01 · 0.95

0.01 · 0.95 + 0.99 · 0.005

= 0.657

22. Let C denote someone with a circulation problem, and S a smoker. Then P[C] = 0.25,P [C ′] = 1− 0.25 = 0.75, and P[S | C] = 2P[S | C ′] so

P[C | S] =P[CS]

P[S]

=P[C] · P[S | C]

P[C] · P[S | C] + P[C ′] · P[S | C ′]

=0.25 · 2P[S | C ′]

0.25 · 2P[S | C ′] + 0.75 · P[S | C ′]

c©2013 The Infinite Actuary, LLC p. 5 A.2 Solutions

Page 13: The In nite Actuary Exam 1/P Online Seminar A.1 ... · The In nite Actuary Exam 1/P Online Seminar A.1 Fundamentals of Probability Solutions Last updated February 23, 2013 1. Let

=0.5

0.5 + 0.75= 2/5

23. P[97 | A, 97− 99] =P[97, A]

P[A, 97− 99]

=0.16 · 0.05

0.16 · 0.05 + 0.18 · 0.02 + 0.20 · 0.03

= 0.45

24. Let L denote those with liability coverage, and P those with property coverage. ThenP[L ∪ P ] = P[L ∩ P ′] + P[P ] = 0.01 + 0.10 = 0.11, so P[(L ∪ P )′] = 1− 0.11 = 0.89

25. Let S denote a smoker, and D the event that someone died. Then

P[S | D] =P[SD]

P[D]

=P[SD]

P[SD] + P[ScD]

=0.10 · 0.05

0.10 · 0.05 + 0.90 · 0.01

= 0.357

26. The fastest way to do this is to use the complement. P[R ∪ S ∪ T ] = 1 − P[R′ ∩ S ′ ∩ T ′] =

1− P[R′] · P[S ′] · P[T ′] = 1− (2/3)3 =19

27

27. P[makes first] = 0.44

P[makes 2nd | makes first] = 0.55

P[makes both] = 0.44 · 0.55

= 0.242

28. Let A be the event that he made the previous shot, and B the event that he makes the currentshot. Then

P[A] = P[B] = 0.44

P[B | A] = 0.55

and we want P[B | A′]. Note that

P[B] = P[A]P[B | A] + P[A′]P[B | A′]

0.44 = 0.44 · 0.55 + 0.56 · P[B | A′]

0.354 = P[B | A′]

Another way to think about it is to suppose he took 1,000 shots over the course of a year. Hemade 44% of them, or 440 shots, leaving 560 misses. He makes 55% of the shots when he made the

c©2013 The Infinite Actuary, LLC p. 6 A.2 Solutions

Page 14: The In nite Actuary Exam 1/P Online Seminar A.1 ... · The In nite Actuary Exam 1/P Online Seminar A.1 Fundamentals of Probability Solutions Last updated February 23, 2013 1. Let

previous shot. There were 440 times when he made the previous shot, he made 55% of those, or 242.That likewise means there were 440-242 = 198 times he made a short after missing the perviousshot. There are 560 total misses, 198 were followed by a make, so P[Make | missed previous shot] =35.35%

29.

A0.4

Ac0.6

B

0.6

Bc

0.4

0.3

0.3

0.1

0.3

P[AB] = P[A]P[B | A] = 0.4 · 0.75 = 0.3

30. See the previous problem for the Venn diagram. We get

P[A | B] =P[AB]

P[B]=

0.3

0.6=

1

2

P[A | B′] =P[AB′]

P[B′]=

1

4

31.P[1st is blue | 2nd is blue] =

P[both are blue]

P[2nd is blue]

=

7

12· 4

87

12· 4

8+

5

12· 3

8

=28

43= 0.65

32. We want the probability of the shaded set:

A B

C

The complement is easier to find: we want 1 − P[ABC ′] = 1 − P[A] ·P[B] · P[C ′] = 1− 0.5 · 0.6 · (1− 0.1) = 0.73

33. Let A be the event that he breaks 80 in 2007, and B the event that he breaks 80 in 2008.Then P[A] = 0.4,P[B] = 0.6 and P[AB] = 0.3. We want

P[B | A] =P[AB]

P[A]=

0.3

0.4= 0.75

c©2013 The Infinite Actuary, LLC p. 7 A.2 Solutions

Page 15: The In nite Actuary Exam 1/P Online Seminar A.1 ... · The In nite Actuary Exam 1/P Online Seminar A.1 Fundamentals of Probability Solutions Last updated February 23, 2013 1. Let

34. Let A be the event that I arrive late, and B the event that I leave late. Then

P[B | A] =P[AB]

P[A]=

P[AB]

0.75

so the max is when P[AB] is maximized, and the min is when P[AB] is minimized. That gives us

0.55

0.75≤ P[B | A] ≤ 0.75

0.75

so our answer is 0.55/0.75 = 0.73

35. 1

2= P[B | A] =

P[AB]

P[A]=

P[AB]

1/41

8= P[AB]

1

3= P[C | AB] =

P[ABC]

1/81

24= P[ABC]

36. Let A be the event that a person has the disease, and B the event that they test positive. Wewant

P[A | B] =P[AB]

P[B]

=P[A] · P[B | A]

P[A] · P[B | A] + P[B] · P[A | B]

=0.01 · 0.85

0.01 · 0.85 + 0.99 · 0.10

= 0.079

37. P[A | {1, 1, 2, 3, 1}] =P[A, {1, 1, 2, 3, 1}]

P[{1, 1, 2, 3, 1}]

=

1

2·(

2

6

)5

1

2·(

2

6

)5

+1

2·(

43

63· 12

62

)=

1

3

38. P[R | R] =P[RR]

P[R]

0.54 =

n

12· 0.32 +

12− n

12· 0.62

n

12· 0.3 +

12− n

12· 0.6

c©2013 The Infinite Actuary, LLC p. 8 A.2 Solutions

Page 16: The In nite Actuary Exam 1/P Online Seminar A.1 ... · The In nite Actuary Exam 1/P Online Seminar A.1 Fundamentals of Probability Solutions Last updated February 23, 2013 1. Let

0.54(7.2− 0.3n) = 4.32− 0.27n

n = 4

39. P[At least one event] = 1− P[Neither Event]

= 1− (1− 0.1)(1− 0.3)

= 0.37

c©2013 The Infinite Actuary, LLC p. 9 A.2 Solutions

Page 17: The In nite Actuary Exam 1/P Online Seminar A.1 ... · The In nite Actuary Exam 1/P Online Seminar A.1 Fundamentals of Probability Solutions Last updated February 23, 2013 1. Let

The Infinite Actuary Exam 1/P Online SeminarA.3 Discrete Moments SolutionsLast updated July 17, 2012

1. Let P denote the payment amount. Comparing X and P , we haveX P Probability

1 1006− 1

15

2 2006− 2

15

3 3006− 3

15

4 3256− 4

15

5 3506− 5

15

E[P ] = 100 · 6− 1

15+ 200 · 6− 2

15+ 300 · 6− 3

15+ 325 · 6− 4

15+ 350 · 6− 5

15=

3200

15

= 213.33

2. E[X] =∑x

x · P[X = x]

= (20)(0.15) + (30)(0.10) + · · ·+ (80)(0.30) = 55

E[X2] = 202(0.15) + (30)2(0.10) + · · ·+ 802 · 0.3 = 3,500

Var[X] = E[X2]− (E[X])2 = 3,500− 552 = 475

SD[X] =√

Var[X] = 21.8

So the mean ±1 SD is from 33 to 77. The possible claims in that range are 40, 50, 60, and 70, and

summing their probabilities gives us 45%

3. Let X denote the loss amount. P[X = x | X > 0] = P[X = x]/(1− 0.9) for x > 0, sox P[X = x] P[X = x | X > 0]0 0.900 0.000

500 0.060 0.6001,000 0.030 0.30010,000 0.008 0.08050,000 0.001 0.010100,000 0.001 0.010

E[loss | loss > 0] = 500(0.6) + 1,000(0.3) + 10,000(0.08) + 50,000(0.01) + 100,000(0.01)

= 2,900

c©2012 The Infinite Actuary, LLC p. 1 A.3 Solutions

Page 18: The In nite Actuary Exam 1/P Online Seminar A.1 ... · The In nite Actuary Exam 1/P Online Seminar A.1 Fundamentals of Probability Solutions Last updated February 23, 2013 1. Let

4. Given that X = 0, there are 9 remaining cases, which we can list out:

y z P[Y = y, Z = z,X = 0] P[Y = y, Z = z | X = 0] Unreimbursed Accidents0 0 0 0 0

0 1 kk

27k0

0 2 2k2k

27k0

1 0 2k2k

27k0

1 1 3k3k

27k0

1 2 4k4k

27k1

2 0 4k4k

27k0

2 1 5k5k

27k1

2 2 6k6k

27k2

The P[Y = y, Z = z | X = 0] column was found by computingP[Y = y, Z = z,X = 0]

P[X = 0], and

P[X = 0] = 27k by summing the 3rd column. The final column is (y + z − 2) if y + z > 2, and is0 otherwise.

The expected value of the final column is then

1 · 4

27+ 1 · 5

27+ 2 · 6

27=

21

27=

7

9

5. Let X denote the price of a randomly selected toy. Then

P[X = 5] = 2 · P[X = 10]

P[X = 5] =1

2· P[X = 25]

1 = P[X = 5] + P[X = 10] + P[X = 25]

1 = P[X = 5] +1

2· P[X = 5] + 2 · P[X = 5]

1 = 3.5 · P[X = 5]

P[X = 5] =2

7

E[X] = 5 · 2

7+ 10 · 1

7+ 25 · 4

7=

120

7= 17.14

6. The sum of all the probabilities is 1, so 1 = 0.2 + 0.3 + 0.4 + p and p = 0.1. Then

E[X] =∑

x · P[X = x] = 1 · 0.2 + 3 · 0.3 + 5 · 0.4 + 9 · 0.1 = 4

c©2012 The Infinite Actuary, LLC p. 2 A.3 Solutions

Page 19: The In nite Actuary Exam 1/P Online Seminar A.1 ... · The In nite Actuary Exam 1/P Online Seminar A.1 Fundamentals of Probability Solutions Last updated February 23, 2013 1. Let

E[X2] =∑

x2 · P[X = x] = 12 · 0.2 + 32 · 0.3 + 52 · 0.4 + 92 · 0.1 = 21

Var[X] = 21− 42 = 5

7. First, let’s make a table of the possible values and their probabilities. As we aren’t initiallygiven µX , we will first find it and then fill in that column of the table.

x P[X = x] |x− µX |1 1/6 2.52 1/6 1.53 1/6 0.54 1/6 0.55 1/6 1.56 1/6 2.5

µX = E[X] = 1 · 1

6+ 2 · 1

6+ 3 · 1

6+ 4 · 1

6+ 5 · 1

6+ 6 · 1

6

=7

2= 3.5

E |X − µx| = 2.5 · 1

6+ 1.5 · 1

6+ 0.5 · 1

6+ 0.5 · 1

6+ 1.5 · 1

6+ 2.5 · 1

6

= 1.5

8. Again, let’s make a table of the possible values. Because we want to find the median, we willwant a column for F (x) = P[X ≤ x] since the median is the smallest x that makes F (x) > 1/2

x P[X = x] P[X ≤ x] |x−m(X)|0 1/4 1/4 52 1/8 3/8 35 1/4 5/8 06 3/8 1 1

The median m(x) is 5 since that is the first time P[X ≤ x] > 1/2. Next, we need to find themedian of |X − 5|. There are 4 possible values of |X − 5|. It is 0 with probability 1/4, 1 with

probability 3/8, 3 with probability 1/8, and 5 with probability 1/4. P[|X−5| ≤ 0] =1

4<

1

2, while

P[|X − 5| ≤ 1] =1

4+

3

8=

5

8>

1

2, so the first time P[|X − 5| ≤ t] > 1/2 is when t = 1, giving us

an answer of 1

9. Let x denote the value of the 6th face, and Y the outcome of a roll. Then

4 = E[Y ] =1

6·[2 + 2 + 3 + 5 + 7 + x

]24 = 19 + x

x = 5

10. E[(2Y + 1)2

]= E

[4Y 2 + 4Y + 1

]c©2012 The Infinite Actuary, LLC p. 3 A.3 Solutions

Page 20: The In nite Actuary Exam 1/P Online Seminar A.1 ... · The In nite Actuary Exam 1/P Online Seminar A.1 Fundamentals of Probability Solutions Last updated February 23, 2013 1. Let

= 4E[Y 2] + 4E[Y ] + 1

So we first need to find E[Y 2]. To do that, we use

Var[Y ] = E[Y 2]− (E[Y ])2

3 = E[Y 2]− 22

E[Y 2] = 7

and plugging back in gives

E[(2Y + 1)2

]= 4 · 7 + 4 · 2 + 1 = 37

11. Var[3Y + 4] = Var[3Y ]

= 32 · Var[Y ]

= 32 · 3 = 27

12. E[(X − 23] = E[X3 − 6X2 + 12X − 8]

E[(X − 2)3] = E[X3]− 6E[X2] + 12E[X]− 8

0 = 9− 6E[X2] + 12 · 2− 8

E[X2] =25

6

Var[X] = E[X2]− (E[X])2 =25

6− 22

Var[X] =1

6

13. Let’s list the possible values of X and X − µX (again, first we find µX and then fill in thatcolumn of the table).

x P[X = x] x− µX

1 1/4 −43 1/4 −28 1/2 3

E[X] = 1 · 1

4+ 3 · 1

4+ 8 · 1

2= 5

Var[X] = E[X2]− (E[X])2 = E[(X − µx)2]

=1

4(−4)2 +

1

4(−2)2 +

1

232 = 9.5

SD[X] =√

Var(X) =√

9.5

E[(X − µx)3] =1

4(−4)3 +

1

4(−2)3 +

1

2(3)3 = −4.5

Skewness[X] =E[(X − µX)3]

[SD(X)]3

c©2012 The Infinite Actuary, LLC p. 4 A.3 Solutions

Page 21: The In nite Actuary Exam 1/P Online Seminar A.1 ... · The In nite Actuary Exam 1/P Online Seminar A.1 Fundamentals of Probability Solutions Last updated February 23, 2013 1. Let

=−4.5

9.53/2= −0.15

14. We have the same distribution as the previous problem.

E[(X − µx)4] =1

4(−4)4 +

1

4(−2)4 +

1

2(3)4 = 108.5

Kurtosis(X) =E[(X − µX)4]

[SD(X)]4=

108.5

9.54/2= 1.20

15. Again, we found SD[X] and E[X] in the previous problems. Then

CV[X] =SD[X]

E[X]=

√9.5

5= 0.616

16. 1 = 0.2 + 0.3 + a+ b

0.5 = a+ b

5 = E[X]

5 = 0.2 · 1 + 0.3 · 2 + 6a+ 9b

a = 0.1 b = 0.4

E[X2] = 0.2 · 12 + 0.3 · 22 + 0.1 · 62 + 0.4 · 92

E[X2] = 37.4

Var[X] = −52 = 12.4

17. P[N = n] = cn, and we know that the sum of the probabilities is 1, so

1 = c · 1 + c · 2 + c · 3 + c · 4 = 10 · cc = 0.1

E[(N − 2)2

]=

4∑n=1

(n− 2)2 · P[N = n] =4∑

n=1

(n− 2)2 · 0.1n

= 0.1 · (1− 2)2 + 0.2 · (2− 2)2 + 0.3 · (3− 2)2 + 0.4 · (4− 2)2 = 2

18. Let X be the number of people who pick that team as their favorite. Then the distribution ofX is as follows:

x P[X = x]3 0.212 0.224 0.227 0.234 0.2

so E[X] = 0.2 · 3 + 0.2 · 12 + · · ·+ 0.2 · 34 = 20

19. Let X be the number of people who pick that team as their favorite. Now P[X = x] = x/100,so the distribution of X is as follows:

c©2012 The Infinite Actuary, LLC p. 5 A.3 Solutions

Page 22: The In nite Actuary Exam 1/P Online Seminar A.1 ... · The In nite Actuary Exam 1/P Online Seminar A.1 Fundamentals of Probability Solutions Last updated February 23, 2013 1. Let

x P[X = x]3 0.0312 0.1224 0.2427 0.2734 0.34

This means that E[X] = 0.03 · 3 + 0.12 · 12 + · · ·+ 0.34 · 34 = 26.14

20. N is uniform on {100, 101, . . . , 999}, so E[N ] =100 + 999

2= 549.5

21. Y is uniform on {1, 2, 3, 4} so E[Y ] = (1 + 4)/2 = 2.5 and Var[Y ] =42 − 1

12= 1.25.

E[25 + 100Y ] = 25 + 100E[Y ] = 275

Var[25 + 100Y ] = Var[100Y ]

= 1002Var[Y ] = 12,500

CV[25 + 100Y ] =

√12, 500

275= 0.407

22. E[X] = 5 =a+ b

2

Var[X] = 2 =(b− a+ 1)2 − 1

125 = b− a+ 1

10 = a+ b

15 = 2b+ 1

b = 7 a = 3

P[X ≤ 4] =2

5

c©2012 The Infinite Actuary, LLC p. 6 A.3 Solutions

Page 23: The In nite Actuary Exam 1/P Online Seminar A.1 ... · The In nite Actuary Exam 1/P Online Seminar A.1 Fundamentals of Probability Solutions Last updated February 23, 2013 1. Let

The Infinite Actuary Exam 1/P Online SeminarA.4 Combinatorics SolutionsLast updated August 2, 2012

1. There are two cases, based on the color that is transferred, and the answer is

7

10·

(5

2

)(

10

2

) +3

10·

(4

2

)(

10

2

) =44

225

2. There are a total of 3n counters in the urn, so there are

(3n

2

)ways to choose the two counters.

There are 3 ·(n

2

)ways for them to be the same color, and n ·

(3

2

)ways for them to be the same

number. There is no overlap between those possibilities as if they are the same color they cannotbe the same number.

Combining this gives us

3 ·(n

2

)+ n ·

(3

2

)(

3n

2

) =

3n(n− 1)

2+ 3n

3n(3n− 1)

2

=(n− 1) + 2

3n− 1=

n + 1

3n− 1

3. We want 3 aces and therefore 10 non-aces, for an answer of(4

3

)·(

48

10

)(

52

13

)4. He will plant 7 white bushes in a row if a) the 2 red bushes are both in the first 3, or b) thered bushes are both in the last 3 or c) the red bushes are 1st and 9th, 1st and 10th, or 2nd and

10th. There are

(3

2

)= 3 ways for a) to occur, and also the same number of ways for b) to occur.

c) also had 3 cases, so there are 9 ways that work. There are

(10

2

)ways to choose where to put

the red bushes for a final answer of

3 · 3(10

2

) =9

45=

1

5

5. We will keep on rolling until we get either a 7 or an 8. All we care about is the result of thelast roll, and since the rolls are independent and identically distributed, we can ignore all of therolls until then. The fastest way to do so is to condition on rolling a 7 or an 8. Then we want

P[X = 7 | X = 7 or 8] =P[X = 7]

P[X = 7 or 8]=

6/36

6/36 + 5/36=

6

11

c©2012 The Infinite Actuary, LLC p. 1 A.4 Solutions

Page 24: The In nite Actuary Exam 1/P Online Seminar A.1 ... · The In nite Actuary Exam 1/P Online Seminar A.1 Fundamentals of Probability Solutions Last updated February 23, 2013 1. Let

6. P[X | 1 ineffective vial] =P[X, 1 inneffective vial]

P[1 ineffective vial]

=(1/5) · 0.1 · 0.929 · 30

(1/5) · 0.1 · 0.929 · 30 + (4/5) · 0.02 · 0.9829 · 30

= 0.096 = 0.10

7. Let N be the number of employee’s who achieve high performance and get a bonus. The totalcost of the bonuses is therefore CN . First, let’s figure out the distribution of N .

P[N = 0] = (0.98)20 = 0.668

P[N = 1] =

(20

1

)(0.98)19(0.02)1 = 0.272

P[N ≤ 1] = 0.668 + 0.272 = 0.940

P[N = 2] =

(20

2

)(0.98)18(0.02)2 = .053

P[N ≤ 2] = 0.993

So there is a better than 99% chance that no more than 2 people will get a bonus. In other words,if C is chosen so that we can pay 2 bonuses, then we have more than a 99% chance of being safeand less than a 1% chance of the fund being inadequate.

But if we have 120 in the fund and need to cover 2 bonuses, then we can give 120/2 = 60 for

each of them, so C = 60 is the maximum possible payout.

8. There are 3 basic cases to have at least 2 more high risk losses than low risk losses, namely

Case I : All 4 are high risk

Case II : 3 are high risk, 1 is not

Case III : 2 are high risk, and 2 are moderate risk

Note that in Case II, the 1 who is not high risk can be either low risk or moderate risk. In caseIII, on the other hand, the 2 who are not high risk must be moderate risk since if either of themwere low risk, the number of high risk - number of low risk would be at most 1.

P[Case I] = (0.2)4

P[Case II] =

(4

3

)(0.2)3 (0.8)1

P[Case III] =

(4

2

)(0.2)2 (0.3)2

Answer = sum of 3 cases = 0.0488

9. Let X be the number of hurricanes in 20 years, so X has a binomial distribution with n = 20and p = 0.05. Then

P[X < 3] = P[X = 0] + P[X = 1] + P[X = 2]

c©2012 The Infinite Actuary, LLC p. 2 A.4 Solutions

Page 25: The In nite Actuary Exam 1/P Online Seminar A.1 ... · The In nite Actuary Exam 1/P Online Seminar A.1 Fundamentals of Probability Solutions Last updated February 23, 2013 1. Let

P[X = 0] =

(20

0

)(0.95)20 (0.05)0 = (0.95)20 = 0.358

P[X = 1] =

(20

1

)(0.95)19 (0.05)1 = 0.377

P[X = 2] =

(20

2

)(0.95)18 (0.05)2 = 0.189

P[X < 3] = 0.358 + 0.377 + 0.189 = 0.92

10. Let N1 be the number of people in group 1 who complete the study, and N2 the number ingroup 2 who complete the study. For either one of the groups,

P[Ni ≥ 9] =

(10

9

)(0.8)9 (0.2) +

(10

10

)(0.8)10 = 0.3758

P[N1 ≥ 9, N2 < 9] + P[N1 < 9, N2 ≥ 9] = 0.3758 · (1− 0.3758) + (1− 0.3758) · 0.3758 = 0.469

11. There are a total of 30 · 0.2 + 50 · 0.8 = 10 defective modems in the total inventory of 80.Because we are sampling without replacement, we want a hypergeometric distribution and not abinomial distribution, and our answer is(

10

2

)·(

70

3

)(

80

5

) = 0.102

12. Suppose that all 3 items are chosen from the first 3 rows. There would then be 5 ways tochoose the column for the first row, 4 for the column in the 2nd row, and 3 for the column in thethird row, or 60 ways.

But we don’t know that all 3 items come from the first 3 rows, just that they come from 3 of

the 6 different rows, giving

(6

3

)ways to choose the rows and(

6

3

)· 60 = 20 · 60 = 1200

total ways to do it.

13. We will have at least 3 kings if at least one of the three new cards is a king. When talkingabout “at least one” it is often easier to use a complement.

P[at least one more king] = 1− P[no more kings]

= 1−

(45

3

)(

47

3

) =135

1,081

14. There are

(52

5

)ways to choose 5 cards, so that is our denominator. For the numerator,

c©2012 The Infinite Actuary, LLC p. 3 A.4 Solutions

Page 26: The In nite Actuary Exam 1/P Online Seminar A.1 ... · The In nite Actuary Exam 1/P Online Seminar A.1 Fundamentals of Probability Solutions Last updated February 23, 2013 1. Let

there is 1 way to choose the king of spades,

(3

1

)ways to choose a second king,

(4

2

)ways to

choose two of the four queens, and

(44

1

)ways to choose one non-king/queen from the 52− 4− 4

non-king/queens, for a final answer of (3

1

)·(

4

2

)·(

44

1

)(

52

5

)

15. The range is 3 if we draw 1, (2 or 3), 4, or if we draw 2, (3 or 4), 5. So there are 2 · 2 = 4 ways

for the range to be 3, and

(5

3

)= 10 ways to choose the sample, for a final answer of 4/10 = 2/5

16. The probability of a specific person not responding is 0.5 · 0.6 = 0.3. Let N be the numberwho don’t respond. Then N is a binomial with n = 4 and p = 0.3, so

P[N ≥ 3] = 0.34 + 4 · 0.33 · 0.7

17. For the outcomes to be decreasing, they must all be different. There are

(6

3

)ways to pick

3 different values, and the probability of any one specific sequence occurring is (1/6)3, so the

probability is

(6

3

)· (1/6)3 of having a decreasing sequence of rolls.

That ignores the requirement that the product is even. The only way for the product to be

odd is to have the sequence be 5, 3, 1, so there are

(6

3

)− 1 = 19 ways for the product to be even,

giving us a final answer of 19 · (1/6)3 = 19/216

18. Let W be the number of white balls drawn. To have 1 red and 2 white means that we needW = 3− 1 = 2, so we want

P[W = 2 | W ≥ 2] =P[W = 2,W ≥ 2]

P[W ≥ 2]

=P[W = 2]

P[W = 2] + P[W = 3]

=

(4

1

)·(

6

2

)(

4

1

)·(

6

2

)+

(4

0

)·(

6

3

)=

4 · 15

4 · 15 + 20=

3

4

19. If the committee has all 3 women, then the probability of the chairperson being a woman is

c©2012 The Infinite Actuary, LLC p. 4 A.4 Solutions

Page 27: The In nite Actuary Exam 1/P Online Seminar A.1 ... · The In nite Actuary Exam 1/P Online Seminar A.1 Fundamentals of Probability Solutions Last updated February 23, 2013 1. Let

3/5, which gives

3

(3

3

)(97

2

)(

100

5

) =3 · 97! · 5!

5 · 2! · 100!=

3 · 4! · 97!

2 · 100!

20. Since X and Y are both either 0 or 1, the only way we don’t have X ≤ Y is if X = 1 andY = 0. So

P[X ≤ Y ] = 1− P[X > Y ] = 1− P[X = 1, Y = 0]

= 1− P[X = 1] · P[Y = 0 | X = 1]

= 1− 5

10· 2

11=

10

11

21. There are

(4

2

)ways of choosing which 2 dice are even and which are odd. The chance of two

matching odds is (1/2) · (1/6), and the chance of two matching evens is (1/2) · (1/6), giving(4

2

)· 1

2· 1

6· 1

2· 1

6=

1

24

22. Let R be the number of red balls drawn. We want P[R = 0 or 1]. There are 3 red balls and 7non-red balls, so

P[R = 0] = 0.75

P[R = 1] =

(5

1

)0.3 · 0.74

P[R = 0] + P[R = 1] = 0.75 + 1.5 · 0.74 = 0.5282

23. There are 3 cases: either both socks will be white, both will be red, or both will be black.

Drawing without replacement, there are

(12

2

)ways to draw 2 socks. There are

(4

2

)ways for both

to be white,

(6

2

)for both to be red, etc., giving

(4

2

)+

(6

2

)+

(2

2

)(

12

2

) =1

3

24. Drawing with replacement, the chance of 2 whites is now4

12· 4

12=

1

9, giving us

4

12· 4

12+

6

12· 6

12+

2

12· 2

12=

7

18

c©2012 The Infinite Actuary, LLC p. 5 A.4 Solutions

Page 28: The In nite Actuary Exam 1/P Online Seminar A.1 ... · The In nite Actuary Exam 1/P Online Seminar A.1 Fundamentals of Probability Solutions Last updated February 23, 2013 1. Let

25. Because we are sampling with replacement, each drawing is independent and so we get amultinomial distribution. (

5

1

)·(

4

2

)4

14

(8

14

)2(2

14

)2

=960

16,807

26. In one round, the probability of losing is1

2· 1

2=

1

4. The chance of exactly 3 losses in 5 plays

is thus (5

3

)·(

1

4

)3

·(

3

4

)2

= 10 · 32

45

27. The chance that there will be the same number of heads as tails is

(2k

k

)·(12

)2k. If the number

of heads and tails are different, it is equally likely that there are more heads than tails as it is thatthere are more tails than heads. So

P[More tails] =1

2·[1−

(2k

k

)· 1

22k

]=

1

2−(

2k

k

)· 1

22k+1

and the answer is D

28. We need the Fire to win games 6 and 7. But for the final total to be 4 games to 3, that meansthat the Fire only won 2 of the first 5.

P[Win 2 of first 5] · P[Win 6 and 7] =

(5

2

)· 1

25· 1

22=

10

128=

5

64

29. There are 6 total pairs of shoes in the closet, so we have 6 · 2 = 12 total shoes. Since we seethe phrase “at least” I want to consider using the complement: P[at least one matching pair] =1− P[no matching pair] and to have no matching pair, we get

P[No matching pair] =12

12· 10

11· 8

10· 6

9=

16

33

To see where that equation comes from, there are 12 choices for the first shoe that we pick, noneof which form a pair, so all 12 are okay for us. After we have picked one shoe, there are 11 shoesleft, 10 of which don’t match the first one. For the next shoe, there are 10 shoes left, 1 matchesthe first, one matches the second, so 8 don’t match either. Finally, for the last shoe there are 9shoes left, 3 of which match our first 3 and 9-3=6 don’t match any of our first 3.

Once we have that equation, we see that

P[At least 1 pair] = 1− 16

33=

17

33

On a side note, it turns out to be somewhat hard to find the probability of exactly 1 matchingpair, but it is easy to find the chance of exactly 2 matching pairs:

There are

(12

4

)ways to choose 4 shoes from the closet. Name the 6 pairs A, B, C, D, E, F .

For any specific two pairs, such as A and B, there are 4 shoes between the two pairs and

(4

4

)= 1

c©2012 The Infinite Actuary, LLC p. 6 A.4 Solutions

Page 29: The In nite Actuary Exam 1/P Online Seminar A.1 ... · The In nite Actuary Exam 1/P Online Seminar A.1 Fundamentals of Probability Solutions Last updated February 23, 2013 1. Let

ways to choose those 4 shoes. There are

(6

2

)ways to choose which two pairs we select, so

P[Choose exactly 2 pairs] =

(6

2

)·(

4

4

)(

12

4

) =1

33

We could now find P[exactly 1 pair] = P[at least 1 pair]− P[2 pairs] =17

33− 1

33=

16

33

30. Method 1: First, let’s find the probability that we have exactly 2 suits, and those suits arehearts (H) and spades (S). There are 4 ways for that to happen: we could have 4 spades and 1heart, 3 spades and 2 hearts, 2 spades and 3 hearts, or 1 spade and 4 hearts. So

P[at least 1 S and at least 1H] =

(13

4

)·(

13

1

)+

(13

3

)·(

13

2

)+

(13

2

)·(

13

3

)+

(13

1

)·(

13

4

)(

52

5

)=

63,206

2,598,960= 0.02432

But that isn’t what we want, as we don’t know which 2 suits we want. Instead, there are

(4

2

)= 6

ways to choose the two suits, giving

P[Exactly 2 suits] = 6 · 0.02432 = 0.146

Method 2: Again, let’s first find the probability that the 2 suits are hearts and spades, and

then multiply by 6. There are

(26

5

)ways for all cards to be hearts and/or spades, but that also

includes the possibility that all 5 are hearts or all 5 are spades. So to have both hearts and spades,we get

P[at least 1 S and at least 1H] =

(26

5

)−(

13

5

)−(

13

5

)(

52

5

)=

63,206

2,598,960

P[Exactly 2 suits] = 6 · 63,206

2,598,960= 0.146

Method 3: Finally, we could try to draw a tree diagram, although this will be pretty compli-cated. After we look at the first card, we have exactly 1 suit in our hand. After looking at thesecond card, we might have 2 cards from the same suit, or we might have 2 different suits. Then

c©2012 The Infinite Actuary, LLC p. 7 A.4 Solutions

Page 30: The In nite Actuary Exam 1/P Online Seminar A.1 ... · The In nite Actuary Exam 1/P Online Seminar A.1 Fundamentals of Probability Solutions Last updated February 23, 2013 1. Let

we need to look at the 3rd card, 4th card, etc., giving us:

22

48

23

49

24

50

4th cardfrom 2suits

39

51

3rd cardfrom 1 of2 suits

2 suits

22

48

23

49

39

50

get2nd

suit

2nd card from same suit

22

48

39

49

2nd suit

still same suit

12

51

39

48

10

49

still same suit

11

50

1 card, 1 suit

Chasing through all these cases gives us

P[Exactly 2 suits] =39

51· 24

50· 23

49· 22

48+

12

51· 39

50· 23

49· 22

48+

12

51· 11

50· 39

49· 22

48+

12

51· 11

50· 10

49· 39

49

31. P[Fake coin | 3 heads] =P[Fake coin and 3 heads]

P[3 heads]

=P[Fake coin and 3 heads]

P[Fake coin and 3 heads] + P[Real coin and 3 heads]

=(1/10) · 1

(1/10) · 1 + (9/10) · (1/2)3=

8

17

32. Using the complement gives

1− P[no 6 | all different] = 1− P[no 6 and all are different]

P[all are different]

= 1−

5

6· 4

6· 3

66

6· 5

6· 4

6

=1

2

c©2012 The Infinite Actuary, LLC p. 8 A.4 Solutions

Page 31: The In nite Actuary Exam 1/P Online Seminar A.1 ... · The In nite Actuary Exam 1/P Online Seminar A.1 Fundamentals of Probability Solutions Last updated February 23, 2013 1. Let

The Infinite Actuary Exam 1/P Online SeminarA.5 Key Discrete Distributions SolutionsLast updated March 19, 2012

1. Since x is the number of businesses examined prior to finding a failing business, x is a geometricstarting at 0. p = P[x = 0] = 0.10, so the answer is A

2. Let N be the number of claims. Taking complements, P[N > 1] = 1 − p0 − p1. We don’tknow p0 and p1, but we do have a recursive connection between the pn that we can use to writeeverything in terms of p0. We can then solve for p0 using the fact that the probabilities sum to 1:

p1 =p05, p2 =

p15

=p052

p3 =p25

=p052· 1

5=p053

pn =p05n

1 =∞∑n=0

pn =(p0 +

p05

+p052

+ · · ·)

1 =p0

1− 15

=5

4p0, p0 =

4

5

and from our recursion relation, p1 = (4/5) · (1/5) = 4/25 so

P[N > 1] = 1− p0 − p1 = 1− 4

5− 4

5· 1

5= 0.04

Alternatively, the distribution is a geometric: As before, pn =p05n

= p0

(1

5

)nBut any time P[X = n] = c (blah)n then X is a geometric random variable. Here, P[X = 0] =

p0 > 0, so we have a geometric starting at 0, and if X is a geometric starting at 0 then

P[X = n] = (1− p)n p =

(1

5

)np0

so 1− p =1

5, p =

4

5p = p0

p1 = (1− p) p =1

5· 4

5=

4

25

1− p0 − p1 = 1− 4

5− 4

25= 0.04

3. Let B denote the benefit amount. The chances of seeing the possible values of B are:

P[B = 4,000] = 0.4

P[B = 3,000] = (0.6)(0.4) = 0.24

P[B = 2,000] = P[fail in 3rd year] = P[Survive first two years, fail in next]

= (0.6)2(0.4) = 0.144

c©2012 The Infinite Actuary, LLC p. 1 A.5 Solutions

Page 32: The In nite Actuary Exam 1/P Online Seminar A.1 ... · The In nite Actuary Exam 1/P Online Seminar A.1 Fundamentals of Probability Solutions Last updated February 23, 2013 1. Let

P[B = 1,000] = (0.6)3(0.4) = .0864

E[B] = (4,000)(0.4) + (3,000)(0.24) + (2,000)(0.144) + (1,000)(0.0864)

= 2,694

4. There are a few different ways to think about this. One is that there are 25 = 32 equally likelyoutcome from flipping a coin 5 times, and so we want #{Ways to get 3 consecutive tails}/32.

We have 3 consecutive tails if we get:

TTTHH TTTHT TTTTH TTTTT

HTTTH HTTTT

HHTTT THTTT

so we have 8 ways that work, for an answer of 8/32 = 1/4

In practice, I don’t want to list all the possible ways and would like to find a shortcut, especiallyif the numbers are larger! There are 3 different types of cases that give us 3 consecutive tails:

1. We could have the first 3 all be tails (probability (1/2)3 = 1/8). That is also the first row ofour complete list, which had 4 cases with probability 1/32 each, for a total of 4/32 = 1/8.

2. The first flip could be heads, followed by 3 tails. That has probability (1/2)4 = 1/16 becausewe are specifying the results of the first 4 flips. That is also the second row of cases, whichhad 2 cases (giving us the same 2/32 = 1/16).

3. The last 3 flips could be tails. To avoid overlap with our previous cases, we need the 2nd flipto be heads. That specifies the last 4 flips, so that has probability (1/2)4 = 1/16, and is also,as you may have guessed, the third row of outcomes (so 2/32 = 1/16).

Summing those 3 cases gives us1

8+

1

16+

1

16=

1

4

5. Let X be the number of rolls until we get a 6. So X is a geometric on {1, 2, . . . } and

P[X = 5 | X ≥ 2] =P[X = 5, X ≥ 2]

P[X ≥ 2]=

P[X = 5]

P[X ≥ 2]

=

(5

6

)4

· 1

6

1− P[X = 0]− P[X = 1]

=

(5

6

)4

· 1

6

1− 0− 1

6

=

(5

6

)3

· 1

6= 0.096

An alternative approach is that in order to have X = 5 given X ≥ 2, we need the 2nd, 3rd, and4th rolls to not be 6, but the 5th roll to be a 6, which gives

P[X = 5 | X ≥ 2] =

(5

6

)3

· 1

6= 0.096

c©2012 The Infinite Actuary, LLC p. 2 A.5 Solutions

Page 33: The In nite Actuary Exam 1/P Online Seminar A.1 ... · The In nite Actuary Exam 1/P Online Seminar A.1 Fundamentals of Probability Solutions Last updated February 23, 2013 1. Let

6. The number of rolls until the first 3 is a geometric on {1, 2, ...} with mean 1/p = 6 and variance(1− p)/p2 = 30.

The number of rolls after the first 3 until the second is likewise a geometric with the sameparameters, as is the number of rolls after the second 3 and until the third, etc.

So N is the sum of 5 independent geometrics, and hence E[N ] = 5 · 6 = 30 and Var[N ] =5 · 30 = 150, so

CV(N) =

√150

30= 0.408

Alternatively, the number of non-3s before the 5th 3 is a negative binomial with r = 5 andp = 1/6, and thus has mean r(1 − p)/p = 25 and variance r(1 − p)/p2 = 150. But that variableis N − 5, so E[N ] = 25 + 5 = 30 and Var[N ] = 150 (the variances are the same since adding orsubtracting a constant doesn’t change the variance) which again gives us 0.408 for the Coefficientof Variation.

7. A lot of people have trouble understanding what the SOA meant by their phrasing, so let’sconsider an example. Suppose that January is the first month of the year, and that there areaccidents in January, March, April, and June, but not in February or May. Then June is the 4thmonth with an accident, and we have had no accidents in two months, so that is not a case that wewant to include in computing our probability. On the other hand, if we have accidents in January,April, June, and September, but none in February, March, May, July, or August, then we havehad 5 months with no accidents prior to our 4th month with an accident (and in particular havehad at least 4 months with no accidents), so that is in the event that we care about. Note that weare not requiring the months with accidents to be consecutive.

Let N denote the number of months with no accident before the fourth month with an accident.We want to find P[N ≥ 4] = 1 − P[N < 4]. In order to have N = k, we need month k + 4 to bethe fourth month with an accident, meaning that the first k + 3 months have k months withoutaccidents and 3 months with an accident, giving us

P[N = 0] =

(3

5

)4

P[N = 1] = (5− 1)

(3

5

)4 (2

5

)P[N = 2] =

(6− 1

2

) (3

5

)4 (2

5

)2

P[N = 3] =

(7− 1

3

) (3

5

)4 (2

5

)3

P[N ≥ 4] = 1− P[N = 0]− P[N = 1]− P[N = 2]− P[N = 3]

= 0.2898

8. Let N denote the number of days of rain, so N ∼ Poisson (0.6). Let X be the amount of thepayment. We want to find SD(X), so first let’s make a table listing the possible payment amounts

c©2012 The Infinite Actuary, LLC p. 3 A.5 Solutions

Page 34: The In nite Actuary Exam 1/P Online Seminar A.1 ... · The In nite Actuary Exam 1/P Online Seminar A.1 Fundamentals of Probability Solutions Last updated February 23, 2013 1. Let

and probabilities.x n P[X = x]0 0 P[N = 0] = e−λ = e−0.6

1,000 1 P[N = 1] = λ e−λ = 0.6e−0.6

2,000 at least 2 P[N ≥ 2] = 1− P[N < 2]= 1− e−0.6 − 0.6 e−0.6

E(X) = 0 · e−.6 + 1,000(.6 e−.6

)+ 2,000

(1− e−.6 − .6 e−.6

)= 573

E(X2) = 02 · e−.6 + 1,0002(.6 e−.6

)+ 2,0002

(1− 1.6 e−.6

)= 816,893

Var(X) = 816,893− 5732 = 488,461

SD(X) =√

488,461 ≈ 699

9. Let X be the number of tests completed, so X is a geometric starting at 1, and E[X] = 1/p,where p is the probability of “success” (which in this case means that the person has high bloodpressure).

1

p= 12.5

p =1

12.5P[X = 6] = (1− p)5 p

=

(11.5

12.5

)5

· 1

12.5= 0.053

10. The point of this problem is the fact that the person made a claim in 1997 means he is morelikely than a random selected policyholder to be high risk, and we have to take that into account.

P[θ = 0.6 | 1 claim in 1997] =P[θ = 0.6, 1 claim in 1997]

P[1 claim in 1997]

=(0.1) (0.6) e−0.6

(0.1) (0.6) e−0.6 + (0.9) (0.1) e−0.1

= 0.288

P[θ = 0.1 | 1 claim in 1997] = 1− 0.288

E[# claims in 1998 | 1 claim in 1997] = P[θ = 0.6 | 1 claim in 1997] · 0.6+ P[θ = 0.1 | 1 claim in 1997] · 0.1= (0.288) (0.6) + (0.712) (0.1)

= 0.244

11. In 12 hours, the mean number of power surges is 1. In 24 hours, the mean number is therefore2. The total number in a 24 hours period is still Poisson (because the sum of independent Poissonsis Poisson), and

P[N ≤ 1] = P[N = 0] + P[N = 1]

c©2012 The Infinite Actuary, LLC p. 4 A.5 Solutions

Page 35: The In nite Actuary Exam 1/P Online Seminar A.1 ... · The In nite Actuary Exam 1/P Online Seminar A.1 Fundamentals of Probability Solutions Last updated February 23, 2013 1. Let

= e−λ + λe−λ = 3e−2

12. Given that Y = 2, we know that the first roll is 1, 2, 3, 4, or 5. So there are 5 possible

outcomes for roll 1, one of which is a 5. Therefore P[X = 1 | Y = 2] = 1/5

Alternatively, P[X = 1, Y = 2] =1

6· 1

6and P[Y = 2] =

5

6· 1

6, and dividing gives us P[X = 1 |

Y = 2] = 1/5.

13. If Y = 2 then the 2nd roll is a 6. If X = 2 then the 2nd roll is a 5. We can’t have both atonce, so the answer is 0

14. We are given that the first rolls is not 6, leaving 5 possibilities. We also want the first roll tonot be a 5 (probability 4/5). We are given that the second roll is a 6 (and in particular not a 5).We aren’t given what happens on the third roll, but need it to be a 5 (probability 1/6), giving us

an answer of4

5· 1

6=

2

15

Alternatively, P[X = 3, Y = 2] =4

6· 1

6· 1

6and P[Y = 2] =

5

6· 1

6, and dividing gives us

P[X = 1 | Y = 3] = 4/30 = 2/15.

15. Given that Y = 2, the first roll is not 6, the 2nd is a 6, and the rolls after that can be anything.X = k requires that the first not be a 5 (probability 4/5 given that it isn’t a six), the second notbe a 5 (probability 1 since we know it is a 6), rolls 2 through k − 1 not be a 5 (probability 5/6

each) and the kth be a 5, so we get4

5· 1 ·

(5

6

)(k−1)−2

· 1

6= E

16. The sum of independent Poisson variables is Poisson, so X + Y ∼ Poisson(3 + 2 = 5) and

P[X = 2 | X + Y = 4] =P[X = 2, X + Y = 4]

P[X + Y = 4]

=P[X = 2, Y = 2]

P[X + Y = 4]=

P[X = 2] · P[Y = 2]

P[X + Y ] = 4

=e−3

32

2!· e−222

2!

e−554

4!

=4!

2! 2!· 32 · 22

54=

(4

2

) (3

5

)2 (2

5

)2

= 0.3456

17. Again, X + Y ∼ Poisson(5), so

P[X = k | X + Y = n] =P[X = k,X + Y = n]

P[X + Y = n]

=P[X = k, Y = n− k]

P[X + Y = n]=

P[X = k] · P[Y = n− k]

P[X + Y ] = n

c©2012 The Infinite Actuary, LLC p. 5 A.5 Solutions

Page 36: The In nite Actuary Exam 1/P Online Seminar A.1 ... · The In nite Actuary Exam 1/P Online Seminar A.1 Fundamentals of Probability Solutions Last updated February 23, 2013 1. Let

=

e−33k

k!· e−2 2n−k

(n− k)!

e−55n

n!

=n!

k!(n− k)!· 3k · 2n−k

5n

=

(n

k

) (3

5

)k (2

5

)n−kand the answer is B

More generally (this is not important for the exam) if X and Y are independent Poissonvariables with means λ and µ then given that X + Y = n, the conditional distribution of X is abinomial with parameters n = X + Y and p = λ/(λ+ µ).

18. There are 8 possible cases for us to consider: 7 claims in week 1 and 0 in week 2, 6 in week 1and 1 in week 2, 5 in week 1 and 2 in week 2, etc. For each case, if there are x claims in week 1and 7− x in week 2, the probability is

2−(x+1) · 2−(7−x+1) = 2−9

and so our final answer is 8 · 2−9 = 2−6 = 1/64

19. Let X be the number of chips in the first cookie and Y the number of chips in the second.We could either list out 4 cases and sum their probabilities (X = 0, Y = 3 or X = 1, Y = 2 orX = 2, Y = 1 or X = 3, Y = 0). A faster approach is to use the fact that the sum of independentPoissons is Poisson, so X + Y ∼ Poisson(4), and so

P[X + Y = 3] = e−4 · 43

3!=

32

3e−4

20. Recall that ex =∑∞

n=0xn

n!. Then

P[X = N = n] = P[X = n] · P[N = n] by independence

= P[n tails, then 1 head] · P[N = n]

=

(1

2

)n+1

· e−1 · 1n

n!=e−1

2·(1/2)n

n!

P[X = N ] =∞∑n=0

P[X = N = n]

=∞∑n=0

e−1

2·(1/2)n

n!=e−1

2

∞∑n=0

(1/2)n

n!

=e−1

2· e1/2 =

e−1/2

2= 0.30

See video solution for how to approximate this using the calculator.

21. Since the sum of independent Poisson variables is Poisson, the total number of accidents over

c©2012 The Infinite Actuary, LLC p. 6 A.5 Solutions

Page 37: The In nite Actuary Exam 1/P Online Seminar A.1 ... · The In nite Actuary Exam 1/P Online Seminar A.1 Fundamentals of Probability Solutions Last updated February 23, 2013 1. Let

13 weeks is a Poisson with mean 13 · 25 = 325, and thus coefficient of variation

√325

325= 0.055

22. We need to have 0 accidents in the first 3 days, and then at least one accident on day4. P[No accidents in first 3 days] = e−3·(1/2) = e−1.5, and P[At least one accident on day 4] =1− e−1/2, for a final answer of

e−1.5[1− e−0.5

]= 0.088

23. Let Ni be the number of storms in month i. Then Ni is a binomial with n = 4 and 1 =E[Ni] = np = 4p so p = 1/4. We are given that there are no tropical storms in the first month, sowe want to find

P[N2 = N3 = 0, N4 > 0] = P[N2 = 0] · P[N3 = 0] · [1− P[N4 = 0]]

=

(3

4

)4·2[

1−(

3

4

)4]

= 0.068

24. The number of tails, T , that comes up before the first heads is a Geometric(p) starting at 0.

E[T ] = 3 =1− pp

p =1

4

Var[T ] =1− pp2

= 12

SD[T ] =√

12

25. Since the standard deviation is 4, the variance is 16. For a Poisson, the mean equals thevariance, giving us an answer of 16

26. Because we are sampling with replacement, each draw is independent. That means that thenumber of white balls drawn before the fourth red ball is a negative binomial with r = 4 andp = 0.4, so

P[X = 6] =

(6 + (4− 1)

4− 1

)· 0.66 · 0.44 = 0.1003

c©2012 The Infinite Actuary, LLC p. 7 A.5 Solutions

Page 38: The In nite Actuary Exam 1/P Online Seminar A.1 ... · The In nite Actuary Exam 1/P Online Seminar A.1 Fundamentals of Probability Solutions Last updated February 23, 2013 1. Let

The Infinite Actuary Exam 1/P Online SeminarA.6 Discrete Deductibles SolutionsLast updated June 21, 2013

1. We want to find the expected payment, but first we need to find K and thus distribution ofloss. The sum of the probabilities of all possible events is 1, so

1 =K

1+

K

2+

K

3+

K

4+

K

5= 2.2833K

K ≈ 0.43796

So taking into account the deductible, the expected payment is

E[payment] = 0.05

(0 · K

1+ 0 · K

2+ (3− 2) · K

3+ (4− 2)

K

4+ (5− 2)

K

5

)= 0.031

2. The tour operator receives 21 · 50 if 20 or fewer tourists show up, but only 21 · 50 − 100 if all21 show up. So the expected revenue is

21 · 50− 100 · P[21 show up] = 1,050− 100 · 0.9821 = 985

3. Let d be the deductible amount. Assuming d > 3 then the payment is 0 if X = 3 and is 12− dif X = 12, so the expected payment is

E[Payment] =0 · P[X = 3] + (12− d) · P[X = 12]

3 =0 + (12− d) · 0.5d = 6

4. Let N be the number of losses. Then a table of possible payment values is

n Uncovered Loss Payment P[N = n]0 0 0 1/51 40 0 (4/5) · (1/5)2 80 0 (4/5)2 · (1/5)3 100 20 (4/5)3 · (1/5)4 100 60 (4/5)4 · (1/5)... 100

......

The uncovered loss column only has 4 possible values, while the payment amount is more compli-cated. So

E[Payment] =E[Total Loss]− E[Uncovered Loss]

=40 · 4−

[0 · 1

5+ 40 · 4

5· 1

5+ 80 ·

(4

5

)2

· 1

5+ 100 ·

(4

5

)3]

c©2013 The Infinite Actuary, LLC p. 1 A.6 Solutions

Page 39: The In nite Actuary Exam 1/P Online Seminar A.1 ... · The In nite Actuary Exam 1/P Online Seminar A.1 Fundamentals of Probability Solutions Last updated February 23, 2013 1. Let

=160− 67.84 = 92.16

Note that P[N ≥ 3] =

(4

5

)3

since N is a geometric starting at 0.

5. Let N denote the number of snowstorms. The insurance company pays

Payment =

{10,000(N − 1) N ≥ 1

0 N = 0

We can view that as saying that the payment is 10,000 per storm, with a deductible of 10,000.Then using our methods for finding the expected payment with a deductible,

E[Total loss] = E[Payment] + E[Uncovered Loss]

10,000E[N ] = E[Payment] + 10,000 · P[N ≥ 1]

10,000 · 1.5 = E[Payment] + 10,000(1− P[N = 0])

E[Payment] = 5,000 + 10,000 · e−1.5 = 7,231

For a more algebraic approach:

E[Payment] =∞∑n=1

10,000(n− 1)P[N = n]

=

[∞∑n=0

10,000(n− 1)P[N = n]

]− 10,000(0− 1)P[N = 0]

where the last line comes from adding and subtracting the n = 0 term

= 10,000 · E[N − 1]− 10,000 · (−1)e−1.5

= 10,000(1.5− 1) + 10,000e−1.5 = 7,231

6. There are 20 bulbs in the shipment, 6 of which are defective. We are testing 5 bulbs, and the

total expected damage in those 5 test bulbs is 5 · 6

20· 10 = 15. The uncovered loss is 10 if any of

the tested bulbs are defective. Let N be the number of tested bulbs that are defective, so

E[Uncovered Loss] =10 · P[N > 0]

=10 · (1− P[N = 0])

=10 ·

1−

(6

0

)·(

14

5

)(

20

5

)

=8.71

E[Payment] =10 · E[N ]− 8.71

=15− 8.71 = 6.29

c©2013 The Infinite Actuary, LLC p. 2 A.6 Solutions

Page 40: The In nite Actuary Exam 1/P Online Seminar A.1 ... · The In nite Actuary Exam 1/P Online Seminar A.1 Fundamentals of Probability Solutions Last updated February 23, 2013 1. Let

7. Let N be the number of months with storms.

E[Total Losses] =E[10,000N ] = 10,000 · 5 · 0.3 = 15,000

E[Uncovered Loss] =5,000 · P[N ≥ 1] = 5,000(1− 0.75

)= 4,160

E[Payment] =15,000− 4,160 = 10,840

8. The loss amount will exceed the policy limit only if there are either 4 months with storms (foran excess of 5,000) or if there are 5 months with storms (and an excess of 15,000). So

E[Total Losses] =15,000

E[Uncovered Loss] =5,000 ·(

5

4

)0.34 · 0.7 + 15,000 · 0.35 = 178

E[Payment] =15,000− 178 = 14,822

9. The number of months with storms ranges from 0 to 5, with the following resulting payments:

n Uncovered Loss Payment P[N = n]0 0 0 0.75

1 5,000 5,000 5 · 0.74 · 0.32 5,000 15,000 10 · 0.73 · 0.32

3 5,000 25,000 10 · 0.72 · 0.33

4 10,000 30,000 5 · 0.7 · 0.34

5 20,000 30,000 0.35

And we see that the answer is 10,662 Alternatively, the deductible of 5,000 saves 4,160 (from

# 7). The max payment is 30,000, which saved 178 in # 8, so the total expected payment is15,000− 4,160− 178 = 10,662.

10. Let N be the loss amount. Then

n Uncovered Loss Payment P[N = n]0 0 0 e−4

1 1 0 4e−4

2 2 0 (42/2!)e−4

3 3 0 (43/3!)e−4

4 3 1 (44/4!)e−4

5 3 2 (45/5!)e−4

6 4 2 (46/6!)e−4

E[Payment] =P[N = 4] + 2 · P[N ≥ 5]

=P[N = 4] + 2 · (1− P[N ≤ 4])

=44

4!e−4 + 2 ·

(1− e−4

[1 + 4 +

42

2!+

43

3!+

44

4!

])= 0.935

c©2013 The Infinite Actuary, LLC p. 3 A.6 Solutions

Page 41: The In nite Actuary Exam 1/P Online Seminar A.1 ... · The In nite Actuary Exam 1/P Online Seminar A.1 Fundamentals of Probability Solutions Last updated February 23, 2013 1. Let

11. Let X be the loss amount, so X is uniform on {0, 1, 2, 3, 4, 5, 6}.

x Uncovered Loss Payment P[X = x]0 0 0 1/71 1 0 1/72 1 1 1/73 1 2 1/74 1 3 1/75 2 3 1/76 3 3 1/7

E[Uncovered Loss] = 1 · 4

7+ 2 · 1

7+ 3 · 1

7=

9

7

12. One is to make a table with the possible values of X, their probabilities, and the resultingpayment.

k P[X = k] Payment1 5/15 502 4/15 1503 3/15 2504 2/15 3505 1/15 450

So E[Payment] = 50 · 5

15+ 150 · 4

15+ · · · = 183.3

Alternatively, E[100X] = 233.3, and the deductible always reduces the payment by 50 (i.e., the

uncovered loss is always 50), giving us 233.3− 50 = 183.3

13. Again, making a table gives

k P[X = k] Payment1 5/15 1002 4/15 2003 3/15 3004 2/15 3005 1/15 300

for a final answer of 100 · 5

15+ 200 · 4

15+ 300 · 6

15= 206.7

14. Note that our table is slightly adjusted:

k P[X = k] Payment1 5/15 502 4/15 1503 3/15 2504 2/15 3005 1/15 300

c©2013 The Infinite Actuary, LLC p. 4 A.6 Solutions

Page 42: The In nite Actuary Exam 1/P Online Seminar A.1 ... · The In nite Actuary Exam 1/P Online Seminar A.1 Fundamentals of Probability Solutions Last updated February 23, 2013 1. Let

50 · 5

15+ 150 · 4

15+ 250 · 3

15+ 300 · 3

15= 166.7

15. Let N be the number of snowstorms. If N = 0 then there is no loss and therefore no payment.If N = 1 then the loss is 10,000 and since that is less than the deductible, there is still no payment.If N = 2 then the loss is 20,000 and the payment is 5,000. So

P[Payment = 0] = P[N = 0] + P[N = 1] = e−1.5 + 1.5e−1.5 = 0.558

P[Payment = 5,000] = P[N = 2] =1.52

2e−1.5 = 0.251

P[Payment ≤ 5,000] = 0.809

So 5,000 is the first time that the cdf exceeds 0.75 and is therefore the 75th percentile.

Note that the 75th percentile of N is 2, which leads to a loss of 20,000 and a payment of 5,000.That is another way to find percentiles of payments.

c©2013 The Infinite Actuary, LLC p. 5 A.6 Solutions

Page 43: The In nite Actuary Exam 1/P Online Seminar A.1 ... · The In nite Actuary Exam 1/P Online Seminar A.1 Fundamentals of Probability Solutions Last updated February 23, 2013 1. Let

The Infinite Actuary Exam 1/P Online SeminarA.7 Discrete Reveiw SolutionsLast updated April 6, 2013

1. When we have only two sets, I often prefer drawing a 2x2 grid to a traditional Venn diagram.That is especially true when doing conditional problems, or when dealing with complements (thelatter of which applies here). But you can do it either way.

A 0.65

A′ 0.35

B B′

0.15 1− 0.5

(A′ ∪ B)′ = (A ∩ B′) so P[A ∩ B′] = 1 − 0.5 = 0.5. We then have P[A] = P[AB] + P[AB′] =

0.15 + 0.5 = 0.65 and P[A′] = 1− 0.65 = 0.35

2. P[A ∪B] = P[A] + P[B]− P[AB]

0.8 = 0.6 + 0.5− P[AB]

P[AB] = 0.3

3. P[A ∩ B is a probability, so 0 ≤ P[A ∩ B] ≤ 1. Also, P[A ∩ B] ≤ min{P[A],P[B]} = 0.3. If wesolve P[A ∪B] = P[A] + P[B]− P[AB] for P[AB] we get P[AB] = P[A] + P[B]− P[A ∪B] so

0 ≤ P[A ∩B] ≤ 0.3

0 ≤ P[A] + P[B]− P[A ∪B] ≤ 0.3

0 ≤ 0.7− P[A ∪B] ≤ 0.3

0.4 ≤ P[A ∪B] ≤ 0.7

From this, we get a minimum possible value of 0.4 which is realized if A ⊂ B.

4. From the previous part, we had a maximum possible value of 0.7 which is realized if A and Bare mutually exclusive.

5. Let L denote people who need lab work, and S those who need a specialist. Then

P[(L ∪ S)′] = 0.35

P[L ∪ S] = 0.65

P[L ∪ S] = P[L] + P[S]− P[LS]

0.65 = 0.40 + 0.30− P[LS]

P[LS] = 0.05

P[LS ′] = P[L]− P[LS] = 0.4− 0.05 = 0.35

c©2013 The Infinite Actuary, LLC p. 1 A.7 Solutions

Page 44: The In nite Actuary Exam 1/P Online Seminar A.1 ... · The In nite Actuary Exam 1/P Online Seminar A.1 Fundamentals of Probability Solutions Last updated February 23, 2013 1. Let

6.

A B

C

0

0

0 0x

13− x1

4− x

5

12=

(1

4− x)

+

(1

3− x)

=7

12− 2x, x =

1

12

P[(A ∪B ∪ C)′] = 1− x−(

1

4− x)−(

1

3− x)

= 1− 1

12− 3− 1

12− 4− 1

12=

1

2

7.

A B

C0.1

0.1 0.10.12

0.120.12 x

P[all 3 | AB] =P[all 3 and AB]

P[AB]1

3=

x

0.12 + xx = 0.06

P[none | A′] =P[none]

P[A′]

=1− 3 · 0.1− 3 · 0.12− 0.06

1− 0.10− 2 · 0.12− 0.06

=0.28

0.60= 7/15

8. Let A, H, and R denote those who have auto, homeowners, and renters insurance, respectively.Since P[HR] = 0, we know that those two sets don’t overlap. To fill in the Venn diagram, Iuse properties (i) then (v) then (ii) then (iv) to obtain the numbers that are filled in. Then letx = P[R∩A′] and use the fact that the probabilities sum to 1 to see that P[A∩R′∩H ′] = 0.53−x.That gives

c©2013 The Infinite Actuary, LLC p. 2 A.7 Solutions

Page 45: The In nite Actuary Exam 1/P Online Seminar A.1 ... · The In nite Actuary Exam 1/P Online Seminar A.1 Fundamentals of Probability Solutions Last updated February 23, 2013 1. Let

H R

A0.160.53− x

0.11 x

0.080.12

We haven’t yet used (iii), which gives

0.12 + 0.08 + 0.53− x = 2(0.08 + x)

x = 0.19

9. Let M denote people who insure more than 1 car, and S those who insure a sports car.

P[M ′ ∩ S ′] = 1− P[(M ′ ∩ S ′)′]= 1− P[M ∪ S]

= 1− (P[M ] + P[S]− P[MS])

= 1− 0.64− 0.20 + 0.12

= 0.28

M

M ′

S S ′

0.12 0.52 0.64

0.08 0.28

10. Let M denote people who insure more than 1 car, and S those who insure a sports car. Thedifference between this problem and the previous one is that here we are given P[S | M ] instead

of P[MS] in condition (iv). P[MS] = P[M ] · P[S |M ] = 0.64 · 0.15 = 0.096 so P[M ′S ′] = 0.256

M

M ′

S S ′

0.096 0.544 0.64

0.104 0.256

11. Let g(k) be the payment for k days in the hospital.

E[g(X)] =∑

g(k) · P[X = k]

= 100 · P[X = 1] + 200 · P[X = 2] + . . .

= 100 · 5

15+ 200 · 4

15+ 300 · 3

15+ 350 · 2

15+ 400 · 1

15

= 220

c©2013 The Infinite Actuary, LLC p. 3 A.7 Solutions

Page 46: The In nite Actuary Exam 1/P Online Seminar A.1 ... · The In nite Actuary Exam 1/P Online Seminar A.1 Fundamentals of Probability Solutions Last updated February 23, 2013 1. Let

12. Let N be the number of claims that are filed. Then

P[N = 2] = 3 · P[N = 4]

λ2

2!e−λ = 3 · λ

4

4!e−λ

1 =λ2

4

λ = 2

13. Let p4 − p5 = c. From (ii), we know that

p3 − p5 = (p3 − p4) + (p4 − p5) = 2c

p2 − p5 = (p2 − p3) + (p3 − p5) = c+ 2c = 3c

p1 − p5 = 4c

p0 − p5 = 5c

so we can rewrite our pi terms as

p0 = 5c+ p5, p1 = 4c+ p5, p2 = 3c+ p5 etc.

We now have two unknowns: c and p5. But

1 = p0 + p1 + p2 + p3 + p4 + p5

1 = (5 + 4 + 3 + 2 + 1)c+ 6p5 = 15c+ 6p5

and from (iii),

0.4 = p0 + p1 = (5 + 4)c+ 2p5 = 9c+ 2p5

1.2 = 27c+ 6p5

0.2 = 12c, c =1

60

6p = 1− 15

60=

3

24

p4 + p5 = c+ 2p5 =1

60+

3

12

p4 + p5 =16

60= 0.27

14. Since Y = 2, then X 6= 2 so we have 2 cases: either X = 1 or X > 2. Then

E[X | Y = 2] = E[X | Y = 2, X = 1] · P[X = 1 | Y = 2] + E[X | Y = 2, X > 2] · P[X > 2 | Y = 2]

= 1 · P[X = 1 | Y = 2] + E[X | X > 2] · P[X > 2 | Y = 2]

= 1 · 1

5+ (2 + 6) · 4

5= 6.6

15. The number of hurricanes N that don’t damage our home before we have damage from 2

c©2013 The Infinite Actuary, LLC p. 4 A.7 Solutions

Page 47: The In nite Actuary Exam 1/P Online Seminar A.1 ... · The In nite Actuary Exam 1/P Online Seminar A.1 Fundamentals of Probability Solutions Last updated February 23, 2013 1. Let

hurricanes is a negative binomial with r = 2 and p = 0.4, so P[N = n] =

(n+ (2− 1)

2− 1

)· 0.6n · 0.42

Using the MultiView, we see that this is maximized when n = 1. So the mode of N is 1, but thevariable that we care about is N+2 (since we want to add on the 2 hurricanes that cause damage),

and so that has mode 1 + 2 = 3

16. E[total bonus] = 400E[bonus for 1 low risk] + 600E[bonus for 1 high risk]

= 400 · 5 · 0.9 · 12 + 600 · 5 · 0.8 · 12 = 50,400

17. Let H, L, and N denote heavy, light, and non-smokers respectively, and D the event that aparticipant died. Then

P[N | D] =P[ND]

P[D]

=P[ND]

P[HD] + P[LD] + P[ND]

=0.6P[D | N ]

0.15P[D | H] + 0.25P[D | L] + 0.6P[D | N ]

=0.6P[D | N ]

0.15 · 4 · P[D | N ] + 0.25 · 2 · P[D | N ] + 0.6P[D | N ]

= 0.3529

18. P[Serious | Survived] =P[Serious, Survived]

P[Survived]=

0.25 · 0.90.05 · 0.6 + 0.25 · 0.9 + 0.7 · 0.99

= 0.237

19. Let H denote someone who is high risk, and let N1, N2 be the number of claims in years 1 and2 respectively. Then

P[H | N1 = N2 = 4] =P[H,N1 = N2 = 4]

P[N1 = N2 = 4]

=0.5 · e−4 · 44

4!· e−4 · 44

4!

0.5 · e−4 · 44

4!· e−4 · 44

4!0.5 · e−2 · 24

4!· e−2 · 24

4!

= 0.824

20. Let A,B and C denote the 3 types of dice, and let X1 be the outcome of the first die roll.Then

P[A | X1 = 3] =P[A,X1 = 3]

P[X1 = 3]=

3

6· 2

63

6· 2

6+

2

6· 2

6+

1

6· 0

=3

5

P[B | X1 = 3] =P[A,X1 = 3]

P[X1 = 3]=

2

6· 2

63

6· 2

6+

2

6· 2

6+

1

6· 0

=2

5

c©2013 The Infinite Actuary, LLC p. 5 A.7 Solutions

Page 48: The In nite Actuary Exam 1/P Online Seminar A.1 ... · The In nite Actuary Exam 1/P Online Seminar A.1 Fundamentals of Probability Solutions Last updated February 23, 2013 1. Let

E[X2 | X1 = 3] =3

5E[X2 | A] +

2

5E[X2 | B]

=3

5· 2 +

2

5· 3 = 2.4

21. Let A be the event that the first component fails, and B the event that the second componentfails. Then we are given that

0.28 = P[A ∪B]

0.28 = P[A] + P[B]− P[A ∩B]

0.28 = P[A] + 2P[A]− P[A] · (2P[A])

2(P[A])2 − 3P[A] + 0.28 = 0

and solving the quadratic gives P[A] = 0.1 or P[A] = 1.4. The second is impossible since 0 ≤P[A] ≤ 1, so the answer is P[A] = 0.1

22. For one specific toss, the probability of no 7 or 11 is 1 − 6

36− 3

36=

28

36. All 10 tosses are

independent, so to find the probability of that happening all 10 times, we raise that probability to

the 10th power, giving

(28

36

)10

23. First, let’s find the probability of selecting box k. We are given that P[box 1] = c, P[box 2] = 2cand P[box 3] = 3c. Since we must choose one of the boxes, we have 1 = c+ 2c+ 3c so c = 1/6.

We can then draw a tree diagram:

2/6box 2

1/6 box 1

3/6 box 3

B1

5

R4

5

B2

5

R3

5

B2

5

R3

5

4

4 1

6· 1

5· 4

4· 2

R

1

4

B

3

4

R

2 · 2

6· 2

5· 3

4

2

4

B

2

4

R

2 · 3

6· 3

5· 2

4

3

4

B

c©2013 The Infinite Actuary, LLC p. 6 A.7 Solutions

Page 49: The In nite Actuary Exam 1/P Online Seminar A.1 ... · The In nite Actuary Exam 1/P Online Seminar A.1 Fundamentals of Probability Solutions Last updated February 23, 2013 1. Let

Summing over all of our cases gives a final answer of

2 · 1

6· 1

5· 4

4+ 2 · 2

6· 2

5· 3

4+ 2 · 3

6· 3

5· 2

4=

17

30

24. P[X = 1 | X ≤ 1] =P[X = 1, X ≤ 1]

P[X ≤ 1]

0.8 =P[X = 1]

P[X ≤ 1]=

λe−λ

e−λ + λe−λ

0.8 =λ

1 + λ0.8 + 0.8λ = λ

λ = 4

25. The mode is 0 since the probability is maximized at 0. The median is likewise 0 since theprobability of 0 accidents is greater than 1/2.

The mean is 0 · 0.55 + 1 · 0.2 + 2 · 0.1 + · · · > 0, so the mean is different.That means that (i) and (ii) are both false, but (iii) is correct, so the answer is C

26. Since we are selecting with replacement, each draw is independent and we have a multinomialdistribution. That gives an answer of

6!

2!2!2!

(10

20

)2(5

20

)2(5

20

)2

= 90 ·(

1

2

)10

=45

512

27. Here we are sampling without replacement, so the draws are not independent and we wantthe hypergeometric distribution, not the binomial distribution. There are two cases that we want:namely 2 blue socks, or 2 white socks, and so we get(

6

2

)+

(4

2

)(

10

2

) =

6 · 52

+4 · 3

210 · 9

2

=7

15

28. On each roll, the chance of rolling a seven is 6/36 = 1/6. In order for it to take exactly 10rolls to observe 8 sevens, we need the 10th roll to be a seven (probability 1/6) and for 7 of the first9 rolls to be sevens, giving (

9

7

)(1

6

)7 (5

6

)2

· 1

6=

36 · 52

610=

52

68

29. Let N be the number of variables in our sample that are equal to 6. Then N is a binomialrandom variable with n = 3 and p = 0.6. The distribution of X and N is therefore as follows:

P[N = 0] = P[X = −3] = (0.4)3 = 0.064

c©2013 The Infinite Actuary, LLC p. 7 A.7 Solutions

Page 50: The In nite Actuary Exam 1/P Online Seminar A.1 ... · The In nite Actuary Exam 1/P Online Seminar A.1 Fundamentals of Probability Solutions Last updated February 23, 2013 1. Let

P[N = 1] = P[X = 0] =

(3

2

)(0.4)2 (0.6) = 0.288

P[N = 2] = P[X = 3] =

(3

1

)(0.4)1 (0.6)2 = 0.432

P[N = 3] = P[X = 6] = (0.6)3 = 0.216

so the answer is E

30. P[S] = P[S ∩ T ] + P[S ∩ T ′] =1

10+

2

10=

3

10P[S ∩ T ] = P[S] · P[T ] by independence

1

10=

3

10· P[T ]

P[T ] =1

3P[(S ∪ T )′] = 1− P[S ∪ T ]

= 1−(

P(S) + P(T )− P(S ∩ T ))

= 1−(

3

10+

1

3− 1

10

)=

7

15

31. Let A be the event that the red die is 1, 2, or 3 and let B be the event that the sum is 11.There are two ways that B can occur: either the red die is 5 and the green die is 6, or the red dieis 6 and the green die is 5; in particular, A ∩B = ∅.

P[A ∪B] = P[A] + P[B]− P[AB]

=3

6+

2

36− 0

36

=20

36=

5

9

32.

1/2 T

1/2 H

2 dice sum to 65/36

one roll equals 61/6

1

2· 5

36+

1

2· 6

36=

11

72

33. Since we are drawing with replacement, we have independent draws and hence a multinomialdistribution, so the answer is

3!

1!1!1!

5

10· 3

10· 2

10= 0.18

c©2013 The Infinite Actuary, LLC p. 8 A.7 Solutions

Page 51: The In nite Actuary Exam 1/P Online Seminar A.1 ... · The In nite Actuary Exam 1/P Online Seminar A.1 Fundamentals of Probability Solutions Last updated February 23, 2013 1. Let

34. P[at least one 3] = 1− P[none are 3]

= 1−(

P[first die 6= 3])3

= 1−(

5

6

)3

35. There are

(52

5

)total ways to deal 5 cards, so that is the denominator. The numerator is the

number of ways that work: There is 1 way to deal the king of spades,

(3

1

)ways to have one of

the remaining 3 kings,

(4

2

)ways to have 2 queens, and that leaves 52 − 4 − 4 = 44 cards from

which to choose the one non-king, non-queen card, or

(44

1

)ways. Since we want all of these to

occur at the same time, we multiply to give a final answer of(3

1

)(4

2

)(44

1

)(

52

5

)

36. Let N be the total number of rolls. Then N is a geometric random variable starting at 1 withp = 4/6, so the distribution of N is given by

P[N = 1] =4

6=

2

3

P[N = 2] =

(1

3

)· 2

3

P[N = 3] =

(1

3

)2 (2

3

)...

P[N = k] =

(1

3

)k−1 (2

3

)

P[N is even] =

(1

3

) (2

3

)+

(1

3

)3 (2

3

)+

(1

3

)5 (2

3

)+ · · ·

=

(1

3

) (2

3

) [1 +

(1

3

)2

+

(1

3

)4

+ · · ·

]

=

1

3· 2

3

1−(

1

3

)2 =

2

98

9

=1

4

c©2013 The Infinite Actuary, LLC p. 9 A.7 Solutions

Page 52: The In nite Actuary Exam 1/P Online Seminar A.1 ... · The In nite Actuary Exam 1/P Online Seminar A.1 Fundamentals of Probability Solutions Last updated February 23, 2013 1. Let

37. If we were drawing with replacement, then each draw would be independent and we wouldhave a negative binomial distribution with r = 2 and k = 2. But since we are drawing withoutreplacement, that doesn’t apply. We have 3 cases to consider:

Real, Fake, Fake, RealFake, Real, Fake, RealFake, Fake, Real, Real

Probability of Case 1:10

35· 25

34· ·24

33· 9

32

Probability of Case 2:25

35· 10

34· 24

33· 9

32

Probability of Case 3:25

35· 24

34· 10

33· 9

32

Note that all 3 cases are equally likely. This gives

Total Probability: 3 · 10 · 9 · 25 · 24

35 · 34 · 33 · 32=

675

5,236

38. P[at least one occurs] = 1− P[none of R, S, or T occurs]

= 1−(

P[not R])(

P[not S])(

P[not T ])

= 1−(

2

3

)3

=19

27

39. Since the coins are independent, we simply multiply the probability of the gold coin beingheads 3 times by the probability that the silver coin is heads 6 times, giving(

5

3

)(0.3)3 (0.7)2

(10

6

)(0.1)6 (0.9)4

40. The sum of independent Poissons is Poisson, so W ∼ Poisson(3 + 1 + 4) and

P[W ≤ 1] = e−8 + 8 e−8 = 9 e−8

41. Since tails is twice as likely as heads, P[H]+P[T ] = P[H]+2P[H] = 3P[H] = 1 so P[H] = 1/3and P[T ] = 2/3.

For the third head to be on the fifth trial, we need the fifth trial to be heads, and two of thefirst 4 to also be heads, giving an answer of(

4

2

)·(

1

3

)2

·(

2

3

)2

· 1

3= 6 · 4

35=

8

81

Note that we have a negative binomial distribution with r = 3 (because we want 3 heads) andk = 2 (because we have two tails).

c©2013 The Infinite Actuary, LLC p. 10 A.7 Solutions

Page 53: The In nite Actuary Exam 1/P Online Seminar A.1 ... · The In nite Actuary Exam 1/P Online Seminar A.1 Fundamentals of Probability Solutions Last updated February 23, 2013 1. Let

42. P[X1 < X2 < X3] = P[X1 = 1, X2 = 2, X3 ≥ 3] + P[X1 = 1, X2 = 3, X3 = 4]

+ P[X1 = 2, X3 = 3, X4 = 4]

=1

10· 2

10· 7

10+

1

10· 3

10· 4

10+

2

10· 3

10· 4

10

=14 + 12 + 24

1,000= 0.050

43. (A ∩ B)′ = A′ ∪ B′, so (A ∩ B)′ ∪ C = A′ ∪ B′ ∪ C. Since B and C are mutually exclusive,C ⊂ B′, so A′ ∪B′ ∪ C = A′ ∪B′. But A and B are independent, so

P[A′ ∪B′] = P[A′] + P[B′]− P[A′B′] =3

4+

5

6− 3

4· 5

6=

23

24

44. X has a binomial distribution with parameters n and p, so

E[X] = np

SD[X] =√npq

np =√np(1− p)

n2p2 = np(1− p)np = 1− p

n =1− pp

Since n must be an integer, we need to find an answer choice such that (1− p)/p is an integer. Ifp = 0.1 then n = 9 then that holds, so A is possible. If p = 0.15 then n = 5.67, so B is wrong.Likewise 0.7/0.3 is not an integer, nor is 0.6/0.4 or 0.4/0.6, so C, D and E don’t work either.

The answer is therefore A

45. In one week, the expected number of accidents is 2·3+3·4+2·1 = 20. The sum of independentPoissons is Poisson, so the number of accidents in a week is a Poisson(20) random variable, andthe answer is thus

e−20 · 2018

18!= 0.084

46. Let G be the event that the driver is good and N the number of claims. The complement ofgood is bad, so G′ is the event that the driver is bad. Then

P[G | N = 3] =P[G,N = 3]

P[G,N = 3] + P[G′, N = 3]

=e−1 · 13

3!

e−1 · 13

3!+ e−5 · 53

3!

= 0.304

47. (41

)(0.3)1(0.7)3 ·

(61

)(0.1)1(0.9)5(

42

)(0.3)2(0.7)2 ·

(60

)(0.1)0(0.9)6 +

(41

)(0.3)1(0.7)3 ·

(61

)(0.1)1(0.9)5 +

(40

)(0.3)0(0.7)4 ·

(62

)(0.2)1(0.9)4

c©2013 The Infinite Actuary, LLC p. 11 A.7 Solutions

Page 54: The In nite Actuary Exam 1/P Online Seminar A.1 ... · The In nite Actuary Exam 1/P Online Seminar A.1 Fundamentals of Probability Solutions Last updated February 23, 2013 1. Let

= 0.4703

48. P[S ∪ T ] = P[S] + P[T ]− P[S ∩ T ] = P[S] + P[T ]− P[S] · P[T ] by independence

1

4= P[S] + P[S]− (P[S])2 since P[S] = P[T ]

(P[S])2 − 2P[S] +1

4= 0

P[S] =2±

√4− 4 · 1 · 1

4

2

P[S] =2−√

3

2

Note that we discard the solution to the quadratic that is greater than 1 as it isn’t possible.

49.P[first is red | 2nd is red] =

P[both are red]

P[2nd is red]

=

6

9· 5

96

9· 5

9+

3

9· 7

9

=30

51=

10

17

50. There are three possible cases: 1st accident is minor, second is also minor, 1st is minor, 2ndis moderate, or 1st is moderate and 2nd is minor. That gives 0.5 · 0.5 + 0.5 · 0.4 + 0.4 · 0.5 = 0.65

You can also think of it as P[1 moderate, 1 minor] + P[2 minor]/ There are two ways to have1 moderate and 1 minor leaving 2 · 0.4 · 0.5 + 0.52 = 0.65

51. For the next two problems, let us pretend that one die is blue and the other is red. There are4 ways for the sum to be 5 (red=1 and blue=4, or red=2 and blue =3, or red=3 and blue=2, orred=4 and blue=1). 2 of those ways involve a 3, giving an answer of 2/4=1/2. Alternatively,

P[at least one 3 | sum is 5] =P[at least one 3 and sum is 5]

P[sum is 5]

=2/36

4/36=

1

2

52. There are 5 ways to get a sum of 6 (red=1 and blue=5, or red=2 and blue=4, or red=blue=3,or red=4 and blue=2, or red=5 and blue=1). Only one of those 5 ways involves a 3, so the answer

is 1/5

53. Let N be the number of passengers who arrive, so N ∼ Binom(32, 0.9). Then

P[N ≥ 31] = P[N = 32] + P[N = 31]

= (0.9)32 + 32(0.9)31 (0.1)1

c©2013 The Infinite Actuary, LLC p. 12 A.7 Solutions

Page 55: The In nite Actuary Exam 1/P Online Seminar A.1 ... · The In nite Actuary Exam 1/P Online Seminar A.1 Fundamentals of Probability Solutions Last updated February 23, 2013 1. Let

= 0.1564

54. P[max ≤ 4] = P[all 3 are ≤ 4]

= (P[first die ≤ 4])3

=

(4

6

)3

P[max = 4] = P[max ≤ 4]− P[max ≤ 3]

=

(4

6

)3

−(

3

6

)3

=37

216

You could also do it as a sum of 3 cases: one die is 4 and 2 are less than 4, two dice are 4 and oneis less than 4, and all three are 4, giving

P[max ≤ 4] =

(3

1

)·(

1

6

)·(

3

6

)2

+

(3

2

)·(

1

6

)2

·(

3

6

)+

(3

3

)·(

1

6

)3

=27 + 9 + 1

216=

37

216

55. After two rounds, there are 3 cases:

P[both in after 2 rounds] = (0.8)2(0.8)2 = 0.642

P[exactly 1 in after 2 rounds] = 2(0.8)2(1− 0.82) = 2(0.64)(0.36)

P[both out after 2 rounds] = [1− 0.82][1− 0.82] = 0.362

If both are in after 2 rounds, then for both to be out by round 5 they have to both lose in oneof the next 2 rounds, which has probability 0.362. If exactly one is in after 2 rounds, then theyare both out by round 5 if that person loses in one of the next 2 rounds, which has probability1− 0.82 = 0.36. So

P[both out by round 5 | ≥ 1 in after 2 rounds] =0.642 · 0.362 + 2(0.64)(0.36)(0.36)

0.642 + 2(0.64)(0.36)= 0.2516

56. Let N be the number of defects. Since at least 99 of the sampled items are not defective, weare given that N = 0 or N = 1. If N = 0 then the first item is not defective because there are nodefects. If N = 1, then exactly 1 of the 100 items is defective, and it is equally likely to be eachof the 100 items, so the chance that the first item is not defective given N = 1 is 99/100.

P[N = 0 | N ≤ 1] =0.95100

0.95100 + 100(0.05)(0.95)99=

0.95

5.95

P[N = 1 | N ≤ 1] =100(0.05)(0.95)99

0.95100 + 100(0.05)(0.95)99=

5.00

5.95

Note that we could have also said P[N = 1 | N ≤ 1] = 1− P[N = 0 | N ≤ 1]

P[First is not defective | N ≤ 1] = 1 · 0.95

5.95+

99

100

5

5.95=

5.9

5.95

c©2013 The Infinite Actuary, LLC p. 13 A.7 Solutions

Page 56: The In nite Actuary Exam 1/P Online Seminar A.1 ... · The In nite Actuary Exam 1/P Online Seminar A.1 Fundamentals of Probability Solutions Last updated February 23, 2013 1. Let

The Infinite Actuary Exam 1/P Online SeminarB.1 Densities and CDFs SolutionsLast updated August 7, 2012

1. f(x) = c x2

1 =

3∫0

f(x) dx =

3∫0

c x2 dx =c x3

3

∣∣∣∣30

= 9c, c =1

9

0.75 =

t∫0

f(x) dx =

t∫0

1

9x2 dx =

x3

27

∣∣∣∣t0

=t3

27

t =[(27) · (0.75)

]1/3= 2.73

2. f(y) = k(y + 1)−2

0.5 =

2/3∫0

f(y) dy =

2/3∫0

k(y + 1)−2 dy

1

2= − k(y + 1)−1

∣∣∣2/30

=−k53

+k

1

1

2=

2

5k, k =

5

4

1 =

c∫0

f(y) dy =

c∫0

5

4(y + 1)2dy

1 =−5

4(y + 1)−1

∣∣∣∣c0

=−5

4· 1

1 + c+

5

4· 1

1

5

4· 1

1 + c=

1

4, c = 4

3. Let C be the claim size (with C = 0 if there are no claims). We want P[C > 300].

P[C > 300] = P [claim is filed] · P [C > 300 | claim is filed]

= 0.25 ·∞∫

300

2 · 1002

(x+ 100)3dx

= 0.25−1002

(x+ 100)2

∣∣∣∣∞300

= 0 + 0.25 · 1002

4002=

1

64

4.P[X > 0.90] =

∫ 1

0.9

2xdx = x2∣∣10.9

= 1− 0.81 = 0.19

c©2012 The Infinite Actuary, LLC p. 1 B.1 Solutions

Page 57: The In nite Actuary Exam 1/P Online Seminar A.1 ... · The In nite Actuary Exam 1/P Online Seminar A.1 Fundamentals of Probability Solutions Last updated February 23, 2013 1. Let

5.f(x) =

d

dxF (x) =

d

dx

[1−

3∑k=0

e−xxk

k!

]

= 0−3∑

k=0

[−e−xx

k

k!+ ke−x

xk−1

k!

]= e−x

(1 +

x

1+x2

2!+x3

3!

)− e−x

(0 + 1 +

x

1+x2

2!

)= e−x · x

3

3!=

x3e−x

6

Note: later on we will see another approach to this, namely that X has a Gamma distributionwith θ = 1 and α = 4.

6.0.75 =

∫ t

−∞f(x)dx =

∫ t

−∞ex−2dx = ex−2

∣∣∣t−∞

0.75 = et−2

t = 2 + ln(3/4)

7. The only requirements to be a cdf that are likely to be tested is that it be 0 at −∞ and 1, aswell as increasing. (It must also be right continuous, but that has never been tested). All of ourchoices are 0 at −∞ and 1 at ∞, so we only need to check whether each one is increasing. Todo that, we need to verify that the derivative, when defined, is non-negative, and where it is notdifferentiable that we either have continuity or a jump up.

For (i), F ′(x) = 0 if x < 1 or x > 2, and F ′(x) = 2x− 2 > 0 if 1 < x < 2. At 1, F (x) goes from0 to 12 − 2 + 1 = 0, so there is no jump. At 2, F (x) goes from 22 − 2 · 2 + 1 = 1 to 1, so there isagain no jump. Choice (i) is therefore a valid cdf.

For (ii), F ′(x) = 2x− 2 for 0 < x < 1 +√

2, and 2x− 2 < 0 for 0 < x < 1. Since F ′(x) can benegative, we do not have a valid cdf.

For (iii), we have a jump from 0 to 1/2 at 2. That is a jump up, so it is ok. At 1 +√

6/2we go from 1 to 1, so there is no jump. When differentiable, F ′(x) is either 0 or 2x − 2 > 0 for2 < x < 1 +

√6/2, so it is indeed increasing and hence a cdf.

The answer is therefore B

8.1 =

40∫0

f(x) dx =

40∫0

c

(10 + x)2dx

1 =−c

10 + x

∣∣∣∣400

=−c50

+c

10

50 = −c+ 5c = 4c, c = 12.56∫

0

12.5

(10 + x)2dx =

−12.5

10 + x

∣∣∣∣60

=−12.5

16+

12.5

10= 0.469

c©2012 The Infinite Actuary, LLC p. 2 B.1 Solutions

Page 58: The In nite Actuary Exam 1/P Online Seminar A.1 ... · The In nite Actuary Exam 1/P Online Seminar A.1 Fundamentals of Probability Solutions Last updated February 23, 2013 1. Let

9. P [V > 40,000 | V > 10,000] =P [V > 40,000]

P [V > 10,000]=

P [Y ≥ 0.4]

P [Y ≥ 0.1]

=

1∫0.4

k(1− y)4 dy

1∫0.1

k(1− y)4 dy

=−k5

(1− y)5∣∣10.4

−k5

(1− y)5∣∣10.1

=(0.6)5

(0.9)5= 0.132

10. P [X < 2 | X > 1.5] =P [1.5 < X < 2]

P [1.5 < X]

=

2∫1.5

3x−4 dx

∞∫1.5

3x−4 dx

=−x−3

∣∣21.5

−x−3∣∣∞1.5

=0.296− 0.125

0.296− 0= 0.578

11. If the payment is less than 0.5, then the loss is less than 0.5 + C, so we want to find C suchthat

0.64 = P[X ≤ 0.5 + C]

=

∫ 0.5+C

0

2x dx = x2∣∣∣0.5+C0

0.64 = (0.5 + C)2

C = 0.3

12. Let t denote the 30th percentile and s the 70th. Then

0.3 =

t∫200

f(x) dx =

t∫200

2.5(200)2.5

x3.5dx

0.3 = −2002.5 x−2.5∣∣∣t200

= 1− 2002.5 t−2.5

t =

(0.7

2002.5

)−1/2.5= 231

0.7 =

s∫200

f(x) dx = −2002.5 x−2.5∣∣∣s200

= 1− 2002.5 s−2.5

s =

(0.3

2002.5

)−1/2.5= 324

s− t = 324− 231 = 93

c©2012 The Infinite Actuary, LLC p. 3 B.1 Solutions

Page 59: The In nite Actuary Exam 1/P Online Seminar A.1 ... · The In nite Actuary Exam 1/P Online Seminar A.1 Fundamentals of Probability Solutions Last updated February 23, 2013 1. Let

13. P [X > 1] = 1− P [X ≤ 1]

= 1− F (1) = 1− 1

2=

1

2

14.

1 2X

14

12

1

F (x)

jump size = P[X = 1]

If we want P[X ≥ 1] then we want to include the chance that X = 1, namely by finding

P [X ≥ 1] = P [X = 1] + P [X > 1]

=1

4+

1

2=

3

4

Alternatively, we could have used a limit:

P[X ≥ 1] = 1− P[X < 1]

= 1− limx↑1

F (x)

= 1− 1

4=

3

4

15. 0.75 = F (y) = 1− exp

[−1

2(y − a)2

]ln(0.25) = −1

2(y − a)2

y = a+√−2 ln(0.25)

= a+√

2 ln(22) = a+√

4 ln(2) = a+ 2√

ln 2

Note that we want the positive square root since F (a) = 0 and so P [y ≤ a] = 0 so y > a.

16. P[1 ≤ X ≤ 2] = P[X ≤ 2]− P[X < 1]

= F (2)− limx↑1

F (x)

=

(3

4+

2

12

)− 1

8

=19

24

c©2012 The Infinite Actuary, LLC p. 4 B.1 Solutions

Page 60: The In nite Actuary Exam 1/P Online Seminar A.1 ... · The In nite Actuary Exam 1/P Online Seminar A.1 Fundamentals of Probability Solutions Last updated February 23, 2013 1. Let

17.F (x) =

x∫0

f(t) dt =

x∫0

3t2 dt = t3∣∣∣x0

= x3

For 0 ≤ x ≤ 1, x > x3 (see picture) so

|x− F (x)| = |x− x3| = x− x3 for 0 ≤ x ≤ 1

X

Y y = x(1, 1)

y = x3

G(x) = 2

1∫0

|x− F (x)| dx = 2

1∫0

(x− x3

)dx

= 2

(x2

2− x4

4

)∣∣∣∣10

= 2

(1

2− 1

4

)=

1

2

18. The mode is the value of x that maximizes f(x). This can occur at one of the endpoints(x = 0 or x = 3) or where f ′(x) = 0. For this last case,

f(x) =d

dxF (x) =

4x

9− x2

9

f ′(x) =4− 2x

90 < x < 3

0 = f ′(x) =4− 2x

9x = 2

Plugging in the x values that might maximize f(x) and comparing gives

f(x) = 0 if x = 0

f(x) =3

9=

1

3if x = 3

f(x) =8− 4

9=

4

9if x = 2

And we see that the max (and thus the mode) is when x = 2

Side note: if we plug in 0 into1

9

(2x2 − x3

3

)we get 0, so there is no jump in the cdf at x = 0.

Likewise, plugging in 3 gives1

9

[2 · 32 − 33

3

]= 1, so there is no jump in the cdf at x = 3 either.

That means that X is purely continuous. If X had a mixed distribution, then the mode is notdefined for the purposes of this exam.

c©2012 The Infinite Actuary, LLC p. 5 B.1 Solutions

Page 61: The In nite Actuary Exam 1/P Online Seminar A.1 ... · The In nite Actuary Exam 1/P Online Seminar A.1 Fundamentals of Probability Solutions Last updated February 23, 2013 1. Let

19. P[X < 1.5 | X > 0.5] =P[X < 1.5 and X ≥ 0.5]

P[X ≥ 0.5]

=

1.5∫0.5

2 |x− 1|3 dx

2∫0.5

2 |x− 1|3 dx

Because of the absolute value, we will need to break into cases: |x − 1| = x − 1 if x > 1 and|x− 1| = −(x− 1) = 1− x if x < 1

P[X < 1.5 | X > 0.5] =

1∫0.5

2(1− x)3 dx+1.5∫1

2(x− 1)3 dx

1∫0.5

2(1− x)3 dx+2∫1

2(x− 1)3 dx

=−12

(1− x)4∣∣10.5

+ 12(x− 1)4

∣∣1.51

−12

(1− x)4∣∣10.5

+ 12(x− 1)4

∣∣21

=12

(12

)4+ 1

2

(12

)412

(12

)4+ 1

2(1)4

=2

17

20. P[X > 1] =

∫ θ

1

f(x) dx =

∫ θ

1

4x3

θ4dx

3

4=x4

θ4

∣∣∣∣θ1

= 1− 1

θ4

θ =√

2

21. P[|X − 1/2| > 1/4] = P[X < 1/4] + P[X > 3/4]

= 1− P[1/4 ≤ X ≤ 3/4]

= 1−∫ 3/4

1/4

6x(1− x)dx

= 1−(3x2 − 2x3

) ∣∣∣3/41/4

= 0.3125

22. The mode is either an endpoint (x = 0 or x = 1), or a critical value where f ′(x) = 0. At theendpoints, f(0) = f(1) = 0, while between the density is positive, so the mode must be a criticalvalue.

f(x) = cx2(1− x) = cx2 − cx3

f ′(x) = 2cx− 3cx2

0 = 2x− 3x2

x = 0 or x = 2/3

Since we have already ruled out x = 0, the mode must therefore be 2/3

Note that we didn’t have to find c since it doesn’t affect the value of x that maximizes f(x).

c©2012 The Infinite Actuary, LLC p. 6 B.1 Solutions

Page 62: The In nite Actuary Exam 1/P Online Seminar A.1 ... · The In nite Actuary Exam 1/P Online Seminar A.1 Fundamentals of Probability Solutions Last updated February 23, 2013 1. Let

The Infinite Actuary Exam 1/P Online SeminarB.2 Continuous Moments SolutionsLast updated April 4, 2013

1. Since 1 < X < 3, we know that 1 < E[X] < 3 without doing any work, so clearly choices Dand E are wrong. As for the right choice,

1 =

3∫1

cx2 dx =cx3

3

∣∣∣∣31

=26

3c, c =

3

26

E[X] =

3∫1

x f(x) dx =

3∫1

3

26x3 dx

=3

26· 1

4x4

∣∣∣∣31

=3

104(81− 1) =

30

13

2. Using the survival method,

E[X] =

∞∫0

P[X > x] dx

=

3∫0

1 dx+

4∫3

(1− x

5

)dx+

∞∫4

0 dx

= 3 + 1−(x2

10

)∣∣∣∣43

+ 0

= 4− 16− 9

10= 3.3

Or you could do E[X] = 3 · P[X = 3] + 4 P[X = 4] +

4∫3

x · f(x) dx

P[X = 3] = F (3)− limx↑3

F (x) =3

5− 0 =

3

5

P[X = 4] = F (4)− limx↑4

F (x) = 1− 4

5=

1

5

i.e., F (x) has a jump of 3/5 at 3 and 1/5 at 4, so P[X = 3] = 3/5 and P[X = 4] = 1/5

f(x) =d

dxF (x) =

d

dx· x

5=

1

5if 3 < x < 4

E[X] = 3 · 3

5+ 4 · 1

5+

4∫3

x · 1

5dx

=13

5+x2

10

∣∣∣43

=26

10+

7

10= 3.3

c©2013 The Infinite Actuary, LLC p. 1 B.2 Solutions

Page 63: The In nite Actuary Exam 1/P Online Seminar A.1 ... · The In nite Actuary Exam 1/P Online Seminar A.1 Fundamentals of Probability Solutions Last updated February 23, 2013 1. Let

3.E[X] =

∫ k

0

x · f(x)dx =

∫ k

0

2x2

k2dx =

2k3

3k2=

2

3k

E[X2]

=

∫ k

0

x2 · f(x)dx =

∫ k

0

2x3

k2dx =

2k4

4k2=k2

2

Var[X] =k2

2−(

2k

3

)2

=k2

18

2 =k2

18, k = 6

4.E[X] =

4∫−2

x f(x) dx =

0∫−2

x · −x10

+

4∫0

x · x10dx

= −0∫

−2

x2

10dx+

4∫0

x2

10dx = −x

3

30

∣∣∣∣0−2

+x3

30

∣∣∣∣40

= −[0− (−2)3

30

]+

[43

30− 0

]=−8 + 64

30=

28

15

5. Let Y denote the unpaid losses, and X the total losses. Then

Y =

{X if X < 2

2 if X ≥ 2

We can do this in three ways. First, we could write Y = g(X) so

E[Y ] = E[g(X)] =

∫ ∞0.6

g(x) · f(x) dx

=

∫ 2

0.6

x · 2.5(0.6)2.5

x3.5dx+

∫ ∞2

2 · 2.5(0.6)2.5

x3.5dx

=−2.5(0.6)2.5

1.5x1.5

∣∣∣20.6

+−2 · 2.5(0.6)2.5

2.5x2.5

∣∣∣∞2

= 0.934

Or we could use the survival function, which gives

E[Y ] =

∞∫0

P[Y > y] dy

=

0.6∫0

1 dy +

2∫0.6

P[Y > y] dy +

∞∫2

0 dy

= 0.6 +

2∫0.6

P[Y > y] dy

c©2013 The Infinite Actuary, LLC p. 2 B.2 Solutions

Page 64: The In nite Actuary Exam 1/P Online Seminar A.1 ... · The In nite Actuary Exam 1/P Online Seminar A.1 Fundamentals of Probability Solutions Last updated February 23, 2013 1. Let

To find P[Y > y], note that if y < 2 then P[Y > y] = P[X > y] so for 0.6 < y < 2,

P[Y > y] =

∞∫y

2.5 (0.6)2.5

x3.5dx =

−(0.6)2.5

x2.5

∣∣∣∣∞y

=0.62.5

y2.5

so E[Y ] = 0.6 +

2∫0.6

0.62.5

y2.5dy = 0.6− 0.62.5

1.5

1

y1.5

∣∣∣∣20.6

= 0.934

Or: Y is a mixed distribution. Either Y = 2 (when X ≥ 2) or 0.6 < y < 2 and fY (y) =2.5(0.6)2.5

y3.5

E[Y ] =

2∫0.6

yfY (y) dy

+ 2 · P[Y = 2]

=

2∫0.6

2.5(0.6)2.5

y2.5dy + 2

∞∫2

2.5 (0.6)2.5

x3.5dx

=−2.5

1.5

(0.6)2.5

y1.5

∣∣∣∣20.6

+−2 (.6)2.5

x2.5

∣∣∣∣∞2

= 0.934

6. The payment has a mixed distribution, with a density up to 10, and then a positive probabilityof being equal to 10, so

E[Y ] =

10∫1

y · f(y) dy + 10 · P[Y ≥ 10]

=

10∫1

2

y2dy + 10

∞∫10

2

y3dy

=−2

y

∣∣∣∣10

1

+ 10−1

y2

∣∣∣∣∞10

=

(−2

10+ 2

)+

1

10= 1.9

7. There are 3 cases: either there is no damage, partial damage, or total damage. In the partialdamage case with a damage of X, the payment amount is 0 when X < 1, (X − 1) when X > 1and our conditional density is 0.5003e−x/2 so

E[payment | partial damage] =

1∫0

0 · f(x) dx+

15∫1

(x− 1) .5003e−x/2 dx

c©2013 The Infinite Actuary, LLC p. 3 B.2 Solutions

Page 65: The In nite Actuary Exam 1/P Online Seminar A.1 ... · The In nite Actuary Exam 1/P Online Seminar A.1 Fundamentals of Probability Solutions Last updated February 23, 2013 1. Let

To do this, we will use integration by parts, with u and v given by

u = (.5003)(x− 1)

du = .5003 dx

dv = e−x/2 dx

v = −2 e−x/2

15∫1

(x− 1)(.5003)e−x/2 dx = uv∣∣∣x=15

x=1−∫ x=15

x=1

vdu

= (.5003)(x− 1)(−2 e−x/2)∣∣∣15

1−∫ 15

1

−2 e−x/2 (.5003) dx

= .5003[(x− 1)

(− 2 e−x/2

)− 4 e−x/2

] ∣∣∣15

1

= 1.2049 in thousands

= 1,205

So E[Payment] = 0 · P[no loss] + 1,205 · P[Partial loss] + 14,000 · P[total loss] = 328

8.1 =

∞∫0

f(x) dx =

∞∫0

c

(1 + x)4dx

=−c3

(1 + x)−3

∣∣∣∣∞0

= 0 +c

3, c = 3

E[X] =

∞∫0

x · 3

(1 + x)4dx =

∞∫0

f(x) · x dx

To evaluate, substitute u = 1 + x, so du = dx, and x = u− 1

E[X] =

∞∫1

3 · u− 1

u4du =

∞∫1

3

(1

u3− 1

u4

)du

=−3

2u2+

1

u3

∣∣∣∣∞1

=3

2− 1 =

1

2

Alternatively, we could use the survival function:

P[X > x] =

∞∫x

3

(1 + t)4dt =

−1

(1 + t)3

∣∣∣∣∞x

=1

(1 + x)3

E[X] =

∞∫0

P[X > x]dx =

∞∫0

1

(1 + x)3dx

=−1

(1 + x)2· 1

2

∣∣∣∣∞0

=1

2

9. Since X = J +K + L we have

MX(t) = MJ(t)MK(t)ML(t) = (1− 2t)−10

c©2013 The Infinite Actuary, LLC p. 4 B.2 Solutions

Page 66: The In nite Actuary Exam 1/P Online Seminar A.1 ... · The In nite Actuary Exam 1/P Online Seminar A.1 Fundamentals of Probability Solutions Last updated February 23, 2013 1. Let

E[X3]

=d3

dt3MX(t)

∣∣∣∣t=0

d

dtMX(t) = (−10)(−2)(1− 2t)−11

d2

dt2MX(t) = (20)(−11)(−2)(1− 2t)−12

d3

dt3MX(t) = 20 · 22(−12)(−2)(1− 2t)−13

Plugging in t = 0 gives us 20 · 22 · 24(1− 0)−13 = 10,560

10. Let X denote the cost before inflation, so E[X] = 200 and Var[X] = 260. After inflation, ournew cost is 1.2X, so the variance of the new cost is

Var(1.2X) = (1.2)2Var[X] = (1.44)(260) = 374.4

11. Since F (x) is defined piecewise, we may have a mixed distribution with possible discrete piecesat 1 and 2.

P[X = 1] = F (1)− limx↑1

F (x) =12 − 2 + 2

2− 0 =

1

2

P[X = 2] = F (2)− limx↑2

F (x) = 1− 22 − 2 · 2 + 2

2= 0

f(x) =2x− 2

2= x− 1, 1 < x < 2

E[X] = 1 · P[X = 1] +

∫ 2

1

x · f(x)dx

=1

2+

(x3

3− x2

2

)∣∣∣∣21

=1

2+

2

3− −1

6=

4

3

E[X2] = 12 · P[X = 1] +

∫ 2

1

x2 · f(x)dx

=1

2+

(x4

4− x3

3

)∣∣∣∣21

=1

2+

4

3− −1

12=

23

12

Var[X] =23

12−(

4

3

)2

=5

36

Alternatively, since F (x) is defined piecewise we may want to use the survival method.

E[g(X)] =

∫ ∞0

g′(x) · P[X > x]dx

E[X] =

∫ 1

0

(1− 0)dx+

∫ 2

1

(1− x2 − 2x+ 2

2

)dx

c©2013 The Infinite Actuary, LLC p. 5 B.2 Solutions

Page 67: The In nite Actuary Exam 1/P Online Seminar A.1 ... · The In nite Actuary Exam 1/P Online Seminar A.1 Fundamentals of Probability Solutions Last updated February 23, 2013 1. Let

= 1 +

∫ 2

1

(x− x2

2

)dx

= 1 +

(x2

2− x3

6

)∣∣∣∣21

= 1 +

(2− 4

3− 1

2+

1

6

)=

4

3

E[X2]

=

∫ 1

0

2x · (1− 0)dx+

∫ 2

1

2x ·(x− x2

2

)dx

= x2∣∣∣10

+

(2

3x3 − 1

4x4

)∣∣∣∣21

= 1 +

(16

3− 4− 2

3+

1

4

)=

23

12

Var[X] =23

12−(

4

3

)2

=5

36

12. Y has a mixed distribution. Either the machine fails in the first 4 years, during which timefY (x) = fX(x) = 1/5, or the machine doesn’t fail in the first 4 years and is replaced at time 4 (soP[Y = 4] = P[X ≥ 4] = 1/5. To find the variance, we will compute

Var(Y ) = E[Y 2]− (E[Y ])2

E[Y ] =

4∫0

y · 1

5dy + 4 · P[Y = 4]

=y2

10

∣∣∣∣40

+ 4

5∫4

1

5dy

=8

5+ 4 · 1

5=

12

5

E[Y 2]

=

4∫0

y2 · 1

5dy + 42 · P[Y = 4]

=y3

15

∣∣∣∣40

+ 42 · 1

5=

112

15

Var[Y ] =112

15−(

12

5

)2

= 1.7

13. The value after 3 years of use is 100(0.5)X , and so we want

E[100(0.5)X

]= 100 · E

[0.5X

]= 100 · E

[(eln 0.5

)X]= 100 · E

[eX ln 0.5

]= 100MX(ln 0.5)

c©2013 The Infinite Actuary, LLC p. 6 B.2 Solutions

Page 68: The In nite Actuary Exam 1/P Online Seminar A.1 ... · The In nite Actuary Exam 1/P Online Seminar A.1 Fundamentals of Probability Solutions Last updated February 23, 2013 1. Let

= 100 · 1

1− 2 ln 0.5= 41.9

14. The man purchases the life insurance policy on his 40th birthday, so we know that he livedto be 40. The policy pays 5,000 if he dies before turning 50, and 0 otherwise, so the expectedpayment is

5,000 · P[T < 50 | T > 40] = 5,000 · P[40 < T < 50]

P[40 < T ]

= 5,000 · F (50)− F (40)

1− F (40)

= 5,000 · e(1−1.140)/1000 − e(1−1.150)/1000

e(1−1.140)/1000

= 348

15. E[total benefits] = 32E[one person]

= 32 · 1

6E[payment | there is a claim]

=32

6

1∫0

yf(y) dy =16

3

1∫0

(2y − 2y2

)dy

=16

3

(y2 − 2 y3

3

)∣∣∣∣10

=16

3

(1− 2

3

)=

16

9

16. Let X denote the amount of a claim.

E[X] = E[X | good driver] · P[good] + E[X | bad] · P[bad]

= 1,400 (0.6) + (2, 000)(0.4) = 1,640

E[X2]

= E[X2 | good] · P[good] + E[X2 | bad] · P[bad]

= (40,000 + 1,4002) · 0.6 + (250,000 + 2,0002) · 0.4= 2,900,000

Var[X] = E[X2]− [EX]2 = 2,900,000− 1,6402

= 210,400

17. We will use Var[X] = E[X2]− [EX]2.

E[X] =d

dtM(t)

∣∣∣t=0

= 9

(2 + et

3

)8

· et

3

∣∣∣t=0

= 9 · 18 · 1

3= 3

E[X2]

=d2

dt2M(t)

∣∣∣∣t=0

=d

dt3

(2 + et

3

)8

et

∣∣∣∣∣t=0

c©2013 The Infinite Actuary, LLC p. 7 B.2 Solutions

Page 69: The In nite Actuary Exam 1/P Online Seminar A.1 ... · The In nite Actuary Exam 1/P Online Seminar A.1 Fundamentals of Probability Solutions Last updated February 23, 2013 1. Let

= 3 · 8(

2 + et

3

)7et

3et + 3

(2 + et

3

)8

· et∣∣∣∣∣t=0

= 8 + 3 = 11

Var[X] = 11− 32 = 2

If you prefer to use the cumulant g(t) = ln[M(t)] to find the variance,

g(t) = ln[M(t)] = 9 ln

(2 + et

3

)g′(t) = 9 ·

13et

2+et

3

=9et

2 + et

g′′(t) =9et

2 + et− 9e2t

(2 + et)2

g′′(0) =9

3− 9

32= 2

18. The payment is 1 if X > 1 and X if X < 1.

P[X > 1] =

3∫1

x(4− x)

9dx =

3∫1

4x− x2

9dx

=2x2

9− x3

27

∣∣∣∣31

= 1− 5

27=

22

27

E(payment) =

1∫0

x f(x) dx+ 1 · P[X ≥ 1]

=

1∫0

4x2 − x3

9dx+

22

27

=4x3

27− x4

36

∣∣∣∣10

+22

27

=4

27− 1

36+

22

27= 0.935

19. E[X] =d

dtMX (t)

∣∣∣∣t=0

=d

dtet

2+3t

∣∣∣∣t=0

= (2 t+ 3)et2+3t

∣∣∣t=0

= 3 · 1 = 3

20. From the previous problem we know that E[X] = 3.

E[X2]

=d2

dt2MX (t)

∣∣∣t=0

=d

dt(2 t+ 3)et

2+3t∣∣∣t=0

c©2013 The Infinite Actuary, LLC p. 8 B.2 Solutions

Page 70: The In nite Actuary Exam 1/P Online Seminar A.1 ... · The In nite Actuary Exam 1/P Online Seminar A.1 Fundamentals of Probability Solutions Last updated February 23, 2013 1. Let

= 2et2+3t + (2 t+ 3)2et

2+3t∣∣∣t=0

= 2 + 32

Var[X] = (2 + 32)− 32 = 2

Alternatively, since we want the variance we could have taken the second derivative of the cumulantgenerating function at 0:

g(t) = ln[MX(t)] = t2 + 3t

g′(t) = 2t+ 3

g′′(t) = 2, g′′(0) = 2

21. Note that A, D, and E aren’t possible answer choices since MY (0) = E[e0·Y ] = 1 and plugging

in t = 0 into those choices gives you something that isn’t 1.

MY (t) = E[et Y ]

= E[et(2X+3)]

= E[e(2t)X+3t]

= e3tE[e(2t)X ]

= e3tMX(2t)

= e3te(2t)2+3·2t

= e4t2+9t

The third to last line could also have been found using the fact that the MGF of aX+b is ebtMX(at)with a = 2 and b = 3.

22.M(t) =

e3t

1− t2= e3t · (1− t2)−1

M ′(t) = 3e3t · (1− t2)−1 + e3t · 2t(1− t2)−2

M ′(0) = E[X] = 3

M ′′(t) = 9e3t · (1− t2)−1 + 3e3t · 2t(1− t2)−2 + e3t · 2(1− t2)−2 + e3t · 2t · 2t · 2(1− t2)−3

M ′′(0) = E[X2] = 9 + 0 + 2 + 0 = 11

Var[X] = 11− 32 = 2

or using the cumulant generating function,

g(t) = lnMX(t) = 3t− ln(1− t2)

g′(t) = 3 +2t

1− t2g′(0) = E[X] = 3

g′′(t) = 0 + 2 · (1− t2)−1 + 2t · 2t · 2(1− t2)−2

g′′(0) = Var[X] = 2

and the answer is C

c©2013 The Infinite Actuary, LLC p. 9 B.2 Solutions

Page 71: The In nite Actuary Exam 1/P Online Seminar A.1 ... · The In nite Actuary Exam 1/P Online Seminar A.1 Fundamentals of Probability Solutions Last updated February 23, 2013 1. Let

23.E[X] =

∫ 2

0

x · x2dx =

x3

3 · 2

∣∣∣∣20

=4

3

E|X − EX| =∫ 4/3

0

−(x− 4

3

)f(x)dx+

∫ 2

4/3

(x− 4

3

)f(x)dx

=

∫ 4/3

0

(2x

3− x2

2

)dx+

∫ 2

4/3

(x2

2− 2x

3

)dx

=

((4/3)2

3− (4/3)3

6

)+

(23

6− 22

3

)−(

(4/3)3

6− (4/3)2

3

)=

32

81

24.E[X] =

2∫−2

x f(x) dx =

0∫−2

x−x4dx+

2∫0

xx

4dx

=−x3

12

∣∣∣0−2

+x3

12

∣∣∣20

=(−2)3

12+

23

12− 0 = 0

E[X2]

=

2∫−2

x2 f(x) dx =

0∫−2

−x3

4dx+

2∫0

x3

4dx

=−x4

16

∣∣∣0−2

+x4

16

∣∣∣20

=(− 0 + 1

)+(1− 0

)= 2

Var[X] = E[X2]− (EX)2 = 2− 02 = 2

25.Coefficient of Var =

SD[X]

E[X]

E[X] =

3∫−1

x f(x) dx =

0∫−1

−x2

5dx+

3∫0

x2

5dx

=−x3

15

∣∣∣0−1

+x3

15

∣∣∣30

=(−1)3

15+

33

15− 0 =

26

15

E[X2]

=

3∫−1

x2 f(x) dx =

0∫−1

−x3

5dx+

3∫0

x3

5dx

=−x4

20

∣∣∣0−1

+x4

20

∣∣∣30

=34 + 14

20=

41

10

E[X] =26

15E[X2]

=41

10

c©2013 The Infinite Actuary, LLC p. 10 B.2 Solutions

Page 72: The In nite Actuary Exam 1/P Online Seminar A.1 ... · The In nite Actuary Exam 1/P Online Seminar A.1 Fundamentals of Probability Solutions Last updated February 23, 2013 1. Let

Var[X] =41

10−(

26

15

)2

= 1.096

Coefficient of Var =

√1.096

26/15= 0.604

26. Since our density involves |x+ 1|, we want to break things into two cases: one when x+ 1 > 0,i.e., x > −1, and one when x+ 1 < 0, i.e., when x < −1. Doing so gives

E[X] =

−1∫−∞

x f(x) dx+

∞∫−1

x f(x) dx

=

−1∫−∞

x

2· ex+1 dx+

∞∫−1

x

2· e−x−1 dx

And for both of these integrals we will use integration by parts. Let u = x/2 and dv = ex+1dx sodu = dx/2 and v = ex+1 to get∫

x

2ex+1dx =

x

2ex+1 −

∫ex+1dx

2

=x

2ex+1 − 1

2ex+1

−1∫−∞

x

2ex+1dx =

x

2ex+1 − 1

2ex+1

∣∣∣∣−1

−∞

=−1

2− 1

2= −1

and for the other one, again let u = x/2 but now set dv = e−x−1dx so v = −e−x−1. Then∫x

2e−x−1 dx =

x

2

(−e−x−1

)−∫−e−x−1dx

2

=−x2e−x−1 − e−x−1

2∞∫−1

x

2e−x−1 dx =

−x2e−x−1 − e−x−1

2

∣∣∣∣∞−1

= (0− 0)−(

+1

2e0 − 1

2

)= 0

So E[X] = −1 + 0 = −1

Alternatively, f(x) =1

2e−(x+1) is symmetric about x = −1, so E[X] = −1

27. We are given that E[X] = µ = 3 and that E [X2] = 13. That gives us that Var[X] = 13−32 = 4so σ = 2. Plugging into Chebyshev’s inequality,

P[|X − µ| > k σ] ≤ 1

k2

c©2013 The Infinite Actuary, LLC p. 11 B.2 Solutions

Page 73: The In nite Actuary Exam 1/P Online Seminar A.1 ... · The In nite Actuary Exam 1/P Online Seminar A.1 Fundamentals of Probability Solutions Last updated February 23, 2013 1. Let

P[|X − µ| > 6] = P[|X − 3| > 3σ]

P[|X − 3| > 6] ≤ 1

32=

1

9

28. In this problem it isn’t specified that X is symmetric, so all we can conclude is that

P[X > 9] = P[X − 3 > 9− 3] = P[X − 3 > 6]

≤ P[|X − 3| > 6] =1

9

29. This problem is different in that now we know that X is symmetric, so P[X − µ > kσ] =1

2P[|X − µ| > kσ], and we can use Chebyshev’s inequality to bound that:

P[X > 7] = P[X − 1 > 7− 1]

= P[X − µ > 2σ]

≤ 1

2· 1

22=

1

8

30. The slope for 80 < x < 120 is −0.01/40 so our density is

f(x) =

0.01 0 < x < 80

0.01− (x− 80) · 0.01

4080 < x < 120

and integrating x · f(x) gives

E[X] =

∫ 80

0

x(0.01) dx+

∫ 120

80

x

[0.01− (x− 80)(0.01)

40

]dx

E[X] = 802 · 0.005 +

∫ 120

80

x(0.01)− x2(0.01)

40+ x(0.02) dx

E[X] = 802 · 0.005 + (1202 − 802)(0.005 + 0.010)− 1203 − 803

40 · 3· 0.01

E[X] =152

3= 50.7

31. The area is length · width = 2w · w = 2w2, so the expected area is∫ 1

0

2w2f(w) dw =

∫ 1

0

2 · 3w4 dw =6

5= 1.200

32. M(0) = 1 since M(t) is an MGF. Likewise, in order to be MGFs, the other functions mustalso be 0 at t = 0, But (iii) fails that as 3e0 ·M(0) = 3 so (iii) is not an MGF. The other two are:M(0) ·M(0) = 1 and e0 ·M(0) = 1, so they are probably MGFs, meaning the answer is probably

C (there are other conditions that an MGF must satisfy, but they are too complicated to verify

c©2013 The Infinite Actuary, LLC p. 12 B.2 Solutions

Page 74: The In nite Actuary Exam 1/P Online Seminar A.1 ... · The In nite Actuary Exam 1/P Online Seminar A.1 Fundamentals of Probability Solutions Last updated February 23, 2013 1. Let

under exam conditions.) To verify, note that (i) is the MGF of X1 + 3X2 where X1 and X2 are iidvariables with the same distribution as X, and (ii) is the MGF of X + 3.

33. M(0) = 1

M ′(0) = E[X] =

∫ 1

0

2x2 dx =2

3

M ′′(0) = E[X2] =

∫ 1

0

2x3 dx =2

4

M(0) +M ′(0) +M ′′(0) =13

6

34.E[Payment] =

∫ 2

1

(x− 1)f(x) dx

=

∫ 2

1

3x3

8− 3x2

8dx

=

(3

4

16

8− 8

8

)−(

3

4

1

8− 1

8

)= 0.53

35.E[Payment] =

∫ ∞2

(3

x3− 6

x4

)dx

=3

8− 6

24=

1

8

36. Let t be the 75th percentile of losses that result in positive payments. To result in a payment,the loss must exceed 3, so being the 75th percentile means that

0.75 = P[X ≤ t | X > 3]

0.25 = P[X > t | X > 3] =P[X > t]

P[X > 3]

0.25 · P[X > 3] = P[X > t]

0.25 · 1

32=

1

t2

t = 6

Comments: You could also have used P[X ≤ t | X > 3] = P[3 < X ≤ t]/P[X > t]. Also, we usedthe fact that P[X > x] = 1/x2.

c©2013 The Infinite Actuary, LLC p. 13 B.2 Solutions

Page 75: The In nite Actuary Exam 1/P Online Seminar A.1 ... · The In nite Actuary Exam 1/P Online Seminar A.1 Fundamentals of Probability Solutions Last updated February 23, 2013 1. Let

The Infinite Actuary Exam 1/P Online SeminarB.3 Key Continuous Distributions SolutionsLast updated January 11, 2012

1. Let X be the loss amount, so X is uniform on (0, c). The payment is X − 5 if X > 5, and 0otherwise. Given X > 5, the payment is uniform on (0, c− 5) so

8 = E[payment] = E[X − 5|X > 5] · P[X > 5]

=c− 5

2· c− 5

c16c = c2 − 10c+ 25

0 = c2 − 26c+ 25

c = 25

Note: c = 1 is also a solution to the quadratic, but not to our problem as we need c > 5 in orderfor it to be possible for a payment to be made.

2. Let X denote the loss amount. The expected payment is E[X − 20 | X > 20] · P[X > 20] =40 · 0.8 = 32, and the expected loss amount is 50, so the expected uncovered loss is 50− 32 = 18.

Alternatively, the uncovered loss Y is a mixed distribution that has density 1/100 for 0 < y < 20and a probability of 0.8 of being equal to 20, so

E[Y ] =

∫ 20

0

0.01dy + 20 · 0.8 = 2 + 16 = 18

3. Let T be the amount of time spent in the hospital, and Y the payment. The payment is $100per day, rounded up, so Y is a discrete random variable that can be 100, 200, or 300. Making atable, we get

t y P[Y = y]0 < t < 1 100 1/31 < t < 2 200 1/32 < t < 3 300 1/3

and we want E[Y ] =1

3· 100 +

1

3· 200 +

1

3· 300 = 200

4. E[X] =1 + a

2

Var[X] =(a− 1)2

121 + a

2= 6 · (a− 1)2

121 + a = a2 − 2a+ 1

a2 − 3a = 0

a = 3

Again, the quadratic has an extraneous solution (a = 0) that contradicts the assumptions of ourproblem so we discard it.

c©2012 The Infinite Actuary, LLC p. 1 B.3 Solutions

Page 76: The In nite Actuary Exam 1/P Online Seminar A.1 ... · The In nite Actuary Exam 1/P Online Seminar A.1 Fundamentals of Probability Solutions Last updated February 23, 2013 1. Let

5. Let X denote the loss amount, so the payment is X − 500 if X > 500 and 0 otherwise. Then

E[Payment] = E[X − 500 | X > 500] · P[X > 500]

=0 + 4500

2· 4500

5000= 2,025

6. Saying that we want the average value over the interval means that we want E[c(T )] where Tis chosen uniformly from the interval. The interval length is 2, so f(t) = 1/2.

E[c(T )] =

∫ 2

0

c(t) f(t) dt =

2∫0

t cos (t2)

2· 1

2dt

To do this integral, substitute u = t2, so du = 2tdt. Then tdt/4 = du/8, so we are now integratingcos(u)du/8. The limits also change: when t = 0 we have u = 0, so the lower limit remains 0, butwhen t = 2 we get u = 4 so the upper limit becomes 4, giving

E[c(T )] =

4∫0

cos (u)

8du

=1

8sinu

∣∣∣∣40

=1

8sin 4

This problem is from an older syllabus. It is theoretically still testable, but in practice you rarelysee trig functions on the exam today.

7. We want to find P[T ≤ 80] and are given that T is an exponential with P[T ≤ 50] = 0.3.Plugging into the formula for the cdf of an exponential gives

P[T > t] = e−λt = e−t/θ

P[T ≤ t] = 1− e−λt

0.3 = 1− e−λ·50

e−λ·50 = 0.7

−λ · 50 = ln(0.7)

λ = 0.00713

P[T ≤ 80] = 1− e−λ·80 = 0.4349

8. Let T denote the lifetime of one printer, and X the payment amount for that printer. We want100E[X]. Since the payment is either 0, 100, or 200, we have a discrete payment amount and cansimply list the possible payments and their probabilities to find E[X].

X T P[X = x]200 t ≤ 1 P[T ≤ 1] = 1− e−1/2 = 0.3934100 1 < t ≤ 2 P[T ≤ 2]− P[T ≤ 1] = e−1/2 − e−2/2 = 0.23870 t > 2 P[T > 2] = e−2/2 = 0.3679

So 100E[X] = 100 [0.3934 · 200 + 0.2387 · 100 + 0.3679 · 0] = 10,256

c©2012 The Infinite Actuary, LLC p. 2 B.3 Solutions

Page 77: The In nite Actuary Exam 1/P Online Seminar A.1 ... · The In nite Actuary Exam 1/P Online Seminar A.1 Fundamentals of Probability Solutions Last updated February 23, 2013 1. Let

9. Let T be the failure time and X the discovery time. Since the machine isn’t checked for thefirst two years, P[X > x] = 1 for x < 2. After two years, a failure is discovered when it occurs, soP[X > x] = P[T > x] = e−x/3 for x > 2. Using the survival function method,

E[X] =

∞∫0

P[X > x] dx

=

2∫0

1 dx+

∞∫2

e−x/3 dx

= 2 +(−3 e−x/3

) ∣∣∣∞2

= 2 + 3 e−2/3

Alternatively, X has a mixed distribution with a positive probability of being equal to 2 (X = 2if T < 2) and density (1/3)e−x/3 for x > 2, so

E[X] = 2 · P[X = 2] +

∞∫2

x · f(x) dx

That expression is also what we get if you view X as g(T ) with g(T ) = 2 when T ≤ 2 and g(T ) = Tfor T > 2 and integrate g(t) · f(t)dt. To compute this integral, we can use tabular integration

differentiate integrate

1

3x e−x/3

1

3−3 e−x/3

0 9 e−x/3

+

so

∞∫2

xf(x) dx =−1

3x · 3 · e−x/3 − 3 e−x/3

∣∣∣∣∞2

= 2 e−2/3 + 3 e−2/3

2 · P[X = 2] = 2P[T ≤ 2] = 2[1− e−2/3] = 2− 2 e−2/3

E[X] =(2− 2 e−2/3

)+ 5 e−2/3 = 2 + 3 e−2/3

Finally, we can use the memoryless property:

E[X] = 2 · P[T ≤ 2] + E[T | T > 2] · P[T > 2]

= 2(1− e−2/3) + (E[T − 2 | T > 2] + 2) · P[T > 2]

= 2(1− e−2/3) + E[T ] · e−2/3 + 2 · e−2/3 = 2 + 3e−2/3

10. Let T be the failure time, so P[T > t] = e−t/10 and P[T ≤ t] = 1− e−t/10.

E[Payment] = x · P[T ≤ 1] +x

2· P[1 < T ≤ 3] = x · P[T < 1] +

x

2· (P[T ≤ 3]− P[T ≤ 1])

c©2012 The Infinite Actuary, LLC p. 3 B.3 Solutions

Page 78: The In nite Actuary Exam 1/P Online Seminar A.1 ... · The In nite Actuary Exam 1/P Online Seminar A.1 Fundamentals of Probability Solutions Last updated February 23, 2013 1. Let

1,000 = x(

1− e−1/10)

+x

2

(1− e−3/10 −

(1− e−1/10

))1,000 = x · 0.177, x = 5,644

11. First, let’s ignore the maximum benefit and find the median loss. We can recognize the densityof f(x) as that of an exponential random variable. For an exponential, f(x) = λe−λx, and we havece−0.004x, so c = λ = 0.004.

For the median loss,

0.5 = P[X ≤ t] = 1− e−0.004t

e−0.004 t = 0.5

t =ln(0.5)

−0.004= 173

So the median loss amount is 173. Since 173 is less than the maximum benefit of 250, a loss of173 results in a benefit of 173 as well, and the median benefit is 173

12. Let X be the failure time.

0.5 = P[X ≤ 4] = 1− e−4λ

λ =ln(0.5)

−4P[X ≥ 5] = P[X > 5] since X is continuous

= e−λ·5

= e−5 ln(0.5)/(−4) = 0.42

13. Since we are looking only at those losses that exceed the deductible, we have a conditionalstatement. Conditioning and having an exponential distribution is a hint that we probably wantto use the memoryless property, which says that the distribution of X− c, given that X > c, is thesame as the original random variable. If you prefer, the distribution of the payment, given thatthere is a payment, is the same as the distribution of the original loss.

Here, we want the 95th percentile of losses that exceed the deductible, so we are conditioningon X > 100. First, note that the 95th percentile of the loss (with no conditioning) is

0.95 = P[X ≤ x] = 1− e−x/300

− ln(0.05) =x

300x = 898.8

The memoryless property means that the 95th percentile of the original loss distribution is equalto the 95th percentile of the payment given that the loss exceeds the deductible, so 899 is also the95th percentile of the payment given loss exceeds the deductible.

The loss amount is the payment plus the deductible, so the 95th percentile of the loss, giventhat X > 100, is 899 + 100 = 999

If you prefer not to use the memoryless property, then

0.95 = P[X ≤ t | X > 100] =P[100 < X ≤ t]

P[X > 100]

c©2012 The Infinite Actuary, LLC p. 4 B.3 Solutions

Page 79: The In nite Actuary Exam 1/P Online Seminar A.1 ... · The In nite Actuary Exam 1/P Online Seminar A.1 Fundamentals of Probability Solutions Last updated February 23, 2013 1. Let

0.95 =

(1− e−t/300

)−(1− e−100/300

)e−100/300

0.95 =e−1/3 − e−t/300

e−1/3

(0.95− 1)e−1/3 = e−t/300

t = 999

14. Since we care about the effect of a deductible on payment amounts, we want to use thememoryless property. Initially, our loss amounts are exponential with mean θ. After we impose adeductible of d, our expected payment is 0.9θ. But the expected payment amount given that theloss exceeds the deductible is θ by the memoryless property, by which in symbols I mean

E[X − d | X > d] = E[X] = θ

E[Payment] = E[Payment | X > d] · P[X > d] + E[Payment | X ≤ d] · P[X ≤ d]

0.9θ = θ · P[X > d] + 0 · P[X ≤ d]

0.9 = P[X > d]

E[Payment2

]= E

[Payment2

]· P[X > d] + 0 · P[X ≤ d]

= (2θ2) · 0.9 = 1.8θ2

Var[Payment] = 1.8θ2 − (0.9θ)2 = 0.99θ2

for a 1% reduction.

15. Since the density f(x) = xe−x, then X is a Gamma random variable with α = 2 and θ = 1.P[X > 1] = e−1/1 + (1/1)e−1/1 = 2e−1 = 0.736. If there are 100 possible claims, each resulting in a

payment with probability 0.736, then the expected number of payments is 73.6

16.E[Payment] =

7∫0

v(t)f(t) dt+

∫ ∞7

0 · f(t) dt

=

7∫0

e7−0.2t(0.1)e−0.1t dt

= 0.1e77∫

0

e−0.3t dt =−1

3e7e−0.3t

∣∣∣∣70

=1

3e7 − 1

3e4.9 = 320.78

17. f(x) = λe−λx

F (x) = 1− e−λx

0.5 = 1− e−λ/3

−λ3

= ln(0.5) = − ln(2)

c©2012 The Infinite Actuary, LLC p. 5 B.3 Solutions

Page 80: The In nite Actuary Exam 1/P Online Seminar A.1 ... · The In nite Actuary Exam 1/P Online Seminar A.1 Fundamentals of Probability Solutions Last updated February 23, 2013 1. Let

λ = 3 ln(2)

18. f(x) =1

2e−x/2

F (x) = 1− e−x/2

0.25 = 1− e−x/2

−x2

= ln(0.75) = ln

(3

4

)x = −2 ln

(3

4

)= ln

[(3

4

)−2]

= ln

(16

9

)

19. P[T > t] = e−t/15

P[T > 30] = e−30/15 = e−2

20. P[T > 45 | T > 30] =P[T > 45]

P[T > 30]

=e−45/15

e−45/30= e−3−(−2) = e−1

Or, since we are conditioning an exponential, we can use the memoryless property:

P[T > 45 | T > 30] = P[T − 30 > 45− 30 | T > 30]

= P[T − 30 > 15 | T > 30]

= P[T > 15] = e−15/15 = e−1

21. We are summing 3 iid exponential random variables, and thus have a Gamma distributionwith α = 3 and θ = 1/4. So we want to find the Gamma cdf at 1/2. The easiest way to do it isto just know the Gamma cdf:

α = 1 : P[X ≤ t] = 1− e−t/θ

α = 2 : P[X ≤ t] = 1− e−t/θ −(t

θ

)e−t/θ

α = 3 : P[X ≤ t] = 1− e−t/θ −(t

θ

)e−t/θ − (t/θ)2

2!· e−t/θ

To remember these, the α = 1 case is just the CDF of an exponential, and then for larger α, eachtime α goes up by 1, we get one more term that is the next term of the probability distribution ofa Poisson(t/θ) random variable.

Plugging in t = 1/2 and α = 3 gives us

P

[X ≤ 1

2

]= 1− e−2 − 2e−2 − 22

2!e−2

c©2012 The Infinite Actuary, LLC p. 6 B.3 Solutions

Page 81: The In nite Actuary Exam 1/P Online Seminar A.1 ... · The In nite Actuary Exam 1/P Online Seminar A.1 Fundamentals of Probability Solutions Last updated February 23, 2013 1. Let

= 1− 5e−2

If you prefer, we could also do

P

[X ≤ 1

2

]=

∫ 1/2

0

xα−1

θα(α− 1)!e−x/θdx

=

∫ 1/2

0

x2

2!(1/4)3e−4xdx

and then you can evaluate that integral using tabular integration or integration by parts.

22. If Y is exponential with mean θ then E(Y ) = θ and Var(Y ) = θ2.A Gamma is a sum of α exponentials, so if X is Gamma (α, θ) then E[X] = αθ and Var[X] = αθ2.For us that means that

4 = E[X] = αθ

8 = Var[X] = αθ2

2 = θ, 2 = α

fX (x) =xα−1e−x/θ

θα(α− 1)!=xe−x/2

4

P[X ≤ 4] =

4∫0

x

4e−x/2 dx

=

(−x2e−x/2 − e−x/2

)∣∣∣∣40

= 1− 3 e−2

Or if you prefer the memorization / connection the Poisson route

F (x) = 1− e−x/θ −(xθ

)e−x/θ

F (4) = 1− e−4/2 − 2e−4/2 = 1− 3e−2

23. If X is a Gamma(α, θ) random variable then MX(t) =

(1

1− θt

)α. We are given that

MX(t) = (1− 3t)−4 so α = 4 and θ = 3. That means that

E[X] = αθ = 4 · 3 = 12

Var[X] = αθ2 = 4 · 9 = 36

SD[X] = 6

CV[X] =SD[X]

E[X]=

6

12=

1

2

If you don’t recognize the MGF, then you can do

MX(t) = (1− 3t)−4

c©2012 The Infinite Actuary, LLC p. 7 B.3 Solutions

Page 82: The In nite Actuary Exam 1/P Online Seminar A.1 ... · The In nite Actuary Exam 1/P Online Seminar A.1 Fundamentals of Probability Solutions Last updated February 23, 2013 1. Let

d

dtMX(t) = (−3)(−4)(1− 3t)−5 = 12(1− 3t)−5

E[X] = 12(1− 3 · 0)−5 = 12

d2

dt2MX(t) = 12(−3)(−5)(1− 3t)−6 = 180(1− 3t)−6

E[X2] = 180

Var[X] = 180− 122 = 36, SD[X] = 6

CV[X] =6

12=

1

2

24. This is somewhat unlikely to appear on the exam as the only way to do this problem is to

recognize that for an exponential with mean θ, MX(t) =1

1− θt. Here,

2

2− t=

2 · 12

(2− t) · 12

=1

1− (1/2)t

So X is an exponential with θ = 1/2 and so

P[X > 1] = e−t/θ = e−1/(1/2) = e−2

25. Let T be the failure time.

P[2 < T < 4] = F (4)− F (2)

= 1− e−4/θ −(1− e−2/θ

)= e−2/θ −

(e−2/θ

)20.25 = e−2/θ

(1− e−2/θ

)0.5 = e−2/θ

P[X > 6] = e−6/θ =(e−2/θ

)3=

1

8

26. Let X denote the lifetime.

SD[X] =√

Var[X] =√θ2 = θ

θ = E[X] = 3

27. Since X is continuous, P[X = 2] = 0 so

P[X < 2] = P[X ≤ 2] = 1− e−2/θ

P[X ≥ 2] = P[X > 2] = e−2/θ

1− e−2/θ = e−2/θ

1

2= e−2/θ

θ =−2

ln(1/2)=

−2

− ln(2)=

2

ln(2)

c©2012 The Infinite Actuary, LLC p. 8 B.3 Solutions

Page 83: The In nite Actuary Exam 1/P Online Seminar A.1 ... · The In nite Actuary Exam 1/P Online Seminar A.1 Fundamentals of Probability Solutions Last updated February 23, 2013 1. Let

The Infinite Actuary Exam 1/P Online SeminarB.4 Transformations of Random Variables SolutionsLast updated February 19, 2013

1. First, let’s figure out which x correspond to −3 < y < −2.75.

−3 < y < −2.75

−3 < x2 − 3 < −2.75

0 < x2 < 0.25

0 < x < 0.5

so −3 < y < −2.75 if 0 < x < 0.5. In that region, F (x) = x2, so

P[Y ≤ y] = P[X2 − 3 ≤ y]

= P[X2 ≤ y + 3] = P[X ≤√y + 3]

= FX(√y + 3) = y + 3

2. −2.75 < y < 2

−2.75 < x2 − 3 < 2

0.25 < x2 < 1

0.5 < x < 1

FX(x) = x

P[Y ≤ y] = P[X2 − 3 ≤ y]

= P[X2 ≤ y + 3] = P[X ≤√y + 3]

= FX(√y + 3) =

√y + 3

Side note: X has a mixed distribution since P[X = 0.5] = 0.5− 0.52 = 0.25.

3. Recall that if Y = g(X) where g is increasing then

FY (y) =

g−1(y)∫−∞

fX(x) dx

fY (y) = fX(g−1(y))

∣∣∣∣ ddyg−1(y)

∣∣∣∣For us, y = g(x) = 10x0.8 so( y

10

)1.25= x = g−1(y)

d

dy

[g−1(y)

]=

1.25( y

10

)0.2510

c©2013 The Infinite Actuary, LLC p. 1 B.4 Solutions

Page 84: The In nite Actuary Exam 1/P Online Seminar A.1 ... · The In nite Actuary Exam 1/P Online Seminar A.1 Fundamentals of Probability Solutions Last updated February 23, 2013 1. Let

fX(x) = e−x

fY (y) = e−(y/10)1.25

· 0.125(0.1y)0.25

Alternatively, we could have used the cdf:

P[Y ≤ y] = P[10X0.8 ≤ y]

FY (y) = P[X ≤ (0.1y)1.25

]= 1− e−(0.1y)1.25

fY (y) = −e−(0.1y)1.25[− (0.1y)0.25(1.25)(0.1)]

= (0.125)e−(0.1y)1.25

(0.1y)0.25

4. R =Number Customers Served

Time=

10

T, T =

10

R

R = g(T ), g(t) =10

t, g−1(r) =

10

r

fR(r) = fT(g−1(r)

)·∣∣∣∣ ddr · 10

r

∣∣∣∣=

1

4

∣∣∣∣−10

r2

∣∣∣∣ =5

2 r2

Or, using the CDF method,

g(T ) =10

T= R is a decreasing function of R

P[R ≤ r] = P

[10

T≤ r

]= P

[T ≥ 10

r

]T is uniform on (8, 12) so

P[R ≤ r] = P

[T ≥ 10

r

]=

12− 10

r4

= 3− 5

2r

fR(r) =5

2r2

5. Let X denote the profit of Company II, and Z the profit of Company I. Then

X = 2Z,Z =X

2

g(z) = 2z = x g−1(x) =x

2= z

fX(x) = fZ(g−1(x)

)·∣∣∣∣ ddx g−1(x)

∣∣∣∣= f

(x2

) ∣∣∣∣ ddx x2∣∣∣∣ =

1

2f(x

2

)

c©2013 The Infinite Actuary, LLC p. 2 B.4 Solutions

Page 85: The In nite Actuary Exam 1/P Online Seminar A.1 ... · The In nite Actuary Exam 1/P Online Seminar A.1 Fundamentals of Probability Solutions Last updated February 23, 2013 1. Let

Using the cdf method,

P[X ≤ x] = P[2Z ≤ x]

= P[Z ≤ x

2

]= FZ(x/2)

fX(x) =d

dxFZ

(x2

)= fZ

(x2

)· ddx

(x2

)=

1

2f(x

2

)6. Let X denote the loss amount, and Y the reimbursement.

G(115) = P[Y ≤ 115 | Y > 0] = P[X ≤ x | X > 20]

where x is the loss amount that results in a payment of 115. A loss of 20 results in a payment of0, a loss of 120 results in a payment of 100, and a loss of 120 + t results in a payment of 100 + 0.5t.That gives

100 + 0.5t = 115

t = 30

120 + t = 150 = x

G(115) = P[X ≤ 150 | X > 20] =P[20 < X ≤ 150]

P[X > 20]

G(115) =e−0.2 − e−1.5

e−0.2= 0.727

Note: for the last step, since X has an exponential distribution we could have used the memorylessproperty: P[X ≤ 150 | X > 20] = P[X ≤ 130] = 1− e−1.3

7. fX(x) = 5 e−5x

y = g(x) = 3 x2 + 5

y − 5 = 3 x2√y − 5

3= x = g−1(y)

We want the positive square root since x > 0.

d

dyg−1(y) =

1

2

(y − 5

3

)−1/2· 1

3

fY (y) = fX(g−1(y)

) ∣∣∣∣ ddy g−1(y)

∣∣∣∣= 5 e−5

√(y−5)/3

(1

6

)(y − 5

3

)−1/2

c©2013 The Infinite Actuary, LLC p. 3 B.4 Solutions

Page 86: The In nite Actuary Exam 1/P Online Seminar A.1 ... · The In nite Actuary Exam 1/P Online Seminar A.1 Fundamentals of Probability Solutions Last updated February 23, 2013 1. Let

=5

6exp

(−5[(y − 5)/3]1/2

)(y − 5

3

)−1/2

8. ln(X) is increasing, so Y = −2 ln(X) is decreasing. If y = −2 ln(x) then x = e−y/2, so

P[Y ≤ y] = P[−2 ln(X) ≤ y]

= P[X ≥ e−y/2] = 1− e−y/2

fY (y) =1

2e−y/2

Or, using the density formula,

y = g(x) = −2 ln(x)

x = g−1(y) = e−y/2

fX(x) =1

1− 0= 1 because X is uniform(0,1)

fY (y) = fX(g−1(y)

)·∣∣∣∣ ddy g−1(y)

∣∣∣∣= 1 ·

∣∣∣∣ ddy e−y/2∣∣∣∣ =

∣∣∣∣−1

2e−y/2

∣∣∣∣fY (y) =

1

2e−y/2

9. Y is an exponential with mean 1/θ = 2, θ = 1/2, so

P[Z > z] = P[√Y > z] = P[Y > z2]

= e−z2/(1/2) = e−2z

2

10. Since Y = X2, we potentially have X = ±√Y . One question, though, is whether or not we

have both roots. If we knew X > 0, then we would only have the positive root. But −2 < X < 3.That means that we have 2 roots when 0 < Y < 4, as they correspond to −2 < X < 2, but weonly have one root when 4 < Y < 9 (as that means that 2 < X < 3).

So we have two cases: one for fY (y) when 0 < y < 4, and one for 4 < y < 9. If 0 < y < 4 then

fY (y) = fX [√y]

∣∣∣∣ ddy√y∣∣∣∣+ fX [−√y]

∣∣∣∣ ddy (−√y)

∣∣∣∣=

1

5· 1

2√y

+1

5· 1

2√y

=1

5√y

while if 4 < y < 9 so g−1(y) =√y only we have

fY (y) = fX [√y]

∣∣∣∣ ddy√y∣∣∣∣

c©2013 The Infinite Actuary, LLC p. 4 B.4 Solutions

Page 87: The In nite Actuary Exam 1/P Online Seminar A.1 ... · The In nite Actuary Exam 1/P Online Seminar A.1 Fundamentals of Probability Solutions Last updated February 23, 2013 1. Let

=1

5· 1

2√y

=1

10√y

To combine both cases into one equation, we get

fY (y) =

1

5√y

0 < y < 4

1

10√y

4 < y < 9

With the cdf method, we still have 2 cases based on whether or not y < 4. If 0 < y < 4 then

P[Y ≤ y] = P[X2 ≤ y] = P[−√y ≤ X ≤ √y]

=

√y − (−√y)

5=

2√y

5

fY (y) =1

5√y

while if 4 < y < 9 then

P[Y ≤ y] = P[X2 ≤ y] = P[−2 ≤ X ≤ √y]

=

√y − (−2)

5=

√y + 2

5

fY (y) =1

10√y

and combining the two cases gives the same answer as before. For the cdf method, if you havetrouble following the equations, try plugging in specific values for y such as 3 or 7 and see whatyou get.

11. g(x) = y = e−2x

g−1(y) = x =− ln(y)

2

fY (y) = fX

(− ln(y)

2

)·∣∣∣∣ ddy − ln(y)

2

∣∣∣∣=

(eln(y) +

1

2e[ln(y)]/2

)· 1

2y

=y +

1

2

√y

2y=

1

2+

1

4√y

Using the cdf method

F (x) =

∫ x

0

e−2t +1

2e−tdt

=1

2− 1

2e−2x +

1

2− 1

2e−x = 1− 1

2e−2x − 1

2e−x

c©2013 The Infinite Actuary, LLC p. 5 B.4 Solutions

Page 88: The In nite Actuary Exam 1/P Online Seminar A.1 ... · The In nite Actuary Exam 1/P Online Seminar A.1 Fundamentals of Probability Solutions Last updated February 23, 2013 1. Let

fY (y) =d

dyP[Y ≤ y]

=d

dyP[e−2X ≤ y]

=d

dyP[−2X ≤ ln(y)]

=d

dyP[X ≥ − ln(y)

2]

=d

dy

[1

2e2 ln(y)/2 +

1

2eln(y)/2

]=

d

dy

[1

2y +

1

2y1/2

]=

1

2+

1

4√y

12. g(x) = y = 8− x3

g−1(y) = x = (8− y)1/3

fY (y) = fX((8− y)1/3

)·∣∣∣∣ ddy (8− y)1/3

∣∣∣∣=

8− y4· 1

3(8− y)−2/3 =

1

12(8− y)1/3

Using the cdf method

FX(x) =

∫ x

0

t3

4dt =

t4

16

P[Y ≤ y] = P[8−X3 ≤ y]

= P[X ≥ (8− y)1/3]

= 1−[(8− y)1/3

]416

= 1− (8− y)4/3

16

fY (y) =d

dyP[Y ≤ y]

=(4/3)(8− y)1/3

16

=1

12(8− y)1/3

13. P[Y ≤ 3] = P[X2 − 1 ≤ 3] = P[X2 ≤ 4]

= P[X ≤ 2] (we only want the positive roots since X > 0)

= 1− e−2

14. The point of this problem is that it isn’t a transformation problem! We don’t need to find the

c©2013 The Infinite Actuary, LLC p. 6 B.4 Solutions

Page 89: The In nite Actuary Exam 1/P Online Seminar A.1 ... · The In nite Actuary Exam 1/P Online Seminar A.1 Fundamentals of Probability Solutions Last updated February 23, 2013 1. Let

density of Y = 1/X, but instead just want to find

E[1/X] =

∫ 3

1

1

x· f(x) dx =

∫ 3

1

1 + 3x

30dx

=x+ 1.5x2

30

∣∣∣∣31

=7

15

15. Here, since X can be both positive and negative, Y = X2 isn’t a 1− 1 function. We have twochoices. Using the cdf method:

P[Y ≤ y] = P[X2 ≤ y] = P[−√y ≤ X ≤ √y]

=

∫ 0

−√y(1 + x)dx+

∫ √y0

(1− x)dx

= 2√y − y

fY (y) =1√y− 1

or we can use the change of variable formula and sum the two roots:

g(x) = y = x2

g−1(y) = x = ±√y

fY (y) = fX(√y)

∣∣∣∣ ddy√y∣∣∣∣+ fX(−√y)

∣∣∣∣ ddy (−√y)

∣∣∣∣= (1−√y)

1

2√y

+ (1−√y)1

2√y

=1√y− 1

16. fX(x) =d

dxFX(x) =

d

dx[1− P [X > x]]

= − d

dx

25

(x+ 5)2=

2 · 25

(x+ 5)3

y = (x+ 5)2 = g(x)√y − 5 = x = g−1(y)

fY (y) = fX(√y − 5)

∣∣∣∣ ddy (√y − 5)

∣∣∣∣=

1

2√y· 2 · 25

(√y − 5 + 5)3

=25

y2

17. Y = 1/X3 is a decreasing function of X, so

P[Y > 5] = P

[1

X3> 5

]

c©2013 The Infinite Actuary, LLC p. 7 B.4 Solutions

Page 90: The In nite Actuary Exam 1/P Online Seminar A.1 ... · The In nite Actuary Exam 1/P Online Seminar A.1 Fundamentals of Probability Solutions Last updated February 23, 2013 1. Let

= P

[X <

1

51/3

]= P[X < 0.5848]

=

0.5848∫0

2

(1 + x)3dx

=−1

(x+ 1)2

∣∣∣∣0.58480

= 0.602

18.P[Y ≤ y] = P

[ln

(1 +

X

θ

)≤ y

]= P

[1 +

X

θ≤ ey

]= P[X ≤ θey − θ]

= 1− θα

(θey − θ + θ)α= 1− 1

(ey)α

= 1− e−αy

19. Let Y = eT . We want

P[Y ≤ 3] = P[eT ≤ 3]

= P[T ≤ ln(3)]

= 1− e− ln(3)/1

= 1− 1

3=

2

3

20. Y = 1/(X − 1) is a decreasing function of X, so

P[Y ≤ y]P[1

X − 1≤ y]

= P

[X − 1 ≥ 1

y

]= P

[X ≥ 1 +

1

y

]=

2

1 + (1/y)=

2y

y + 1

fY (y) =2

y + 1− 2y

(y + 1)2

=2y + 2− 2y

(y + 1)2=

2

(y + 1)2

c©2013 The Infinite Actuary, LLC p. 8 B.4 Solutions

Page 91: The In nite Actuary Exam 1/P Online Seminar A.1 ... · The In nite Actuary Exam 1/P Online Seminar A.1 Fundamentals of Probability Solutions Last updated February 23, 2013 1. Let

The Infinite Actuary Exam 1/P Online SeminarB.6 Continuous Review SolutionsLast updated May 16, 2012

1. Let X and Y denote the error amounts.

X + Y

2∼ N

(0 + 0

2,(0.0056h)2 + (0.0044h)2

4

)X + Y

2∼ N

(0, (0.00356h)2

)P

[|X + Y |

2< 0.005h

]= P

[−0.005h <

X + Y

2< 0.005h

]= P

[−0.005h− 0

0.00356h<

(X + Y )/2− 0

0.00356h<

0.005h− 0

0.00356h

]= P[−1.404 < std normal < 1.404]

= Φ(1.404)− Φ(−1.404)

= Φ(1.404)− (1− Φ(1.404)) = 2Φ(1.404)− 1

≈ 2 · 0.92− 1 = 0.84

2. P[X > 16 | X > 8] =P[X > 16]

P[X > 8]

=0.005

∫ 20

16(20− x)dx

0.005∫ 20

8(20− x)dx

=

(20x− x2

2

)∣∣∣∣2016(

20x− x2

2

)∣∣∣∣208

=8

72=

1

9

3. f(x) =c

(10 + x)20 < x < 40

1 =

∫ 40

0

f(x) dx =

∫ 40

0

c

(10 + x)2dx

1 =−c

10 + x

∣∣∣∣400

=−c50

+c

10

50 = −c+ 5c = 4c, c = 12.5

P[X < 5] =

5∫0

12.5

(10 + x)2dx =

−12.5

10 + x

∣∣∣∣50

=−12.5

15+

12.5

10= 0.417

c©2012 The Infinite Actuary, LLC p. 1 B.6 Solutions

Page 92: The In nite Actuary Exam 1/P Online Seminar A.1 ... · The In nite Actuary Exam 1/P Online Seminar A.1 Fundamentals of Probability Solutions Last updated February 23, 2013 1. Let

4. If the payment is less than 0.5, then the loss is less than 0.5 + C, so

0.64 =P[X ≤ 0.5 + C]

=

∫ 0.5+C

0

2x dx = x2∣∣∣0.5+C0

0.64 =(0.5 + C)2

C = 0.3

5. The fastest approach is to say that

X ∼ U(0, 1000) d = deductible

Y = Payment after deductible =

{0 if X ≤ d

X − d if X > d

E[Y ] = E[Y | X ≤ d] · P[X ≤ d] + E[Y | X > d] · P[X > d]

if X ≤ d then Y = 0 while if X > d then Y ∼ Uniform(0, 1,000− d)

E[Y ] = 0 · P[X < 0] +1,000− d

2· P[X > d]

=1,000− d

2· 1,000− d

1,000=

(1,000− d)2

2,000

We want to find d so that E[Y ] =E[X]

4

E[X] =0 + 1,000

2= 500

E[Y ] =(1,000− d)2

2,000=

E[X]

4

(1,000− d)2

2,000=

500

4

(1,000− d)2 = 5002

d = 500

The calculus approach to E[Y ] is

P[Y ≤ y] = P[X ≤ y + d] =y + d

1,000

fY (y) =1

1,000for y > 0

E[Y ] = 0 · P[Y = 0] +

1,000−d∫0

y

1,000dy

c©2012 The Infinite Actuary, LLC p. 2 B.6 Solutions

Page 93: The In nite Actuary Exam 1/P Online Seminar A.1 ... · The In nite Actuary Exam 1/P Online Seminar A.1 Fundamentals of Probability Solutions Last updated February 23, 2013 1. Let

=y2

2· 1

1,000

∣∣∣∣1,000−d0

=(1,000− d)2

2,000

and then we proceed as before.

6. The calculus approach is that

M(t) =1

(1− 2500t)4

M ′(t) =−4 · (−2500)

(1− 2500t)5

E[X] =d

dtMX(t)

∣∣∣t=0

= 10,000

M ′′(t) =10,000 · (−5) · (−2500)

(1− 2500t)6

E[X2]=

d2

dt2MX(t)

∣∣∣t=0

= 1.25 · 108

Var[X] = E[X2]− (E[X])2 = 2.5 · 107

SD[X] =√

Var[X] = 5,000

Or we can recognize the MGF and see that X ∼ Gamma(α = 4, θ = 2500)

Var[X] = α · θ2

= 4 · (2,500)2

SD[X] = 2 · 2,500 = 5,000

7. Let t denote the 25th percentile and s the 75th. Then

0.25 =

t∫200

(2.5)(200)2.5

x3.5dx

0.25 =−2002.5

x2.5

∣∣∣∣t200

= 1− 2002.5

t2.5

t =

(0.75

2002.5

)−1/2.5= 224.4

0.75 =

s∫200

(2.5)(200)2.5

x3.5dx

0.75 =−2002.5

x2.5

∣∣∣∣5200

= 1− −2002.5

s2.5

s = 348.2

s− t = 123.8

8. Let X be the damage amount, and Y the payment. Then

X ∼ U(0, 1500)

c©2012 The Infinite Actuary, LLC p. 3 B.6 Solutions

Page 94: The In nite Actuary Exam 1/P Online Seminar A.1 ... · The In nite Actuary Exam 1/P Online Seminar A.1 Fundamentals of Probability Solutions Last updated February 23, 2013 1. Let

Y =

{0 X ≤ 250

X − 250 X > 250

E[Y ] = E[Y | X ≤ 250] · P[X ≤ 250] + E[Y | X > 250] · P[X > 250]

= 0 · 1

6+ E[U(0, 1250)] · 5

6

= 625 · 5

6= 520.833

E[Y 2] = E[Y 2 | X ≤ 250] · P[X ≤ 250] + E[Y 2 | X > 250] · P[X > 250]

= 02 · 1

6+ E[(U(0, 1250))2] · 5

6

=

(6252 +

12502

12

)· 5

6= 434,028

SD[Y ] =√

Var[Y ] =[434,028− 520.8332

]1/2= 403

9. Method 1: CDF method:

fY (y) =d

dyFY (y) =

d

dyP[Y ≤ y]

=d

dyP [T ≤ √y]

=d

dy

[1−

(2√y

)2]

=d

dy

[1− 4

y

]= −(−1)

4

y2=

4

y2

Method 2: Density approach:

fT (t) =d

dt

[1−

(2

t

)2]

=8

t3, t > 2

fY (y) = fT[g−1(y)

] ∣∣∣∣ ddy g−1(y)

∣∣∣∣y = g(t) = t2

t = g−1(y) =√y

fY (y) = fT [√y] ·∣∣∣∣ ddy √y

∣∣∣∣=

(8

(√y)3

)·∣∣∣∣12 y−1/2

∣∣∣∣ =4

y2

10. FV (v) = P[V ≤ v] = P[10,000eR ≤ v

]= P

[eR ≤ v

10, 000

]= P[R ≤ ln(v)− ln(10,000)]

c©2012 The Infinite Actuary, LLC p. 4 B.6 Solutions

Page 95: The In nite Actuary Exam 1/P Online Seminar A.1 ... · The In nite Actuary Exam 1/P Online Seminar A.1 Fundamentals of Probability Solutions Last updated February 23, 2013 1. Let

=ln(v)− ln(10,000)− 0.04

0.04= 25

[ln

(v

10, 000

)− 0.04

]

11. Sn = X1 + · · · +X1,250

Xi ∼ Poisson(2)

E[Xi] = 2, Var[Xi] = 2

E[Sn] = 2 · 1,250 = 2,500

Var[Sn] = 2 · 1,250 = 2,500

SD[Sn] = 50

P[2,450 < Sn < 2,600] = P

[2,450− 2,500

50<Sn − E[Sn]

SD[Sn]<

2,600− 2,500

50

]= P[−1 < Z < 2]

= Φ(2)− Φ(−1) = Φ(2)− (1− Φ(1))

= 0.9772− (1− 0.8413) = 0.8185

12. T1 = length time that 1st works

T2 = length of time 2nd works

Var[T1 + T2] = Var[T1] + Var[T2]

T1, T2 ∼ exp with mean 10

Var[T1] = Var[T2] = 102

Var[T1] + Var[T2] = 100 + 100 = 200

13. P[max{S, F, T} > 3] = 1− P[max{S, F, T} ≤ 3]

= 1− P[S ≤ 3, F ≤ 3, T ≤ 3]

= 1− P[S ≤ 3] · P[F ≤ 3] · P[T ≤ 3]

= 1−(1− e−3/1

) (1− e−3/1.5

) (1− e−3/2.4

)= 0.414

14. Let X be the loss amount and Y the payment.

Y =

{0 X ≤ 3

0.6(X − 3) 3 < X < 8

Since 0.6(X−3) is equally likely to be any value from 0.6(3−3) = 0 to 0.6(8−3) = 3 when X > 3,we have that (Y | X > 3) ∼ U(0, 3), so

E[Y ] = E[Y | X ≤ 3] · P[X ≤ 3] + E[Y | X > 3] · P[X > 3]

= 0 +3

2· 5

8=

15

16E[Y 2] = E[Y 2 | X ≤ 3] · P[X ≤ 3] + E[Y 2 | X > 3] · P[X > 3]

c©2012 The Infinite Actuary, LLC p. 5 B.6 Solutions

Page 96: The In nite Actuary Exam 1/P Online Seminar A.1 ... · The In nite Actuary Exam 1/P Online Seminar A.1 Fundamentals of Probability Solutions Last updated February 23, 2013 1. Let

= 0 +

(1.52 +

32

12

)· 5

8=

15

8

Var[Y ] =15

8−(

15

16

)2

= 0.996

Or, if you really want to use calculus,

E[Y ] =

∫ 3

0

0 · f(x) dx+

∫ 8

3

0.6(x− 3) · 1

8dx

=0.3x2 − 1.8x

8

∣∣∣∣83

=15

16

E[Y 2] =

∫ 3

0

02 · f(x) dx+

∫ 8

3

0.62(x− 3)2 · f(x) dx

=

∫ 8

3

0.36x2 − 2.16x+ 3.24

8dx

=0.12x3 − 1.08x2 + 3.24x

8

∣∣∣∣83

=15

8

Var[Y ] =15

8−(

15

16

)2

= 0.996

15. The 90th percentile of the loss X is when x/8 = 0.9 or x = 7.2. When the loss is 7.2, the

payment is 0.6 · (7.2− 3) = 2.52 and the unreimbursed loss is 4.68 .The reason why that works is that P[unreimbursed loss < 4.68] = P[X < 7.2] = 0.9.

16. The maximum of f(x) can occur at an endpoint (x = 0 or x = 3) or where f ′(x) = 0.

f(x) =4x

9− x2

9

f ′(x) =4

9− 2x

9

0 =4

9− 2x

9, x = 2

so the mode must be 0, 2, or 3. f(0) = 0, f(3) = 1/3 and f(2) = 4/9 > 1/3 so the mode is 2

17. F (x) =

∫ x

−∞f(t) dt =

∫ x

−∞et−2 dt

= et−2∣∣∣x−∞

= ex−2

0.75 = ex−2

x = 2 + ln(3/4)

18. F (x) =

∫ x

0

f(t) dt =

∫ x

0

2t

θ2dt =

x2

θ2

0.5 =[m(X)]2

θ2

c©2012 The Infinite Actuary, LLC p. 6 B.6 Solutions

Page 97: The In nite Actuary Exam 1/P Online Seminar A.1 ... · The In nite Actuary Exam 1/P Online Seminar A.1 Fundamentals of Probability Solutions Last updated February 23, 2013 1. Let

m(X) = θ√

0.5

E[X] =

∫ θ

0

x · 2x

θ2dx =

2x3

3θ2

∣∣∣∣θ0

=2

−0.12 = E[X]−m(X) =2

3θ − θ

√0.5

θ = 2.97

19. E[X] =

∫ θ

0

x · 3x2

θ3dx

15 =3x4

4θ3

∣∣∣θ0

=3

θ =4 · 15

3= 20

20.

E[X] =

∫ θ

0

x · 2x

θ2dx =

2x3

3θ2

∣∣∣∣θ0

=2

E[X2] =

∫ θ

0

x2 · 2x

θ2dx =

2

4θ2

Var[X] =

[2

4−(

2

3

)2]θ2 =

θ2

18

SD[X] = 2 =θ√18

θ = 6√

2

E[X] = 4√

2 = 5.7

21. There are 3 cases for the payment amount:

Payment =

0 X ≤ 5,000

X − 5,000 5,000 < X ≤ 25,000

20,000 X ≥ 25,000

E[Payment] = E[Payment | X ≤ 5,000] · P[X ≤ 5,000]

+ E[Payment | 5,000 < X ≤ 25,000] · P[5,000 < X ≤ 25,000]

+ E[Payment | 25,000 < X] · P[25,000 < X]

= 0 + E[Uniform(0, 20,000)] · 25− 5

50+ 20,000 · 50− 25

50

= 0 + 10,000 · 0.4 + 20,000 · 0.5 = 14,000

22. We don’t know that the distribution is symmetric (in fact, it probably isn’t since there cannotbe a negative number of crimes but could potentially be a large positive number). So

P[X ≥ 10] = P[X − µ ≥ 10− 2]

c©2012 The Infinite Actuary, LLC p. 7 B.6 Solutions

Page 98: The In nite Actuary Exam 1/P Online Seminar A.1 ... · The In nite Actuary Exam 1/P Online Seminar A.1 Fundamentals of Probability Solutions Last updated February 23, 2013 1. Let

= P[X − µ ≥ 8] = P[X − µ ≥ kσ]

where σ =√

4 = 2 and kσ = 8 so k = 4

P[X ≥ 10] = P[X − µ ≥ 4σ]

≤ P[|X − µ| ≥ 4σ] ≤ 1

42

23. Let S =∑Xi. Then

E[S] = 100E[Xi] = 100 · 1/2 = 50

Var[S] = 100Var[Xi] = 100 · (1/2)2 = 25

P[S > 57] = P

[S − E[S]

SD[S]>

57− 50

5

]= 1− Φ(1.4) = 1− 0.9192 = 0.08

24. Let X denote the loss amount.

E[Payment] = P[X ≤ 1] · E[Payment | X ≤ 1] + P[X > 1] · E[Payment | X > 1]

2 =1

θ· 0 +

θ − 1

θ· E[X − 1 | X > 1]

2 =θ − 1

θ· θ − 1

2θ2 − 6θ + 1 = 0

θ = 5.83

25. For a payment to be 3, the loss must be 3 + 2 = 5. Let X denote the loss amount. We aregiven

0.60 = P[X ≤ 5 | X > 2] =P[2 < X ≤ 5]

P[X > 2]

0.60 =F (5)− F (2)

1− F (2)=

(5

θ

)3

−(

2

θ

)3

1−(

2

θ

)3

0.6 =53 − 23

θ3 − 23

θ = 5.877

26. E[X] = E[X | T ≤ 2] · P[T ≤ 2] + E[X | T > 2] · P[T > 2]

= 2 · P[T ≤ 2] +2 + 5

2· 5− 2

5− 1=

25

8

c©2012 The Infinite Actuary, LLC p. 8 B.6 Solutions

Page 99: The In nite Actuary Exam 1/P Online Seminar A.1 ... · The In nite Actuary Exam 1/P Online Seminar A.1 Fundamentals of Probability Solutions Last updated February 23, 2013 1. Let

The Infinite Actuary Exam 1/P Online SeminarC.1 MultiVariate Distributions SolutionsLast updated February 25, 2013

1.

1 2

X

2

Y

The device fails either if X < 1 (blue area) or if X > 1 but Y < 1 (green area). The probabilityof this occurring is

2∫0

1∫0

f(x, y) dx dy +

1∫0

2∫1

f(x, y) dx dy =

2∫0

x2

16+xy

8

∣∣∣∣10

dy +

1∫0

x2

16+xy

8

∣∣∣∣21

dy

=

2∫0

1

16+y

8dy +

1∫0

3

16+y

8dy

=

(1

16+y2

16

)∣∣∣∣20

+

(3

16+y2

16

)∣∣∣∣10

=2

16+

4

16+

3

16+

1

16=

5

8

= 0.625

2. Let G denote the first claim time for a good driver, and B the first time for a bad driver.

P[G < 3, B < 2] = P[G < 3]P[B < 2]

=(1− e−3/6

) (1− e−2/3

)= 1− e−2/3 − e−1/2 + e−7/6

3. We want x > 20 and y > 20. The inner integral in the answer choices is dy, so we start aty = 20 (the smallest possible y). x < 50 − y implies that y < 50 − x, so the upper limits of ourinner integral are 50− x.

Combining y > 20 and x < 50− y gives x < 50− 20 = 30 so the outer limits are 20 < x < 30,and the answer is B

6

125,000

30∫20

50−x∫20

(50− x− y) dy dx

c©2013 The Infinite Actuary, LLC p. 1 C.1 Solutions

Page 100: The In nite Actuary Exam 1/P Online Seminar A.1 ... · The In nite Actuary Exam 1/P Online Seminar A.1 Fundamentals of Probability Solutions Last updated February 23, 2013 1. Let

4. Let D denote the time until the next deluxe claim, and B the time until the next basic claim.

We are interested in the probability of being in the shaded region: B

D

P[D < B] =

∞∫0

∞∫d

f(b, d) db dd

=

∞∫0

∞∫d

1

3e−d/3 1

2e−b/2 db dd

=

∞∫0

1

3e−d/3P[B > d] dd =

∞∫0

1

3e−d/3e−d/2 dd

=

∞∫0

1

3e−d/3e−d/2 dd =

∞∫0

1

3e−5d/6 dd

=1

3·(−6

5

)e−5d/6

∣∣∣∣∞0

=2

5e−0 =

2

5

= 0.400

5. The company accepts a bid if it is in either triangle marked with a star. They consider the bidsfurther if it is in the shaded region

2000 2200

2000

2200

2020

F

2180

2020

F

2180

180

180

Since we have a uniform distribution, the probability that we want is

1− 2 · area of 1 triangle

total area= 1− 2 · 1802/2

2002= 0.19

c©2013 The Infinite Actuary, LLC p. 2 C.1 Solutions

Page 101: The In nite Actuary Exam 1/P Online Seminar A.1 ... · The In nite Actuary Exam 1/P Online Seminar A.1 Fundamentals of Probability Solutions Last updated February 23, 2013 1. Let

6. Conditioning on X = 1/3 means that (X, Y ) lie on the following line:

X

Y

1/3

(1/3, 2/3)

P[Y < X | X = 1/3] =

1/3∫0

fY |X (y | X = 1/3)

fY |X (y | X = 1/3) =fX,Y

(13, y)

2/3∫0

fX,Y

(13, y)dy

=24 · 1

3· y

2/3∫0

243y dy

=8 y

4 y2∣∣2/3

0

=8 y

4 · 22

32

=9

2y

So P

[Y < X

∣∣∣x =1

3

]=

1/3∫0

9

2y dy =

9

4y2∣∣∣1/3

0=

1

4

7. Since x is a proportion, 0 < x < 1. Y is a subset of X, so 0 < y < x, and our picture is

0.1 1X

Y

We want P[Y < 0.05 | X = 0.10] and are given f(x, y) = 2(x+ y)

fY |X(y | X = 0.1) =f(0.1, y)

0.1∫0

f(0.1, y) dy

=2(0.1 + y)

0.1∫0

2(0.1 + y) dy

=0.2 + 2y

2 · 0.12 + 0.12=

0.2 + 2y

3(0.1)2

P[Y < 0.05 | X = 0.1] =

0.05∫0

fY |X(y | X = 0.1) dy

=1

(3)(0.1)2

0.05∫0

(0.2 + 2y) dy

=1

(3)(0.1)2

[(0.2)(0.05) + 0.052

]= 0.417

c©2013 The Infinite Actuary, LLC p. 3 C.1 Solutions

Page 102: The In nite Actuary Exam 1/P Online Seminar A.1 ... · The In nite Actuary Exam 1/P Online Seminar A.1 Fundamentals of Probability Solutions Last updated February 23, 2013 1. Let

8. We want P[X < 0.2] and have 0 < x < 1, 0 < y < 1− x, so our picture is

0.2 1X

Y1

x+ y = 1

y = 1− x

0.2∫0

1−x∫0

f(x, y) dy dx =

0.2∫0

1−x∫0

6[1− (x+ y)] dy dx

=

0.2∫0

6

(y − xy − y2

2

)∣∣∣∣1−x0

dx

=

0.2∫0

6

[1− x− x+ x2 − (1− x)2

2

]dx

=

0.2∫0

6− 12x+ 6x2 − 3 (1− 2x+ x2) dx

=

0.2∫0

3− 6x+ 3x2 dx =

0.2∫0

3(x− 1)2 dx

= (x− 1)3∣∣0.20

= (−0.8)3 − (−1)3 = 0.488

9. We want x2 < y < x, so our picture isx

1

y

1

y = x2

x =√y

y = x

For any specific y, the rangeof x is from y to

√y so

fY (y) =

√y∫

y

f(x, y) dx =

√y∫

y

15 y dx = 15 y (√y − y) = 15y3/2(1− y1/2)

10. fX,Y (x, y) = fX (x) · fY |X (y | X = x) = 1 · 1 = 1. Since this is constant, (X, Y ) are uniformon the set of their possible values (which has total area 1/f(x, y) = 1).

c©2013 The Infinite Actuary, LLC p. 4 C.1 Solutions

Page 103: The In nite Actuary Exam 1/P Online Seminar A.1 ... · The In nite Actuary Exam 1/P Online Seminar A.1 Fundamentals of Probability Solutions Last updated February 23, 2013 1. Let

x

y

1

1

2

0.5 (.5, .5)

We want P[Y > 0.5], which is the probability of being in the shaded area. That has a compli-

cated shape, but using the complement, P[Y > 0.5] = 1− area of triangle

1= 1− 0.5 · 0.5 · 0.5

1=

7

8

11.

X2

Y(

3

2, 3

)3

y = 2x

y = x

We want P[Y > 3] so we want to integrate f(x, y) over the shaded region. So first we need tofind the joint density.

fX (x) =3

8x2

fY |X (y | X = x) =1

x, x < y < 2x

fX,Y (x, y) = fX (x)fY |X (y | X = x)

=3

8x2 · 1

x=

3

8x

P[Y > 3] =

2∫1.5

2x∫3

3

8x dy dx =

2∫1.5

3

8x (2x− 3) dx

=

2∫1.5

(3

4x2 − 9

8x

)dx =

x3

4− 9

16x2

∣∣∣∣21.5

= 0.172

12. P[4 < S < 8] = P[N = 1, 4 < S < 8] + P[N > 1, 4 < S < 8]

= P[4 < S < 8 | N = 1] · P[N = 1] + P[4 < S < 8 | N > 1] · P[N > 1]

=1

3

[e−4/5 − e−8/5

]+

1

6

[e−4/8 − e−8/8

]= 0.1223

c©2013 The Infinite Actuary, LLC p. 5 C.1 Solutions

Page 104: The In nite Actuary Exam 1/P Online Seminar A.1 ... · The In nite Actuary Exam 1/P Online Seminar A.1 Fundamentals of Probability Solutions Last updated February 23, 2013 1. Let

13.

1X

Y

We are given that Y = y, so we are given that we are somewhere on the horizontal line drawn.

f(x, y) = fX(x) · fY |X(y | X = x) = 2x · 1

x= 2

fX|Y (x | Y = y) =f(x, y)∫ 1

yf(x, y)dx

=2∫ 1

y2dx

=1

1− y

14. The maximum only comes into play is if both employee’s have a loss. That means that

P[Total Loss > 8,000] = P[both employees have a loss]× P[loss > 8,000 | both have a loss]

= 0.4× P[Total Loss > 8,000 | both have a loss]

and to find P[Total Loss > 8,000 | both have a loss] we draw a picture and look at areas since thejoint distribution is uniform.

1 2 5older

1

5

younger (3, 5)

(5, 3)

P[total loss > 8,000 | both have a loss, older loss > 2,000] =12· 2 · 23 · 4

=1

6

Final answer = 0.4 · 1

6=

1

15

15.

1 2X

1

3

Y

c©2013 The Infinite Actuary, LLC p. 6 C.1 Solutions

Page 105: The In nite Actuary Exam 1/P Online Seminar A.1 ... · The In nite Actuary Exam 1/P Online Seminar A.1 Fundamentals of Probability Solutions Last updated February 23, 2013 1. Let

fX|Y (x | Y = 3) =fX,Y (x, 3)

∞∫1

fX,Y (x, 3) dx

=

1

9x2

∞∫1

1

9x2dx

=

1

x2

−1

x

∣∣∣∣∞1

=1

x2

P[X > 2 | Y = 3] =

∞∫2

dx

x2=−1

x2

∣∣∣∞2

=1

2

16. Now that we have added the constraint that x < y our picture changes:

1

1

3 (3, 3)

fX|Y (x | Y = 3) =

2

x2 · 93∫1

2

x2 · 9dx

=

1

x2

−1

x

∣∣∣∣31

=

1

x2

1− 13

=3

2· 1

x2

P[X > 2 | Y = 3] =

3∫2

3

2· 1

x2dx

=3

2

[1

2− 1

3

]=

1

4

17.

x 1X

1

Y

y = x2

√y = x

0

c©2013 The Infinite Actuary, LLC p. 7 C.1 Solutions

Page 106: The In nite Actuary Exam 1/P Online Seminar A.1 ... · The In nite Actuary Exam 1/P Online Seminar A.1 Fundamentals of Probability Solutions Last updated February 23, 2013 1. Let

f(x, y) = fY (y) · fX|Y (x | Y = y)

=1

1− 0· 1√y − 0

=1√y

fX (x) =

1∫x2

1√ydy = 2

√y∣∣∣1x2

= 2− 2x = 2 (1− x)

18. We are given that X and Y are uniformly chosen from a disk with radius 2/√π. The area of

that disk is πr2 = π · 4

π= 4.

X

x2 + y2 = r2

y = ±√r2 − x2

= ±√

4

π− x2

Y(x,

√4

π− x2

)

(x,−

√4

π− x2

)

x 2√π

f (x, y) =1

area=

1

4

fX (x) =

∫1

4dy =

1

4· 2√

4

π− x2 =

√1

π− x2

4

19. P[(X, Y ) = (x, y)] =16− 4x− 4 y + xy

36

=(4− x)

6· (4− y)

6= P[X = x] · P[Y = y]

Since the probabilities factor and the support is a rectangle, X and Y are independent. The factorsare both (4− variable)/6 so the marginal distributions match as well. The answer is therefore D

20. We want P[X < 1, Y < 1]/P[Y < 1], which is the probability of being in the dark shaded

c©2013 The Infinite Actuary, LLC p. 8 C.1 Solutions

Page 107: The In nite Actuary Exam 1/P Online Seminar A.1 ... · The In nite Actuary Exam 1/P Online Seminar A.1 Fundamentals of Probability Solutions Last updated February 23, 2013 1. Let

region divided by the probability of being in either of the shaded regions.

X2

Y

1

2

x = 2− y

y = 2− x

P[X < 1 | Y < 1] =P[X < 1, Y < 1]

P[Y < 1]

=

1∫0

1∫0

34(2− x− y) dx dy

1∫0

2−y∫0

34(2− x− y) dx dy

1∫0

1∫0

(2− x− y) dx dy =

1∫0

2− 1

2− y dy

=3

2− 1

2= 1

1∫0

2−y∫0

2− x− y dx dy =

1∫0

(2− y)x− x2

2

∣∣∣2−y0

dy

=

1∫0

(2− y)2 − (2− y)2

2dy

=

1∫0

1

2[2− y]2 dy =

1

2· −1

3(2− y)3

∣∣∣10

=−1

6· (+1)3 − −1

6· (23) =

7

6

Answer:1

7/6=

6

7

c©2013 The Infinite Actuary, LLC p. 9 C.1 Solutions

Page 108: The In nite Actuary Exam 1/P Online Seminar A.1 ... · The In nite Actuary Exam 1/P Online Seminar A.1 Fundamentals of Probability Solutions Last updated February 23, 2013 1. Let

21. We are given that we are on the green line segment, and want X < 1/2.

1

1

4

1 (3

4,1

4

)

fX|Y

(x | Y =

1

4

)=

f(x, 1

4

)3/4∫0

f(x, 1

4

)dx

=24x · 1

4

3/4∫0

24x · 14dx

=x

3/4∫0

x dx

=x

x2

x

∣∣3/4

0

fX|Y

(x | Y =

1

4

)=

32

9x

P

[X <

1

2

∣∣∣Y =1

4

]=

1/2∫0

32

9x dx

=16

9x2∣∣∣1/2

0=

4

9

22. To find k, we will use the fact that the total probability is 1:

1 =

∫ ∞1

∫ ∞1

kx−3e−y/3dydx

= k

∫ ∞1

x−3(−3e−y/3

)∣∣∞1dx

= 3ke−1/3

∫ ∞1

x−3 dx = 3ke−1/3 · 1

2

k =2

3e1/3

23. Before doing anything, the marginal of Y can only involve y, so answers D and E must be

c©2013 The Infinite Actuary, LLC p. 10 C.1 Solutions

Page 109: The In nite Actuary Exam 1/P Online Seminar A.1 ... · The In nite Actuary Exam 1/P Online Seminar A.1 Fundamentals of Probability Solutions Last updated February 23, 2013 1. Let

wrong.

x

y

1 2

1

2

Since x ranges from 1 to y, to find the marginal of Y we integrate f(x, y) from 1 to y, i.e.,

fY (y) =

∫ y

1

f(x, y) dx =

∫ y

1

12

25(x+ y2)dx

=

[12

25· x

2

2+

12

25· xy2

]∣∣∣∣y1

=6

25

[(y2 − 1) + 2y2(y − 1)

]=

6

25

[2y3 − y2 − 1

]24.

x

y

1 2

1

From our picture, we see that the top boundary is somewhat complicated (it is y = x if x < 1 andy = 1 if 1 < x < 2) so we would rather have our inner integral be dx to avoid having to split intomultiple cases.

P

[X

2≤ Y ≤ X

]=

∫ 1

0

∫ 2y

y

f(x, y)dx dy

=

∫ 1

0

∫ 2y

y

xy dx dy

=

∫ 1

0

x2y

2

∣∣∣∣2yy

dy

=

∫ 1

0

3y3

2dx

=3

4 · 2y4

∣∣∣∣10

=3

8

c©2013 The Infinite Actuary, LLC p. 11 C.1 Solutions

Page 110: The In nite Actuary Exam 1/P Online Seminar A.1 ... · The In nite Actuary Exam 1/P Online Seminar A.1 Fundamentals of Probability Solutions Last updated February 23, 2013 1. Let

25.

x

y

1 2

1

2

P[X > 1] =

∫ 2

1

∫ 2−x

0

f(x, y) dy dx

=

∫ 2

1

(2− x) · 3x

4dx

=

∫ 2

1

6x− 3x2

4dx

=3x2 − x3

4

∣∣∣∣21

= 1− 1

2=

1

2

c©2013 The Infinite Actuary, LLC p. 12 C.1 Solutions

Page 111: The In nite Actuary Exam 1/P Online Seminar A.1 ... · The In nite Actuary Exam 1/P Online Seminar A.1 Fundamentals of Probability Solutions Last updated February 23, 2013 1. Let

The Infinite Actuary Exam 1/P Online SeminarC.2 MultiVariate Moments SolutionsLast updated April 2, 2012

1.

t1

t2t2 = t1

L (L,L)

f(t1, t2) =1

area=

1

L2/2=

2

L2

E[T 21 + T 2

2 ] =

L∫0

t2∫0

(t21 + t22) ·2

L2dt1 dt2

=

L∫0

(t313

+ t22t1

)∣∣∣∣t20

· 2

L2dt2

=

∫ L

0

8

3L2t32 dt2 =

2 t423L2

∣∣∣∣L0

=2

3· L

4

L2=

2L2

3

2.P[Y = 1] = P[X1 = X2 = X3 = 1] =

(2

3

)3

P[Y = 0] = 1− P[Y = 1] = 1−(

2

3

)3

M(t) = E[et y]

= et·0P[Y = 0] + et·1P[Y = 1]

= 1 · 19

27+ et

8

27

Note that we worked with P[Y = 1] because that was easier to find than P[Y = 0]. In particular,P[Y = 0] is not equal to (P[X = 0])3 since that is P[X1 = X2 = X3 = 0] but Y can also be 0 ifX1 = 1 and X2 = X3 = 0 or if X1 = 0 and X2 = X3 = 1 or ...

3. The new total benefit is (X + 100) + 1.1Y , and

Var(X + 100 + 1.1Y ) = Var(X + 1.1Y )

= Var(X) + 2 · 1.1Cov(X, Y ) + 1.12Var(Y )

c©2012 The Infinite Actuary, LLC p. 1 C.2 Solutions

Page 112: The In nite Actuary Exam 1/P Online Seminar A.1 ... · The In nite Actuary Exam 1/P Online Seminar A.1 Fundamentals of Probability Solutions Last updated February 23, 2013 1. Let

= 5,000 + 2.2Cov(X, Y ) + 12,100

So we just need to know Cov(X, Y ). But we know Var(X + Y ), so we can use

Var(X + Y ) = Var(X) + 2Cov(X, Y ) + Var(Y )

17,000 = 5,000 + 2Cov(X, Y ) + 10,000

Cov(X, Y ) = 1,000

Var(X + 100 + 1.1Y ) = 5,000 + 2.2Cov(X, Y ) + 12,100

= 5,000 + 2,200 + 12,100 = 19,300

4. Since we want the variance of X, one first step is to find the (marginal) distribution of X.

x P[X = x] x2

0 1/6 0

11

12+

1

6=

1

41

27

124

E(X) = 0 · 1

6+ 1 · 1

4+ 2 · 7

12=

17

12

E(X2) = 0 · 1

6+ 1 · 1

4+ 4 · 7

12=

31

12

Var(X) = E(X2)− (EX)2 = 0.576

5. Var(Z) = Var[3X − Y − 5]

= Var[3X − Y ] = Var[3 ·X + (−1) · Y ]

= Var(3 ·X) + 2Cov(3X,−Y ) + Var(−1 · Y )

= 32 · Var[X] + 2 · 3 · (−1)Cov(X, Y ) + (−1)2 Var[Y ]

= 9 · 1 +−6 · 0 + 1 · 2 = 11

6. Finding the covariance from scratch is tedious as it often requires 3 (in this case 4) doubleintegrals, so we often want to look for a shortcut. Here, the shortcut is to notice that the domainis rectangular, and that f(x, y) factors as a function of X (namely kx) times a function of Y

(namely 1), so X and Y are independent and thus Cov (X, Y ) = 0 The long way to do theproblem is as follows: First, we need to find k.

1 =

1∫0

1∫0

k x dx dy =

1∫0

k

2· (12 − 02) dy

=k

2, so k = 2

c©2012 The Infinite Actuary, LLC p. 2 C.2 Solutions

Page 113: The In nite Actuary Exam 1/P Online Seminar A.1 ... · The In nite Actuary Exam 1/P Online Seminar A.1 Fundamentals of Probability Solutions Last updated February 23, 2013 1. Let

E(X) =

1∫0

1∫0

2x · xdx dy =

1∫0

2

3dy =

2

3

E(Y ) =

1∫0

1∫0

2x y dx dy =

1∫0

y dy

=y2

2

∣∣∣∣10

=1

2

E(XY ) =

1∫0

1∫0

2x2y dx dy

=

1∫0

2

3y dy =

1

3y2∣∣∣∣10

=1

3

Cov (X, Y ) = E(XY )− E(X)E(Y )

=1

3− 2

3· 1

2= 0

7.

x1

y

y = 2x

y = x

E(X) =

1∫0

2x∫x

8

3x2 y dy dx =

1∫0

4

3x2 y2

∣∣∣∣2xx

dx

=

1∫0

4x4 dx =4

5x5∣∣∣∣10

=4

5

E[Y ] =

1∫0

2x∫x

8

3x y2 dy dx =

1∫0

8

9x y3

∣∣∣∣2xx

dx

=

1∫0

8 · 79

x4 dx =8 · 79 · 5

∣∣∣∣10

=56

45

c©2012 The Infinite Actuary, LLC p. 3 C.2 Solutions

Page 114: The In nite Actuary Exam 1/P Online Seminar A.1 ... · The In nite Actuary Exam 1/P Online Seminar A.1 Fundamentals of Probability Solutions Last updated February 23, 2013 1. Let

E(XY ) =

1∫0

2x∫x

8

3x2 y2 dy dx =

1∫0

8

9x2 y3

∣∣∣∣2xx

dx

=

1∫0

8 · 79

x5 =8 · 79 · 6

x6∣∣∣∣10

=56

54=

28

27

Cov(X, Y ) =28

27− 4

5· 56

45= 0.04

8. The fact that the random variable Y is described in terms of the variable X (namely as beinguniform on (0, X) if we know X) suggests that we probably want to use double expectation to findany moments involving Y .

E(X) =0 + 12

2= 6

E[Y |X = x] =0 + x

2=x

2

E[Y ] = E[E[Y |X]

]= E

(X

2

)=

1

2E(X) = 3

E[XY |X] = X E[Y |X] = X · X2

=X2

2

E[XY ] = E[E[XY |X]

]= E

(X2

2

)=

1

2· E[X2]

And to find E[X2], we use the fact that we know the mean and variance of a uniform:

Var(X) =(12− 0)2

12= 12

E(X) =0 + 12

2= 6

E[X2] = Var(X) + (EX)2 = 12 + 36 = 48

E[XY ] =1

2· 48 = 24

Cov(X, Y ) = E(XY )− (E[X])(E[Y ])

= 24− 6 · 3 = 6

If we didn’t want to use double expectation, then we could have set

fX(x) =1

120 < x < 12

fY |X(x) =1

x0 < y < x

fX,Y (x, y) = fX(x)fY |X(y) =1

12x0 < y < x < 12

c©2012 The Infinite Actuary, LLC p. 4 C.2 Solutions

Page 115: The In nite Actuary Exam 1/P Online Seminar A.1 ... · The In nite Actuary Exam 1/P Online Seminar A.1 Fundamentals of Probability Solutions Last updated February 23, 2013 1. Let

which gives us a domain of

12

and then we do 3 double integrals (one for each of E(X),E(Y ) and E(XY ).

9. We are given that C1 = X + Y and C2 = X + 1.2Y and want their covariance. First, let’sfigure out what we know about X and Y .

Var(X) = E(X2)− (E[X])2 = 27.4− 25 = 2.4

Var(Y ) = E(Y 2)− (E[Y ])2 = 51.4− 49 = 2.4

Var(X + Y ) = Var[X] + Var[Y ] + 2Cov(X, Y )

8 = 4.8 + 2 Cov (X, Y )

Cov(X, Y ) =3.2

2= 1.6

Cov(C1, C2) = Cov(X + Y,X + 1.2Y )

= Cov(X,X) + Cov(X, 1.2Y ) + Cov(Y,X) + Cov(Y, 1.2Y )

= Var[X] + [1.2 + 1]Cov(X, Y ) + 1.2Var[Y ]

= 2.4 + 2.2 · 1.6 + 1.2 · 2.4 = 8.8

10. The total premium paid is 500 + 500 = 1,000. Since we are conditioning on the husbandsurviving, the possible total claims are either 0 (if the wife survives) or 10,000 (if the wife dies).The unconditional probabilities that we are given are:

H dies

H lives0.96 0.01

0.025 0.005

W lives W dies

With no conditioning

P[W lives|H lives] =0.96

0.97, P[W dies|H lives] =

0.01

0.97E[total premiums− total claims|H lives] = E[1,000− total claims|H lives]

= 1,000P[W lives|H lives]− 9,000P[W dies|H lives]

c©2012 The Infinite Actuary, LLC p. 5 C.2 Solutions

Page 116: The In nite Actuary Exam 1/P Online Seminar A.1 ... · The In nite Actuary Exam 1/P Online Seminar A.1 Fundamentals of Probability Solutions Last updated February 23, 2013 1. Let

= 1,000 · 96

97− 9,000 · 1

97= 897

11. P[X = 1] = 0.05 + 0.125 = 0.175

P[Y = 0 | X = 1] =0.05

0.175, P[Y = 1 | X = 1] =

0.125

0.175

Given X = 1, there are two possible values for Y , namely 0 and 1, so we can use our usual formulafor the variance of a Bernoulli random variable

Var(Y | X = 1) = P[Y = 1 | X = 1] · P[Y = 0 | X = 1]

=0.125

0.175· 0.05

0.175= 0.204

12. The formula for the joint density of X and Y doesn’t depend on y (so long as it is a possiblevalue), so given that X = x, Y is uniform on x < y < x+ 1. Therefore

Var(Y | X = x) =(x+ 1− x)2

12=

1

12

13. Let X denote the number of tornadoes in county P , and Y the number in county Q. We aregiven that X = 0, and P[X = 0] = 0.12 + 0.06 + 0.05 + 0.02 = 0.25 so 1/P[X = 0] = 4. Thatmeans that the conditional distribution of Y is given by

y P[Y = y|X = 0]0 0.12 · 4 = .481 0.06 · 4 = .242 0.05 · 4 = .23 0.02 · 4 = .08

E[Y | X = 0] = 0 · 0.48 + 1 · 0.24 + 2 · 0.2 + 3 · 0.08

E[Y 2 | X = 0] = 02 · 0.48 + 12 · 0.24 + 22 · 0.2 + 32 · 0.08

Var[Y |X = 0] = E[Y 2|X = 0]−(E[Y |X = 0]

)2= 1.76− .7744 ≈ 0.99

14.E(X) =

∫∫x f(x, y) dx dy =

1∫0

10∫2

10x− x2 y2

64dx dy

=

1∫0

(5x2

64− x3 y2

64 · 3

)∣∣∣∣102

dy

=

1∫0

(15

2− 31

6y2)dy =

15

2· 1− 31

6· 1

3= 5.8

15.c©2012 The Infinite Actuary, LLC p. 6 C.2 Solutions

Page 117: The In nite Actuary Exam 1/P Online Seminar A.1 ... · The In nite Actuary Exam 1/P Online Seminar A.1 Fundamentals of Probability Solutions Last updated February 23, 2013 1. Let

E(Y ) =

∞∫0

∞∫x

y f(x, y) dy dx

=

∞∫0

y∫0

6ye−xe−2y dx dy

=

∞∫0

−6 y e−2ye−x∣∣∣yx=0

dy

= 6

∞∫0

y e−2y − 6

∞∫0

y e−3y dy

=6

4− 6

9=

5

6

X

YY = X

To evaluate the last two integrals, we could do integration by parts (or equivalently tabularintegration). Alternatively, if Y is an exponential random variable with mean 1/λ then∫ ∞

0

λe−λydy = E[Y ] =1

λ∫ ∞0

e−λydy =1

λ

∫ ∞0

λe−λydy

=1

λE[Y ] =

1

λ2

and we are using that with λ = 2 and λ = 3 for our two integrals. A final way of looking at it isto use the Gamma trick from the calculus review:∫ ∞

0

xa · e−bxdx =a!

ba+1

Here, we have a = 1 and b = 2 or 3 respectively.

16. Again, since we are looking for the covariance, our hope is that we can find a shortcut. (Sadly,that won’t always be possible on the exam.)

1

1

So our domain is a rectangle, and

f(x, y) =16

x5 y5=

4

x5· 4

y5

c©2012 The Infinite Actuary, LLC p. 7 C.2 Solutions

Page 118: The In nite Actuary Exam 1/P Online Seminar A.1 ... · The In nite Actuary Exam 1/P Online Seminar A.1 Fundamentals of Probability Solutions Last updated February 23, 2013 1. Let

so the density factors. Therefore X and Y are independent, and thus Cov(X, Y ) = 0 .

17. There are two approaches to this. The shorter approach is to notice that our original region isa rectangle, and the joint density factors as f(x, y) = ce−xe−2y, so X and Y are independent. TheY factor of the joint density is that of an exponential random variable with mean 1/2, so that isour marginal distribution of Y . Combining independence with the memoryless property,

Var(Y | X > 3, Y > 3) = Var(Y | Y > 3)

= Var(Y − 3 | Y > 3)

= Var(Y )

=

(1

2

)2

= 0.25

Alternatively, we could find the conditional joint density:

f(x, y | X > 3, Y > 3) =f(x, y)

P[X > 3, Y > 3]x > 3, y > 3

=2e−x−2y

e−3 · e−6= e−(x−3) · 2e−2(y−3)

from which we can either again conclude that the conditional marginal density of Y is a shiftedexponential, or we could do a couple of double integrals to find E[Y ] or E[Y 2].

18. We are given that N1 = 2, and

p(n1, n2 | N1 = 2) =p(2, n2)

P[N1 = 2]

=

3

4

(1

4

)2−1

e−2(1− e−2)n2−1

P[N1 = 2]

= c · (1− e−2)n2

for n2 = 1, 2, . . . . That is the distribution of a geometric random variable starting at 1. The ratioin a geometric is 1 − p = 1 − e−2 so p = e−2. The expected value of a geometric starting at 1 is

1/p, so E[N2 | N1 = 2] = 1/p = e2

19. We can see that we want to use double expectation or the law of total variation since if weknew λ, the problem would be easy. So we can either work with the raw moments:

E[N ] = E[E[N | λ]] = E[λ] =3

2E[N2] = E[E[N2 | λ]] = E[λ2 + λ]

=

[32

12+

(3

2

)2]

+3

2

Var[N ] =3

4+

3

2= 2.25

c©2012 The Infinite Actuary, LLC p. 8 C.2 Solutions

Page 119: The In nite Actuary Exam 1/P Online Seminar A.1 ... · The In nite Actuary Exam 1/P Online Seminar A.1 Fundamentals of Probability Solutions Last updated February 23, 2013 1. Let

or using the law of total variation

Var[N ] = E[Var[N | λ]] + Var[E[N | λ]]

=E[λ] + Var[λ]

=3

2+

32

12= 2.25

20. There are a few ideas going on here. First, the MGF of X + Y is the product of the MGFs ofX and Y by independence, so we want to factor M(t). Second, the MGF is a sum of terms of theform cie

tai , with the ci summing to 1, suggesting that we have a discrete distribution.

M(t) = 0.09e−2t + 0.24e−t + 0.34 + 0.24et + 0.09e2t

= (0.3e−t + 0.4 + 0.3et)2

P[X = −1] = 0.3 P[X = 0] = 0.4 P[X = 1] = 0.3

P[X ≤ 0] = 0.3 + 0.4 = 0.7

21.

fY |X(y | X = 0.75) =

{c · 1.50 0 < y < 0.5

c · 0.75 0.5 < y < 1

1 =

∫ 1

0

fY |X(y | X = 0.75)dy =

∫ 0.5

0

c · 1.50 dy +

∫ 1

0.5

c · 0.75 dy

1 = c · 1.5

2+ c · 0.75

2, c =

2

2.25

fY |X(y | X = 0.75) =

{43

0 < y < 0.523

0.5 < y < 1

E(Y | X = 0.75) =

∫ 0.5

0

4y

3dy +

∫ 1

0.5

2y

3dy

=2 · 0.52

3+

1− 0.52

3=

5

12

E[Y 2 | X = 0.75] =

∫ 0.5

0

4y2

3dy +

∫ 1

0.5

2y2

3dy

=4 · 0.53

9+

2− 2 · 0.53

9=

1

4

Var[Y | X = 0.75] =11

144= 0.076

22. Let N be the number of hurricanes that hit. If we knew N , then we would have the sum of Nindependent exponentials and could find the variance. So we want to use either double expectationor the law of total variation. The law of total variation is easier since we can find the variance ofa sum more easily than the second moment.

Let S be the total loss amount. Then

E[S | N ] = 1,000 ·N Var[S | N ] = 1,0002 ·N

c©2012 The Infinite Actuary, LLC p. 9 C.2 Solutions

Page 120: The In nite Actuary Exam 1/P Online Seminar A.1 ... · The In nite Actuary Exam 1/P Online Seminar A.1 Fundamentals of Probability Solutions Last updated February 23, 2013 1. Let

Var[S] = E[Var(N | S)] + Var[E(N | S)]

= E[1,000,000N ] + Var[1,000N ]

= 1,000,000 · 4 + 1,0002 · 4= 8,000,000

23. Let N be the number of accidents and S the total unreimbursed amount. The reimbursementfor an accident is 70% of the loss amount, so the unreimbursed part is 30%. You could scale theexponential (e.g., say that the unreimbursed loss is exponential with mean 0.3 · 0.8 = 0.24) butthat approach wouldn’t necessarily work with some other distributions.

A more general approach is to let X be a loss and Y the unreimbursed part (so Y = 0.3X).Then E[Y ] = E[0.3X] = 0.3E[X] = 0.24 and Var(Y ) = 0.32Var(X) = 0.242, so

Var(S) = E[Var(S | N)] + Var[E(S | N)]

= E[0.242 ·N ] + Var[0.24 ·N ]

= 0.242(E[N ] + Var[N ])

= 0.242(0.25 · 3 + 0.25 · 0.75 · 3)

= 0.0756

24. fY |X

(y∣∣∣X =

1

3

)=

f(13, y)∫ 1

0f(13, y)dy

=13

+ y∫ 1

013

+ y dy=

13

+ y13

+ 12

=6

5

(1

3+ y

)E

[Y∣∣∣X =

1

3

]=

∫ 1

0

y · 6

5

(1

3+ y

)dy

=

∫ 1

0

(2

5y +

6

5y2)dy

=

(y2

5+

2 y3

5

)∣∣∣∣10

=3

5

25. E[k (X2 − Y 2) + Y 2

]= k E(X2)− k E(Y 2) + E(Y 2)

σ2 = k [12 + σ2]− k [22 + σ2] + (22 + σ2)

σ2 = k + kσ2 − 4k − kσ2 + 4 + σ2

σ2 = 4− 3k + σ2

4 = 3k

k =4

3

26. The marginal density of Y is that of an exponential with mean 1, so E[Y ] = 1 and E[Y 2] =Var(Y ) + (EY )2 = 1 + 1 = 2.

We have two basic approaches. One is to use double expectation, which we can only do withthe raw moments of X, and the other is to use the law of total variation. Using double expectation:

E[X | Y = y] = 3y

c©2012 The Infinite Actuary, LLC p. 10 C.2 Solutions

Page 121: The In nite Actuary Exam 1/P Online Seminar A.1 ... · The In nite Actuary Exam 1/P Online Seminar A.1 Fundamentals of Probability Solutions Last updated February 23, 2013 1. Let

E[X] = E[E[X | Y ]

]= E[3Y ]

= 3E[Y ] = 3 · 1 = 3

E[X2 | Y = y] = Var(X | Y ) + [E(X | Y )]2

= 2 + (3y)2 = 2 + 9y2

E[X2] = E[E[X2 | Y ]

]= E[2 + 9y2]

= 2 + 9 · 2 = 20

Var(X) = E(X2)− (EX)2

= 20− 32 = 11

Or using the law of total variations

Var(X) = E[Var(X | Y )] + Var[E(X | Y )]

= E[2] + Var[3Y ]

= 2 + 9Var(Y ) = 2 + 9 · 1 = 11

27. We are given that the support of (X, Y ) is the shaded triangle:

X

Y Y = X

1

x

The joint density of X and Y doesn’t depend on Y , so given X = x, Y is uniform over thepossible values, i.e., given X = x, Y is uniform on (x, 1) and thus

Var[Y |X = x] =(1− x)2

12

28.

x

y

E[X3Y ] =

∫ 2

0

∫ x

x−2x3y · 1

4dy dx

c©2012 The Infinite Actuary, LLC p. 11 C.2 Solutions

Page 122: The In nite Actuary Exam 1/P Online Seminar A.1 ... · The In nite Actuary Exam 1/P Online Seminar A.1 Fundamentals of Probability Solutions Last updated February 23, 2013 1. Let

=

∫ 2

0

x3y2

8

∣∣∣∣xx−2

dx

=

∫ 2

0

4x4 − 4x3

8dx

=x5

10− x4

8

∣∣∣∣20

=32

10− 2 =

6

5

29. E[X] = −1 · (0.1 + 0.1) + 0 · (0.1 + 0.3) + 1 · (0.2 + 0.2) = 0.2

E[Y ] = 0 · (0.1 + 0.1 + 0.2) + 1 · (0.1 + 0.3 + 0.2) = 0.6

E[XY ] = −1 · (0.1) + 0 · (0.1 + 0.1 + 0.3 + 0.2) + 1 · (0.2) = 0.1

Cov(X, Y ) = E[XY ]− E[X] · E[Y ]

= 0.1− 0.2 · 0.6 = −0.02

30. D is true since that is the definition of the second moment. A is false because we are missingthe marginals and also don’t know that X and Y are independent. B is false because if we aredoing a single integral to find E[X], we want the marginal of X, not the joint density. C is falsebecause we should use the marginals, not joint densities, and also we don’t know that X and Yare independent. E is false because we want a moment of Y , so we should integrate dy using themarginal of Y , not X.

31.

E[X1] =∂

∂t1M(t1, t2)

∣∣∣∣(0,0)

=(0.1et1 + 0.4et1+t2

) ∣∣∣(0,0)

= 0.5

E[X2] =∂

∂t2M(t1, t2)

∣∣∣∣(0,0)

=(0.2et2 + 0.4et1+t2

) ∣∣∣(0,0)

= 0.6

E[2X1 −X2] = 2 · 0.5− 0.6 = 0.4

32. Since the domain is not a rectangle, X and Y are not independent so we cannot just say thatthe answer is 0. Fortunately they gave us E[X] and E[Y ] so we only have to find E[XY ]

E[XY ] =

∫ 1

0

∫ y

0

xy · 6x dx dy

=

∫ 1

0

2x3y∣∣∣y0dy

=

∫ 1

0

2y4 dy =2

5

c©2012 The Infinite Actuary, LLC p. 12 C.2 Solutions

Page 123: The In nite Actuary Exam 1/P Online Seminar A.1 ... · The In nite Actuary Exam 1/P Online Seminar A.1 Fundamentals of Probability Solutions Last updated February 23, 2013 1. Let

Cov(X, Y ) =2

5− 1

2· 3

4=

1

40

33. M(t1, t2) = E[et1X1+t2X2 ]

= et1·0+t2·0 · P[X1 = 0, X2 = 0] + et1·1+t2·0 · P[X1 = 1, X2 = 0]

+ et1·0+t2·1 · P[X1 = 0, X2 = 1] + et1·1+t2·1 · P[X1 = 1, X2 = 1]

= P[X1 = X2 = 0] + et1 · P[X1 = 1, X2 = 0]

+ et2 · P[X1 = 0, X2 = 1] + et1+t2P[X1 = X2 = 1]

= 0.3 + 0.1et1 + 0.2et2 + 0.4et1+t2

P[X1 = X2] = P[X1 = X2 = 0] + P[X1 = X2 = 1] = 0.3 + 0.4 = 0.7

34. MZ(t) = E[etZ ] = E[etX1−tX2 ]

= E[etX1+(−t)X2 ] = MX1,X2(t,−t)= 0.3 + 0.1et + 0.2e−t + 0.4et−t

= 0.7 + 0.1et + 0.2e−t

35. E[Y −X] =1

5(0− 1) +

2

5(1− 0) +

1

5(1− 1) +

1

5(2− 0) =

3

5

E[(Y −X)2] =1

5(0− 1)2 +

2

5(1− 0)2 +

1

5(1− 1)2 +

1

5(2− 0)2 =

7

5

Var[Y −X] =7

5−(

3

5

)2

=26

25

36.E[XY ] =

∫ 1

0

∫ y

0

(xy)

[x+

10

3y2]dx dy

=

∫ 1

0

y4

3+

5

3y5 dy

=1

15+

5

18=

31

90

37. We have two basic approaches. The first is to use double expectation directly with the rawmoments.

E[N | λ] = λ Var[N | λ] = λ E[N2 | λ] = λ+ λ2

E[N ] = E[E[N | λ]] = E[λ] = 3

E[N2] = E[E[N2 | λ]] = E[λ+ λ2] = 3 + 2 · 32 = 21

Var[N ] = 21− 32 = 12

Alternatively, we could use the law of total variation

Var[N ] = E[Var[N | λ]] + Var[E[N | λ]]

c©2012 The Infinite Actuary, LLC p. 13 C.2 Solutions

Page 124: The In nite Actuary Exam 1/P Online Seminar A.1 ... · The In nite Actuary Exam 1/P Online Seminar A.1 Fundamentals of Probability Solutions Last updated February 23, 2013 1. Let

= E[λ] + Var[λ] = 3 + 32 = 12

38. Our region is

Y

X−2 2 Since X and Y are uniform over that region, the

joint density is

f(x, y) =1

area=

1

2 · (1/2) · 2 · 2=

1

4

E[Y ] =

∫ 0

−2

∫ −x0

y

4dy dx+

∫ 2

0

∫ x

0

y

4dy dx

=

∫ 0

−2

y2

8

∣∣∣∣−x0

dx+

∫ 2

0

y2

8

∣∣∣∣x0

dx

=

∫ 0

−2

x2

8dx+

∫ 2

0

x2

8dx =

x3

24

∣∣∣∣0−2

+x3

24

∣∣∣∣20

=8

24+

8

24=

2

3

39.

x

y

y = 1 + x y = 1− x

y = −1 + xy = −1− x

E[X] = 0 since the density is symmetric about the y-axis. Alternatively,

f(x, y) =1

area=

1

4 · (1/2) · 1 · 1=

1

2

E[X] =

∫ 0

−1

∫ 1+x

−1−x

x

2dy dx+

∫ 1

0

∫ 1−x

−1+x

x

2dy dx

=

∫ 0

−1(2 + 2x) · x

2dx+

∫ 1

0

(2− 2x) · x2dx

=

∫ 0

−1x+ x2 dx+

∫ 1

0

x− x2 dx = −(

1

2− 1

3

)+

(1

2− 1

3

)= 0

E[X2] =

∫ 0

−1

∫ 1+x

−1−x

x2

2dy dx+

∫ 1

0

∫ 1−x

−1+x

x2

2dy dx

=

∫ 0

−1(2 + 2x) · x

2

2dx+

∫ 1

0

(2− 2x) · x2

2dx

c©2012 The Infinite Actuary, LLC p. 14 C.2 Solutions

Page 125: The In nite Actuary Exam 1/P Online Seminar A.1 ... · The In nite Actuary Exam 1/P Online Seminar A.1 Fundamentals of Probability Solutions Last updated February 23, 2013 1. Let

=

∫ 0

−1x2 + x3 dx+

∫ 1

0

x2 − x3 dx = −(−1

3+

1

4

)+

(1

3− 1

4

)=

2

3− 2

4=

1

6

Var[X] =1

6− 02 =

1

6

40.

X

YY = X

10.6

Before conditioning,

fX,Y (x, y) = fX(x) · fY |X(y | X = x) = 2x · 1

x= 2

Since the joint density is constant, (X, Y ) are uniform on the large triangle. Given that X < 0.6,every point in the shaded triangle is still equally likely, so the conditional distribution is uniformand hence the conditional joint density is 1/(shaded area) = 1/(0.5 · 0.62) = 50/9. Alternatively,

f(x, y | X < 0.6)f(x, y)

P[X < 0.6]=

2

[0.5 · 0.62]/[0.5 · 12]=

50

9

E[Y | X < 0.6] =

∫ 0.6

0

∫ x

0

y · 50

9dy dx

=

∫ 0.6

0

25x2

9dx =

25 · 0.63

27= 0.2

c©2012 The Infinite Actuary, LLC p. 15 C.2 Solutions

Page 126: The In nite Actuary Exam 1/P Online Seminar A.1 ... · The In nite Actuary Exam 1/P Online Seminar A.1 Fundamentals of Probability Solutions Last updated February 23, 2013 1. Let

The Infinite Actuary Exam 1/P Online SeminarC.3 Sums and Transformations SolutionsLast updated July 23, 2012

1.

X1

Y

1

2

y = 1− x

P [loss ≥ 1] = 1− P [loss < 1]

= 1−1∫

0

1−x∫0

2x + 2− y

4dy dx

= 1−1∫

0

2x + 2

4· (1− x)− (1− x)2

8dx

= 1− 1

8

1∫0

(4x + 4)(1− x)− (1− x)2 dx

= 1− 1

8

1∫0

3 + 2 x− 5x2 dx

= 1− 1

8

[3 + 1− 5

3

]= 0.7083

2. Let X denote the loss with deductible 1, and Y the loss with deductible 2. If X ≤ 1 then thatloss results in 0 payment, so if X ≤ 1 then the total payment is at most 5 when Y ≤ 5 + 2 = 7.Likewise, if Y ≤ 2 then that loss results in 0 payment, so if Y ≤ 2 then the total payment is at most5 if X ≤ 5+1 = 6. Finally, if X > 1 and Y > 2 then the total payment is X−1+Y −2 = X+Y −3so we want X + Y ≤ 5 + 3 = 8. That gives the following region:

ded= 1

ded= 2

10

10

2

7

1 6

x + y = 8

(6, 2)

c©2012 The Infinite Actuary, LLC p. 1 C.3 Solutions

Page 127: The In nite Actuary Exam 1/P Online Seminar A.1 ... · The In nite Actuary Exam 1/P Online Seminar A.1 Fundamentals of Probability Solutions Last updated February 23, 2013 1. Let

P[total paid ≤ 5] =shaded area

total area=

1 · 2 + 5 · 2 + 1 · 5 + 12

52

100= 0.295

3. Since the joint density is constant over the possible values, we have a uniform distribution andthe density is 1/area. So first let’s draw a picture and find the area.

T1

T2

4 6

6(4, 6)

(6, 4)

Area = Area of a 6x6 square

− Area of missing triangle

= 6 · 6− 1

2· 2 · 2 = 34

or Area = Area of blue rectangle

+ Area of green trapezoid

= 4 · 6 + 2 · 4 + 6

2= 34

Joint density =1

area=

1

36− 12· 22

=1

34

E[T1 + T2] =

4∫0

6∫0

t1 + t234

dt2 dt1 +

6∫4

10−t1∫0

t1 + t234

dt2 dt1

=

4∫0

t1t2 + 12t22

34

∣∣∣∣60

dt1 +

6∫4

t1t2 + 12t22

34

∣∣∣∣10−t10

dt1

=

4∫0

6t1 + 18

34dt1 +

6∫4

10t1 − t21 + 50− 10t1 + 12t21

34dt1

=3t21 + 18t1

34

∣∣∣∣40

+

6∫4

50− 12t21

234 dt1

=48

34+

72

34+

1

34

[(50 · 2− t31

6

)∣∣∣∣64

]= 5.73

c©2012 The Infinite Actuary, LLC p. 2 C.3 Solutions

Page 128: The In nite Actuary Exam 1/P Online Seminar A.1 ... · The In nite Actuary Exam 1/P Online Seminar A.1 Fundamentals of Probability Solutions Last updated February 23, 2013 1. Let

4.

T1

T2

x

2

x

2T1 + T2 = x, T2 = x− 2T1

First, let’s find the CDF, FX(x) = P[2T1 + T2 ≤ x] Then we differentiate to get fX(x).

FX(x) =

x/2∫0

x−2 t1∫0

e−t1−t2 dt2 dt1

=

x/2∫0

−e−t1−t2∣∣∣x−2 t10

dt1

=

x/2∫0

−e−t1−x+2 t1 + e−t1 dt1 =

x/2∫0

e−t1 − e−x+t1 dt1

=(−e−t1 − e−x+t1

) ∣∣∣x/20

= −2 e−x/2 − (−1− e−x)

FX(x) = 1− 2 e−x/2 + e−x

fX (x) = −2 · −1

2e−x/2 − e−x = e−x/2 − e−x

= e−x/2(1− e−x/2)

Note that we do not have a Gamma distribution because even though we are summing twoindependent exponentials, they have different means.

5.

P

C

C = xP

c©2012 The Infinite Actuary, LLC p. 3 C.3 Solutions

Page 129: The In nite Actuary Exam 1/P Online Seminar A.1 ... · The In nite Actuary Exam 1/P Online Seminar A.1 Fundamentals of Probability Solutions Last updated February 23, 2013 1. Let

Using the CDF approach,

P[X ≤ x] = P

[C

P≤ x

]= P

[C ≤ xP

]= P

[P ≥ C

x

]=

∞∫0

∞∫C/x

1

2e−P/2e−C dp dc

=

∞∫0

−e−P/2−C∣∣∣∣∞C/x

=

∞∫0

e−C[1+(1/2x)]dx

=

∞∫0

e−C[(2x+1)/(2x)]dc

=−2x

2x + 1e−C[(2x+1)/(2x)]

∣∣∣∣∞0

=2x

2x + 1=

2x + 1− 1

2x + 1= 1− 1

2x + 1

fX(x) =d

dx

[1− 1

2x + 1

]=

1

(2x + 1)2· 2 =

2

(2x + 1)2

If we want to use the change of variable approach, let g(C,P ) = (X,P ) (the second coordinate isarbitrary and is to make finding the inverse easier). To find the inverse, we want to solve for Cand P in terms of X and P . But this is just

P = P

X =C

P, C = PX

g−1(X,P ) = (PX,P )

fX,P (x, p) = fC,P (c = px, p = p) · |J | = 1

2e−px−p/2 · |J |

|J | =

∣∣∣∣∣∂c∂x

∂c∂p

∂p∂x

∂p∂p

∣∣∣∣∣ =

∣∣∣∣ p x0 1

∣∣∣∣ = p · 1− x · 0 = p

fX,P (x, p) =1

2e−px−p/2 · p =

p

2e−p(2x+1)/2)

fX(x) =

∫ ∞0

fX,P (x, p) dp =

∫ ∞0

p

2e−p(2x+1)/2 dp

=1

2· 2

2x + 1

∫ ∞0

p(2x + 1)

2e−p(2x+1)/2 dp

=1

2· 2

2x + 1· 2

2x + 1=

2

(2x + 1)2

c©2012 The Infinite Actuary, LLC p. 4 C.3 Solutions

Page 130: The In nite Actuary Exam 1/P Online Seminar A.1 ... · The In nite Actuary Exam 1/P Online Seminar A.1 Fundamentals of Probability Solutions Last updated February 23, 2013 1. Let

6. We want P[X + Y ≤ 60], or the chance of being in the shaded area.

x

y

60

60x + y = 60

60∫0

60−x∫0

f(x, y) dy dx =1

800

60∫0

60−x∫0

e−x/40e−y/20 dy dx

7.

x

y

3

1

1 3

(2, 3)

(3, 2)y = 5− x

P[Sum > 5] =Area of Triangle

Total Area=

12· 1 · 12 · 2

=1

8

8. Since our joint distribution is discrete, we can just list the possible outcomes.

P

[X +

Y

2≤ 5

]= 1− P

[X +

Y

2> 5

]= 1− (P[X = 4, Y = 4] + P[X = 2, Y = 8] + P[X = 4, Y = 8])

= 1− 4

24 · 4− 8

24 · 2− 8

24 · 4

= 1− 1

24− 4

24− 2

24=

17

24

9. First, we need to find c, then we need to integrate over the region we care about. The range ofpossible values is the rectangle, and the shaded area is the region that we care about.

x

y

1 2 3

1

c©2012 The Infinite Actuary, LLC p. 5 C.3 Solutions

Page 131: The In nite Actuary Exam 1/P Online Seminar A.1 ... · The In nite Actuary Exam 1/P Online Seminar A.1 Fundamentals of Probability Solutions Last updated February 23, 2013 1. Let

1 =

∫ 3

1

∫ 1

0

f(x, y) dy dx =

∫ 3

1

∫ 1

0

(cx +

y

2

)dy dx

=

∫ 3

1

(cx +

1

4

)dx = c · 9− 1

2+

1

21

8= c

P[X + Y > 2] =

∫ 1

0

∫ 3

2−yf(x, y) dy dx =

∫ 1

0

∫ 3

2−y

x

8+

y

2dy dx

=

∫ 1

0

(9

16+

3y

2− (2− y)2

16− 2y − y2

2

)dy

=

∫ 1

0

(9 + 24y − 4 + 4y − y2 − 16y + 8y2

16

)dy

=

∫ 1

0

(7y2 + 12y + 5

16

)dy

=7

48+

12

32+

5

16=

5

6

10.

x

y

(z, 0)

(1, z − 1)

x + y = z

x = 1

We have two choices: we could either integrate the joint density along the green line to directlyfind fZ(z), or we could do a double integral over the blue region to find the cdf of Z and thendifferentiate.

fZ(z) =

∫ x=z

x=1

f(x, z − x) dx

=

∫ z

1

e−x−(z−x)+1dx =

∫ z

1

e−z+1dx = (z − 1)e−z+1

P[Z ≤ z] =

∫ z

1

∫ z−x

0

e−x−y+1 dy dx

=

∫ z

1

(−e−x−y+1

)∣∣z−x0

dx

=

∫ z

1

−e−x−(z−x)+1 + e−x+1dx =

∫ z

1

e−x+1 − e−z+1dx

= −e−z+1 + e−1+1 − (z − 1)e−z+1 = 1− ze−z+1

c©2012 The Infinite Actuary, LLC p. 6 C.3 Solutions

Page 132: The In nite Actuary Exam 1/P Online Seminar A.1 ... · The In nite Actuary Exam 1/P Online Seminar A.1 Fundamentals of Probability Solutions Last updated February 23, 2013 1. Let

fZ(z) = ze−z+1 − e−z+1 = (z − 1)e−z+1

11.

x

y

2

1

P[X + Y < 1.5] =

∫ 1

0

∫ 1.5−x

0

f(x, y) dy dx

=

∫ 1

0

6

7

(x2(1.5− x) +

x(1.5− x)2

4

)dx

=6

7

[1.5

3− 1

4+

1

16− 3

12+

1.52

8

]=

33

112= 0.2946

12. P[Z ≤ z] =

∫ z

0

∫ z−x

0

24xy dy dx

=

∫ z

0

12x(z − x)2 dx

=

∫ z

0

12x3 − 24x2z + 12xz2 dx

= 3z4 − 8z4 + 6z4 = z4

fZ(z) = 4z3

Or, using the density directly

fZ(z) =

∫ z

0

f(x, z − x) dx

=

∫ z

0

24x(z − x) dx =

∫ z

0

24xz − 24x2 dx

= 12z3 − 8z3 = 4z3

13. Because we have a discrete distribution, we again want to sum over possible cases that work,namely (X, Y ) = (1, 1) or (X, Y ) = (2, 2). Summing those probabilities gives us 0.1 + 0.2 = 0.3

14. P[X + Y > 1.5] =

∫ 1

0.5

∫ 1

1.5−x(x + y)dy dx

c©2012 The Infinite Actuary, LLC p. 7 C.3 Solutions

Page 133: The In nite Actuary Exam 1/P Online Seminar A.1 ... · The In nite Actuary Exam 1/P Online Seminar A.1 Fundamentals of Probability Solutions Last updated February 23, 2013 1. Let

=

∫ 1

0.5

x +1

2− x(1.5− x)− (1.5− x)2

2dx

=

∫ 1

0.5

x2

2+ x− 5

8dx

=1− 0.53

6+

1− 0.52

2− 5

16=

5

24

15.

x

y

1

1

P

[X

Y< 2

]= P[X < 2Y ] =

shaded area

area of square=

3

4

16.

x

y

y = x/2

P

[X

Y< 2

]= P[Y > X/2]

=

∫ ∞0

∫ ∞x/2

e−x−ydy dx

=

∫ ∞0

e−xe−x/2dx

=1

−3/2e−3x/2

∣∣∣∣∞0

=2

3

c©2012 The Infinite Actuary, LLC p. 8 C.3 Solutions

Page 134: The In nite Actuary Exam 1/P Online Seminar A.1 ... · The In nite Actuary Exam 1/P Online Seminar A.1 Fundamentals of Probability Solutions Last updated February 23, 2013 1. Let

17.

X

Y

1

1

u = xy, y = u/x

Using the CDF method,

P[U ≤ u] =

u∫1

u/x∫1

2

x3 y2dy dx =

u∫1

−2

x3y

∣∣∣∣u/x1

dx

=

u∫1

(2

x3− 2

x3 · (u/x)

)dx =

u∫1

2

x3− 2

x2udx

=−1

x2+

2

xu

∣∣∣∣u1

=

(2

u2− 1

u2

)−(−1 +

2

u

)FU(u) = 1− 2

u+

1

u2

fU(u) =d

duFU(u) =

2

u2+−2

u3

Using the change of variable method, let g(X, Y ) = (U, Y ). Then

U = XY, X = U/Y

|J | =

∣∣∣∣∣∣∣∣∂x

∂u

∂x

∂y∂y

∂u

∂y

∂y

∣∣∣∣∣∣∣∣ =

∣∣∣∣∣∣1

y

−uy2

0 1

∣∣∣∣∣∣ =1

y· 1− −u

y2· 0 =

1

y

fU,Y (u, y) = fX,Y (x = u/y, y = y) · |J |

=2

(u/y)3 · y2· 1

y=

2

u3

fU(u) =

∫ u

1

f(u, y) dy =

∫ u

1

2

u3dy =

2

u2− 2

u3

18. Step 1: Find g−1, i.e., solve for X and Y in terms of U and V .

U = XY, V =X

Y

c©2012 The Infinite Actuary, LLC p. 9 C.3 Solutions

Page 135: The In nite Actuary Exam 1/P Online Seminar A.1 ... · The In nite Actuary Exam 1/P Online Seminar A.1 Fundamentals of Probability Solutions Last updated February 23, 2013 1. Let

UV = X2, X =√UV

U =√UV · Y, Y =

√U/V

Step 2: Find J

J =

∣∣∣∣∣∣∣∣∂x

∂u

∂x

∂v∂y

∂u

∂y

∂v

∣∣∣∣∣∣∣∣ =

∣∣∣∣∣∣∣∣√v

2√u

√u

2√v

1

2√uv

−√u

2v3/2

∣∣∣∣∣∣∣∣=

√v

2√u· −√u

2v3/2− u

2√v· 1

2√uv

=1

4

[−1

v− 1

v

]=−1

2v

Step 3: Plug everything into the formula

fU,V (u, v) = fX,Y (x =√uv, y =

√u/v) · |J |

=2

u3/2v3/2u2/2v−2/2·∣∣∣−1

2v

∣∣∣ =1

u5/2v3/2

19. Step 1: Solve for X and Y in terms of Z and W .

Z = X − Y

W = X + Y

Z + W = 2X, X =Z + W

2Y =

W − Z

2

Step 2: Find J

J =

∣∣∣∣∣∣∣∣∂x

∂z

∂x

∂w∂y

∂z

∂y

∂w

∣∣∣∣∣∣∣∣ =

∣∣∣∣∣∣∣1

2

1

2−1

2

1

2

∣∣∣∣∣∣∣=

1

2· 1

2− 1

2· −1

2=

1

2

Step 3: Plug into the formula

fZ,W (z, w) = fX,Y (x = (z + w)/2, y = (w − z)/2) · |J |

= 4

(z + w

2

)(w − z

2

)· 1

2=

w2 − z2

2

c©2012 The Infinite Actuary, LLC p. 10 C.3 Solutions

Page 136: The In nite Actuary Exam 1/P Online Seminar A.1 ... · The In nite Actuary Exam 1/P Online Seminar A.1 Fundamentals of Probability Solutions Last updated February 23, 2013 1. Let

20.

X

Y

1

1

z = xy, y = z/x

P [Z ≤ z] = 1− P [Z > z]

= 1−1∫

√z

x∫zx

10x2y dy dx

= 1−1∫

√z

5x2 y2∣∣∣xzx

dx = 1−1∫

√z

5x4 − 5 z2 dx

P [Z ≤ z] = 1−1∫

√z

5x4 − 5 z2 dx

= 1− x5∣∣∣1√

z+ 5 z2 · x

∣∣∣1√z

= 1− (1− z5/2) + 5 z2 − 5 z5/2

= z5/2 + 5 z2 − 5 z5/2

= 5 z2 − 4 z5/2

21. To find the range where the new joint density is positive, we know that Y < 1 so U = XY <X < 1. Since W = X, that means that U < W < 1. We also know that X < Y , so U > X2 = W 2,giving us 0 < W 2 < U < W < 1. That reduces the answer choices to A and D. As for the formulafor the density,

X = W

U = XY = WY, Y =U

WfU,W (u,w) = fX,Y (x = w, y = u/w) · |J |

J =

∣∣∣∣∣∣∣∣∂x

∂w

∂x

∂u∂y

∂w

∂y

∂u

∣∣∣∣∣∣∣∣ =

∣∣∣∣∣∣∣1 0

− u

w2

1

w

∣∣∣∣∣∣∣c©2012 The Infinite Actuary, LLC p. 11 C.3 Solutions

Page 137: The In nite Actuary Exam 1/P Online Seminar A.1 ... · The In nite Actuary Exam 1/P Online Seminar A.1 Fundamentals of Probability Solutions Last updated February 23, 2013 1. Let

= 1 · 1

w− 0 ·

( u

w2

)=

1

w

fU,W (u,w) = 2 · 1

w=

2

w

22. Z = XY W =X

Y

ZW = X2 X =√ZW, Y =

√z

w

fZ,W (z, w) = fX,Y

(x =√zw, y =

√z

w

)· |J |

J =

∣∣∣∣∣∣∣∣∂x

∂w

∂x

∂z∂y

∂w

∂y

∂z

∣∣∣∣∣∣∣∣ =

∣∣∣∣∣∣∣∣∣1

2

√z

w

1

2

√w

z

−1

2

√z

w3

1

2

√1

zw

∣∣∣∣∣∣∣∣∣=

1

4w− −1

4w=

1

2w

fZ,W (z, w) =1

2wfX,Y

(√zw,

√z

w

)

23. U = X + Y, V =X

X + Y=

X

UX = V U, Y = U − V U

J =

∣∣∣∣∣∣∣∣∂x

∂u

∂x

∂v∂y

∂u

∂y

∂v

∣∣∣∣∣∣∣∣ =

∣∣∣∣∣ v u

1− v −u

∣∣∣∣∣= v(−u)− u(1− v) = −u

fU,V (u, v) = fX,Y (x = uv, y = u(1− v))| − u| = 1

12· u = u

c©2012 The Infinite Actuary, LLC p. 12 C.3 Solutions

Page 138: The In nite Actuary Exam 1/P Online Seminar A.1 ... · The In nite Actuary Exam 1/P Online Seminar A.1 Fundamentals of Probability Solutions Last updated February 23, 2013 1. Let

The Infinite Actuary Exam 1/P Online SeminarC.4 Bivariate Normal Solutions

1. Unfortunately neither A nor B is normally distributed because there is a positive probabilityof no claim being made. So before we can use what we know about normal distributions, we haveto condition on a payment being made for both A and B.

I will use A > 0 and B > 0 to denote the case when A and B have claims (that is technicallywrong as there is a very small probability of the normal variables being negative, but it simplifiesthe notation).

P[B > A] = P[B > A,A > 0] + P[B > A,A = 0]

= P[A > 0] · P[B > A | A > 0] + P[A = 0] · P[B > A | A = 0]

= 0.4 · P[B > 0] · P[B > A | A,B > 0] + 0.6 · P[B > 0]

= (0.4)(0.3)P[B > A | A,B > 0] + (0.6)(0.3)

And when A and B both have claims we have B−A is a normal random variable with parameters

E[B − A] = E[B]− E[A] = 9,000− 10,000 = −1,000

Var[B − A] = VarB + (−1)2VarA

= 2,0002 + 2,0002 = 8,000,000

SD[B − A] = 2,828

P[B − A > 0 | A,B > 0] = P

[B − A− (−1,000)

2,828>

0− (−1,000)

2,828| A,B > 0

]= 1− Φ

(1,000

2,828

)= 1− Φ(0.35) = 1− 0.64 = 0.36

P[B > A] = 0.4 · 0.3 · 0.36 + 0.6 · 0.3 = 0.223

2. For one person,

E(X + Y ) = EX + EY = 70

Var(X + Y ) = Var(X) + 2Cov(X, Y ) + Var(Y )

= 50 + 2 · 10 + 30 = 100

and so for all 100 people,

E[T ] = 100 · 70 = 7,000

Var[T ] = 100 · 100 = 1002

T ≈ N(7,000, 100 · 100)

P[T < 7,100] = P

[T − 7,000√

1002<

7,100− 7,000√1002

]≈ Φ(1) = 0.8413

c©2012 The Infinite Actuary, LLC p. 1 C.4 Solutions

Jim James
Highlight
Page 139: The In nite Actuary Exam 1/P Online Seminar A.1 ... · The In nite Actuary Exam 1/P Online Seminar A.1 Fundamentals of Probability Solutions Last updated February 23, 2013 1. Let

3. MW,Z(t1, t2) = E[et1W+t2Z

]= E

[et1(X+Y )+t2(Y−X)

]= E

[eX(t1−t2)+Y (t1+t2)

]= E

[eX(t1−t2)

]· E[eY (t1+t2)

]by independence

= e(t1−t2)2/2e(t1−t2)2/2 using MGF of X and Y

= et21+t22

Alternatively, we can use what we know about MGFs of normals. If N is normal, then

MN(t) = eE[tN ]+(1/2)Var(tN)

MX(t) = et2/2

So X and Y are standard normals

E[W ] = E[X + Y ] = E[X] + E[Y ] = 0

Var[W ] = Var(X + Y ) = 1 + 1 = 2

E[Z] = E[Y ]− E[X] = 0

Var(Z) = Var(Y ) + (−1)2Var(X) = 2

Cov(W,Z) = Cov(X + Y,−X + Y )

= −Var(X) + Cov(X, Y ) + Cov(Y,−X) + Var(Y )

= −1 + 0 + 0 + 1 = 0

MW,Z (t1, t2) = eE [t1W+t2Z]+(1/2)Var [t1W+t2Z]

= exp[0 + 0 + (1/2)(t21 Var(W ) + t22 Var(Z) + 0)]

= exp

[1

2[t21 · 2 + t22 · 2]

]= et

21+t22

4. Y −X is a linear combination of independent normals, and is therefore normal.

E[Y −X] = 5− 3 = 2

Var[Y −X] = VarY + (−1)2VarX

= 16 + 9 = 25

SD[Y −X] = 5

P[Y −X > 7] = P

[Y −X − 2

5>

7− 2

5

]= 1− Φ(1) = 1− 0.84 = 0.16

5. E(X) = 2E[Z1]− E[Z3] = 2 · 0− 0 = 0

E(Y ) = 2E[Z2] + E[Z3] = 2 · 0 + 0 = 0

Var(X) = 22 · VarZ1 + (−1)2Var (Z3)

= 4 + 1 = 5

c©2012 The Infinite Actuary, LLC p. 2 C.4 Solutions

Page 140: The In nite Actuary Exam 1/P Online Seminar A.1 ... · The In nite Actuary Exam 1/P Online Seminar A.1 Fundamentals of Probability Solutions Last updated February 23, 2013 1. Let

Var(Y ) = 22 · 1 + 12 · 1 = 5

Cov(X, Y ) = E(XY )− E(X)E(Y )

= Cov(2Z1 − Z3, 2Z2 + Z3)

= Cov(2Z1, 2Z2) + Cov(2Z1 + Z3) + Cov(−Z3, 2Z2) + Cov(−Z3, Z3)

= 0 + 0 + 0 + (−1) · Var[Z3] = −1

Corr (X, Y ) =−1√5√

5=−1

5

6. For standard normals, E[Z1 | Z2 = z] = ρ · z. For general normals, E[X | Y = y] is E[X] plusρ · SD(x) · z, where z = (y − E[Y ])/SD(Y ) is how many standard deviations y is above the mean.

E(Y ) = −1, Var(Y ) = 4, SD(Y ) = 2

1− (−1)

2= 1

i.e., 1 is one SD above the mean of Y

E[X | Y = 1] = E[X] + ρ · SD(X) · 1

= 1 +−1

2

√5− 12 = 1− 1

2· 2 = 0

7. The conditional variance of Y is 0 if ρ = ±1 and is Var(Y ) if ρ = 0. In general, the formula forthe conditional variance is arguably the simplest formula that meets those requirements, namely

Var(Y | X) = (1− ρ2)Var(Y ) =

[1−

(1

2

)2]· 2 =

3

2

8. E[3X + 2Y − Z] = 3E[X] + 2E[Y ]− E[Z]

= 3 · 1 + 2 · 2− 3 = 4

Var[3X + 2Y − Z] = 32Var[X] + 22Var[Y ] + (−1)2Var[Z]

+2 · 3 · 2Cov(X, Y ) + 2 · 3 · (−1)Cov(X,−Z) + 2 · 2 · (−1)Cov(Y, Z)

= 9 · 4 + 4 · 5 + 9 + 12 · 2− 6 · 3− 4 · 1 = 67

9. E[Y − X] = E[Y ]− E[X] = E[Y ]− E[X]

= 0− 0 = 0

Var[Y − X] = Var[Y ] + Var[X] =Var[Y ]

100+

Var[X]

100

=602

12 · 100+

20√

3)2

12 · 100= 4

P[Y − X < 1] = P

[Y − X − 0√

4<

1− 0

2

]= Φ(0.5) = 0.6915

c©2012 The Infinite Actuary, LLC p. 3 C.4 Solutions

Page 141: The In nite Actuary Exam 1/P Online Seminar A.1 ... · The In nite Actuary Exam 1/P Online Seminar A.1 Fundamentals of Probability Solutions Last updated February 23, 2013 1. Let

10. Let X denote the father’s height, and Y denote the son’s height. Since both are adult males,we have E[X] = E[Y ] = 68 and SD[X] = SD[Y ] = 2.5.

Var[Y | X = 72] = (1− ρ2) · 2.52

2 = (1− ρ2) · 2.52

ρ2 = 0.68, ρ = 0.8246

E[Y | X = 72] = ρ · z · SD[Y ] + E[Y ]

= 0.8246 · 72− 68

2.5· 2.5 + 68

= 71.3

11. Since X and Y are normal, the 60th percentile of Y is µY + 0.2533σY = 0.2533σY and the70th percentile of X is µX + 0.5244σX = 0.5244σX . Those are equal, so σY = (0.5244/0.2533)σX

The 70th percentile of Y is 0.5244σY = (0.52442/0.2533)σX = 1.0854σX , and Φ(1.0854) =0.8611, so that is the 86th percentile of X.

Calculation note: I used a computer to compute all of the numbers to a greater precision thanyou are given on the tables. On the exam, you can either round to 2 decimal places, and thenchoose the answer choice closest to what you end up with (doing so here gives you 0.8599) or youcould do linear interpolation, which will take longer but typically give you something closer to the“exact” answer.

12. Let N denote the number of shutdowns, and S the total cost. Then

P[S > 150] = P[N = 0] · P[S > 150 | N = 0] + P[N = 1] · P[S > 150 | N = 1]

+ P[N = 2] · P[S > 150 | N = 2]

= 0 + 0.5 · P[S − 100

30>

150− 100

30| N = 1

]+ 0.2 · P

[S − 200

30√

2>

150− 200

30√

2| N = 2

]= 0.2 · 0 + 0.5

[1− Φ

(50

30

)]+ 0.3

[1− Φ

(−50

30√

2

)]= 0 + 0.5(1− 0.95) + 0.3(0.88) = 0.29

c©2012 The Infinite Actuary, LLC p. 4 C.4 Solutions

Page 142: The In nite Actuary Exam 1/P Online Seminar A.1 ... · The In nite Actuary Exam 1/P Online Seminar A.1 Fundamentals of Probability Solutions Last updated February 23, 2013 1. Let

The Infinite Actuary Exam 1/P Online SeminarC.5 Order Statistics SolutionsLast updated July 21, 2012

1. Let X1, X2, X3 and X4 denote the 4 sealed bids, and let Y = min{Xi} be the accepted bid.

P[Y ≤ x] = P[all Xi are ≤ x]

=(P[X1 ≤ x]

)4= [F (x)]4

=

(1

2

)4

(1 + sin(πx))4

d

dxP[Y ≤ x] = fY (x) =

4

16(1 + sin(πx))3(π cos(πx))

E[bid] =

5/2∫3/2

1

4π cos(π x)(1 + sin(πx))3x dx

2. Let X1, X2 and X3 be the 3 claims and let Y be their maximum. Then

P[Y ≤ x] = P[all 3Xi ≤ x] = [F (x)]3

=

x∫1

3

t4dt

3

=

(−1

t3

∣∣∣∣x1

)3

=

(1− 1

x3

)3

fY (x) = 3

(1− 1

x3

)2 (3

x4

)= 3(1− 2x−3 + x−6)(x−4) · 3= 9x−4 − 18x−7 + 9x−10

E[Y ] =

∞∫1

fY (x) · x dx

=

∞∫1

9x−3 − 18x−6 + 9x−9 dx

=−9

2x−2 +

18

5x−5 − 9

8x−8∣∣∣∣∞1

=9

2− 18

5+

9

8

= 2.025 thousand = 2,025

3.

P

[Y >

1

2

]= P

[at least 1 Xi is ≥ 1

2

]c©2012 The Infinite Actuary, LLC p. 1 C.5 Solutions

Page 143: The In nite Actuary Exam 1/P Online Seminar A.1 ... · The In nite Actuary Exam 1/P Online Seminar A.1 Fundamentals of Probability Solutions Last updated February 23, 2013 1. Let

= 1− P

[none of the Xi are >

1

2

]= 1−

[FX

(1

2

)]3

= 1−

1/2∫0

3x2 dx

3

= 1−(x3∣∣∣1/20

)3

= 1−(

1

8

)3

= 1− 1

512=

511

512

4. In order for exactly one of the two variables to exceed 1/2, there are two cases: Either X > 1/2and Y ≤ 1/2, or X ≤ 1/2 and Y > 1/2. This means we want

P

[X >

1

2, Y ≤ 1

2

]+ P

[X ≤ 1

2, Y >

1

2

]= P

[X >

1

2

]P

[Y ≤ 1

2

]+ P

[X ≤ 1

2

]P

[Y >

1

2

]= 2 ·

(1− F

(1

2

))F

(1

2

)

F

(1

2

)=

1/2∫0

2− 2 t dt

= 2t− t2∣∣∣1/20

= 1− 1

4=

3

4

Answer = 2

(1− 3

4

)(3

4

)= 2 · 1

4· 3

4=

6

16

5. For one of the variables,

P[X ≤ 1] =

∫ 1

0

√2− x dx

=√

2− 1

2

To have exactly 2 of the 3 variables exceed 1, we also need exactly 1 less than 1, so

P[Exactly 2 Xi > 1] =

(3

1

)P[X ≤ 1] (P[X > 1])2

= 3

(√2− 1

2

)(1−

(√2− 1

2

))2

= 3

(√2− 1

2

)(3

2−√

2

)2

c©2012 The Infinite Actuary, LLC p. 2 C.5 Solutions

Page 144: The In nite Actuary Exam 1/P Online Seminar A.1 ... · The In nite Actuary Exam 1/P Online Seminar A.1 Fundamentals of Probability Solutions Last updated February 23, 2013 1. Let

which is answer choice E

6. Let X be the lifetime of one component. Then

P[X > t] =

∫ ∞t

f(x)dx

=

∫ ∞t

3

x4dx = − 1

x3

∣∣∣∣∞t

=1

t3

If T is the lifetime of the total system, then

P[T > t] = (P[X > t])7

=

(1

t3

)7

= t−21

for t > 1, and is 1 for t ≤ 1. So

E[T ] =

∫ ∞0

P[T > t]dt

=

∫ 1

0

1dt+

∫ ∞1

t−21dt

= 1 +1

20= 1.05

7. Using the formula for the density of order statistics gives

fmin(y) =

(2

1

)· 1 · f(y) · [1− F (y)]

= 2(1− y)

Alternatively, Y > y if and only if X1 > y and X2 > y, so

P[Y > y] = P[X1, X2 > y]

= (1− y)2

P[Y ≤ y] = 1− (1− y)2

fY (y) = 0− 2(−1)(1− y) = 2(1− y)

8. In order to have Y2 < m < Y4 we need to have either exactly 2 of the data points be less thanm, or exactly 3. That gives us

P[Y2 < m < Y4] =

(5

2

)(1/2)2(1/2)3 +

(5

3

)(1/2)3(1/2)2

= 20 · 1

32=

5

8

c©2012 The Infinite Actuary, LLC p. 3 C.5 Solutions

Page 145: The In nite Actuary Exam 1/P Online Seminar A.1 ... · The In nite Actuary Exam 1/P Online Seminar A.1 Fundamentals of Probability Solutions Last updated February 23, 2013 1. Let

9. The event {X1 > X2 > X3} is the intersections of the events {X1 > X2} and {X2 > X3}, so

P[X1 > X2 or X2 > X3] = P[X1 > X2] + P[X2 > X3]− P[X1 > X2 > X > 3]

1 ≥ 0.7 + 0.6− P[X1 > X2 > X3]

P[X1 > X2 > X3] ≥ 0.3

10. Before we start finding the variance, let’s start by finding the density of the median.

P[Med ≤ x] = P[at least 2Xi are ≤ x]

= P[exactly 2 ≤ x] + P[all 3 ≤ x]

= 3 · x2(1− x) + x3

= 3x2 − 3x3 + x3 = 3x2 − 2x3

fmed(x) = 6x− 6x2

Or you could use the order statistics density formula with n = 3 and i = 2

fmed(x) =

(3

1

)(2

1

)· F (x)1(1− F (x))1f(x)

= 3 · 2 · x(1− x) · 1 = 6x− 6x2

E[median] =

1∫0

x · fmed(x) dx

=

1∫0

6x2 − 6x3 dx =6

3x3 − 6

4x4∣∣∣∣10

=1

2

E[median2] =

1∫0

6x3 − 6x4 dx =6

4x4 − 6

5x5∣∣∣∣10

=6

20

Var(median) =6

20−(

1

2

)2

=6

20− 5

20=

1

20

11. P[T ≤ t] = P[at least oneXi ≤ t]

= 1− P[all five Xi > t]

= 1− (P[X1 > t])5

= 1−(e−t/2

)5= 1− e−5t/2

fT (t) =d

dt

[1− e−5t/2

]=

5

2e−5t/2

Or we could use the order stat density formula with n = 5 and i = 1, giving

fT (t) =n!

(i− 1)!(n− i)!FX(t)i−1[1− FX(t)]n−ifX(t)

c©2012 The Infinite Actuary, LLC p. 4 C.5 Solutions

Page 146: The In nite Actuary Exam 1/P Online Seminar A.1 ... · The In nite Actuary Exam 1/P Online Seminar A.1 Fundamentals of Probability Solutions Last updated February 23, 2013 1. Let

=5!

0!4!

(1− e−t/2

)0 (e−t/2

)4 1

2e−t/2 =

5

2e−5t/2

12. Using the cdf approach

P[max{X1, . . . , X4} ≤ y] = P[all 4 are ≤ y] =(y

5

)4P[max{X1, . . . , X4} > y] = 1−

(y5

)4E[max{X1, . . . , X4}] =

5∫0

P[max{X1, . . . , X4} > y] dy

=

5∫0

1−(y

5

)4dy =

[y − 1

5

(y5

)5· 5]∣∣∣∣5

0

= 5− 1 = 4

Using the density approach with n = i = 4

fmax(y) = 4(y

5

)3 1

5

E[max{X1, . . . , X4}] =

5∫0

y · 4

5

(y5

)3dy

=

5∫0

4

54y4 dy =

4

54· 1

5y5∣∣∣∣50

=4 · 55

55= 4

13. P[max = 2] = P[max ≤ 2]− P[max ≤ 1]

=

(e−2 + 2e−2 +

22

2!e−2)5

−(e−2 + 2e−2

)5= 0.1308

14. There are 3 or 4 cases, depending on how you count (I will use 4 here, but my cases 2 and 3can be combined as in the video solution)

Case 1: two rolls equal 5, one roll equals 2

Case 2: one roll equals 5, one roll equals 4, one roll equals 2

Case 3: one roll equals 5, one roll equals 3, one roll equals 2

Case 4: one roll equal 5, two rolls equals 2

P[Case 1] =

(3

1

)(1

6

)2

· 1

6= 3

(1

6

)3

P[Case 2] = 3!

(1

6

)3

= 6 ·(

1

6

)3

c©2012 The Infinite Actuary, LLC p. 5 C.5 Solutions

Page 147: The In nite Actuary Exam 1/P Online Seminar A.1 ... · The In nite Actuary Exam 1/P Online Seminar A.1 Fundamentals of Probability Solutions Last updated February 23, 2013 1. Let

P[Case 3] = 3!

(1

6

)3

= 6 ·(

1

6

)3

P[Case 4] =

(3

1

)1

6

(1

6

)2

= 3

(1

6

)3

Total probability = [3 + 6 + 6 + 3]

(1

6

)3

=1

12

15. In order for only the median to be between 2 and 3, we need two variables to be at most 2,one variable to be between 2 and 3, and two variables to be at least 3. The probability of that is(using a multinomial distribution)

5!

2!1!2!(P[X ≤ 2])2P[2 < X < 3](P[X ≥ 3])2

For the components of that,

F (x) =

∫ x

0

f(t)dt =

∫ x

0

t

8dt =

x2

16

P[X ≤ 2] = F (2) =4

16

P[2 < X ≤ 3] = F (3)− F (2) =9− 4

16=

5

16

P[X > 3] = 1− F (3) =16− 9

16=

7

16

Answer =5!

2!1!2!

(4

16

)25

16

(7

16

)2

= 0.112

16. Let T denote the max.

P[T ≤ t] = (P[X ≤ t])3 = (t2)3 = t6

fT (t) = 6t5

E[T ] =

∫ 1

0

t · 6t5dt =6

7

17. 1

4= P[at least one Xi > d]

1

4= 1− P[both Xi ≤ d]

1

4= 1−

(d

100

)2

d = 50√

3

18. The first machine fails at time min{X, Y } and the second fails at time max{X, Y }, so we want

c©2012 The Infinite Actuary, LLC p. 6 C.5 Solutions

Page 148: The In nite Actuary Exam 1/P Online Seminar A.1 ... · The In nite Actuary Exam 1/P Online Seminar A.1 Fundamentals of Probability Solutions Last updated February 23, 2013 1. Let

E[max{X, Y }]. Our variables are uniformly distributed over

x

y

Within the lightblue region, y > x so max{x, y} = y, while within the green region, x > y andmax{x, y} = x, so

E[max{X, Y }] =

∫ 5

0

∫ 10−y

y

xf(x, y) dx dy +

∫ 5

0

∫ 10−x

x

yf(x, y) dy dx

= 2

∫ 5

0

∫ 10−y

y

x

50dx dy by symmetry

= 2

∫ 5

0

(10− y)2 − y2

100dy

=

∫ 5

0

100− 20y

50dy

= 10− 5 = 5

19. To simplify the numbers, I will do all computations in thousands, so A is uniform on (0, 5),etc. We are given B > 1, so our new region is

max = B

max = A

A

B

5

1

4

E[maxA,B | B > 1] =

∫ 4

1

∫ b

0

b

15da db+

∫ 4

1

∫ 5

b

a

15da db

=

∫ 4

1

b2

15db+

∫ 4

1

25− b2

30db

=43 − 13

45+

25(4− 1)

30− 43 − 13

90= 3.2

c©2012 The Infinite Actuary, LLC p. 7 C.5 Solutions

Page 149: The In nite Actuary Exam 1/P Online Seminar A.1 ... · The In nite Actuary Exam 1/P Online Seminar A.1 Fundamentals of Probability Solutions Last updated February 23, 2013 1. Let

20.

min = A

min = B

A

B

5

1

4

E[minA,B | B > 1] =

∫ 4

1

∫ b

0

a

15da db+

∫ 4

1

∫ 5

b

b

15da db

=

∫ 4

1

b2

30db+

∫ 4

1

5b− b2

15db

=43 − 13

90+

5(42 − 12)

30− 43 − 13

45= 1.8

c©2012 The Infinite Actuary, LLC p. 8 C.5 Solutions

Page 150: The In nite Actuary Exam 1/P Online Seminar A.1 ... · The In nite Actuary Exam 1/P Online Seminar A.1 Fundamentals of Probability Solutions Last updated February 23, 2013 1. Let

The Infinite Actuary Exam 1/P Online SeminarC.6 Multivariate Review SolutionsLast updated April 9, 2012

1.

x

y

1 3

3

1

The device fails during its first hour if either X < 1 or if Y > 1. There are two key cases: X < 1(blue region), and X > 1, Y < 1 (green region), giving∫ 1

0

∫ 3

0

f(x, y) dy dx +

∫ 3

1

∫ 1

0

f(x, y) dy dx =

∫ 1

0

(x + y)2

2 · 27

∣∣∣∣30

dx +

∫ 3

1

(x + y)2

2 · 27

∣∣∣∣10

dx

=

∫ 1

0

(3 + x)2

54− x2

54dx +

∫ 3

1

(1 + x)2

54− x2

54dx

=43 − 33

162− 13

162+

43 − 22

162− 33 − 13

162

=11

27= 0.41

2. Our picture is almost the same as before: either the first component fails in the first half hour(blue area) or the first component survives for the first half hour but the second fails in the firsthalf hour (green area),

s

t

1/2 1

1

1/2

giving an answer of

1∫0

1/2∫0

f(s, t) ds dt +

1/2∫0

1∫1/2

f(s, t) ds dt

3.fY |X (y | X = 2) =

f(2, y)∞∫1

f(2, y)dy

c©2012 The Infinite Actuary, LLC p. 1 C.6 Solutions

Page 151: The In nite Actuary Exam 1/P Online Seminar A.1 ... · The In nite Actuary Exam 1/P Online Seminar A.1 Fundamentals of Probability Solutions Last updated February 23, 2013 1. Let

=24.1

y−3

∞∫0

24.1

y−3 dy

=y−3

−12y−2

∣∣∞1

= 2 y−3 = fY |X (y | X = 2)

And we want

3∫1

2 y−3 dy = −y−2∣∣∣31

=−1

9− −1 =

8

9

4. Let N be the number of people hospitalized and Z the total cost. If N = 0 or 1 then the totalcost is less than 1. If N = 2, with costs X and Y for each, then X and Y are uniformly distributedover the following square:

x

y

The total cost is less than 1 if and only if (X, Y ) are in the shaded region, which takes up halfof the square, so P[Z < 1 | N = 2] = 1/2. That gives us

P[Z < 1] =P[N = 0] + P[N = 1] +1

2· P[N = 2]

= 0.72 + 2 · 0.3 · 0.7 + 0.5 · 0.32 = 0.955

E[N | Z < 1] = 0 · P[N = 0 | Z < 1] + 1 · P[N = 1 | Z < 1] + 2 · P[N = 2 | Z < 1]

= 0 + 1 · 2 · 0.3 · 0.70.955

+ 2 · 0.5 · 0.32

0.955= 0.534

5. Let H denote the event that the coin comes up heads, and T a tails. Then

E[X] =E[X | H] · P[H] + E[X | T ] · P[T ]

=1 + 6

2· 1

2+

1 + 4

2· 1

2= 3

6. Let H and T again denote heads and tails. From before, we know that E[X] = 3. Then

E[X2]=E[X2 | H

]· P[H] + E

[X2 | T

]· P[T ]

=1

(62 − 1

12+

[7

2

]2)+

1

(42 − 1

12+

[5

2

]2)=

34

3

Var[X] = E[X2]− (E[X])2 =34

3− 32 =

7

3

Or you could use the law of total variation:

Var[X] = E[Var[X | flip result] + Var[E[X | flip result]]

c©2012 The Infinite Actuary, LLC p. 2 C.6 Solutions

Page 152: The In nite Actuary Exam 1/P Online Seminar A.1 ... · The In nite Actuary Exam 1/P Online Seminar A.1 Fundamentals of Probability Solutions Last updated February 23, 2013 1. Let

=

(1

2

62 − 1

12+

1

2

42 − 1

12

)+

(7

2− 5

2

)21

2· 1

2

=25

12+

1

4=

7

3

7. Let X be the running time for the assembling machine, and Y the running time for the packingmachine. Then f(x, y) = x + y for 0 < x < 1 and 0 < y < 1. We want P[Y < 2X], so we want

x

y

1

1

P[Y < 2X] =

∫ 1

0

∫ 1

y/2

(x + y) dx dy

=

∫ 1

0

12 − (y/2)2

2+ y ·

(1− y

2

)dy =

∫ 1

0

1

2+ y − 5y2

8dy

=1

2+

1

2− 5

24=

19

24

8.

x

y

1

1

0.5

0.5

The unshaded region is less complicated, so we will use the complement.

P[X > 1/2 ∪ Y > 1/2] = 1− P[X ≤ 1/2, Y ≤ 1/2]

= 1−∫ 1/2

0

∫ 1/2

0

(x + y) dy dx

= 1−∫ 1/2

0

1

2x +

1

8dx

= 1−(

1

16+

1

16

)=

7

8

c©2012 The Infinite Actuary, LLC p. 3 C.6 Solutions

Page 153: The In nite Actuary Exam 1/P Online Seminar A.1 ... · The In nite Actuary Exam 1/P Online Seminar A.1 Fundamentals of Probability Solutions Last updated February 23, 2013 1. Let

9.

x

y

1

1

0.5

0.5

∫ 1/2

0

∫ 1

1/2

(x + y) dy dx +

∫ 1

1/2

∫ 1/2

0

(x + y) dy dx =

∫ 1/2

0

x

2+

12 − (1/2)2

2dx +

∫ 1

1/2

x

2+

(1/2)2

2dx

=1

16+

3

16+

12 − (1/2)2

4+

1

16=

4

8

10.E[X + Y ] =

∫ 1

0

∫ 1

0

(x + y)(x + y) dy dx =

∫ 1

0

∫ 1

0

(x + y)2 dy dx

=

∫ 1

0

(x + y)3

3

∣∣∣∣10

dx =

∫ 1

0

(1 + x)3 − x3

3dx

=(1 + x)4 − x4

12

∣∣∣∣10

=16− 1

12− 1

12=

7

6

11. From the previous problem, we know that E[X + Y ] = 7/6, so we need the second moment.But

E[(X + Y )2] =

∫ 1

0

∫ 1

0

(x + y)2(x + y) dy dx =

∫ 1

0

∫ 1

0

(x + y)3 dy dx

=

∫ 1

0

(x + y)4

4

∣∣∣∣10

dx =

∫ 1

0

(1 + x)4 − x4

4dx

=(1 + x)5 − x5

20

∣∣∣∣10

=32− 1

20− 1

20=

3

2

Var[X + Y ] =3

2−(

7

6

)2

=5

36= 0.139

12.E

[X

Y

]=

1

1· P[X = Y = 1] +

1

2· P[X = 1, Y = 2]

+2

1· P[X = 2, Y = 1] +

2

2· P[X = Y = 2]

= 1 · 2

9+

1

2· 1

9+ 2 · 4

9+ 1 · 2

9

=25

18

c©2012 The Infinite Actuary, LLC p. 4 C.6 Solutions

Page 154: The In nite Actuary Exam 1/P Online Seminar A.1 ... · The In nite Actuary Exam 1/P Online Seminar A.1 Fundamentals of Probability Solutions Last updated February 23, 2013 1. Let

13.

x

y

1

1

Since (X, Y ) are uniform over the region, f(x, y) = c for some constant c, and the marginal of Xis

fX(x) =

∫ √xy=0

c dy = c√x

Since the marginal is a density, we can find c by setting

1 =

∫ 1

0

fX(x) dx =

∫ 1

0

c√x dx

1 = c · 2

3x3/2

∣∣∣∣10

= c · 2

3

c =3

2, fX(x) =

3

2

√x

14.

P[X = 0] = P[X = 0, Y = 1] + P[X = 0, Y = 2] =2 · 0 + 1

12+

2 · 0 + 2

12=

1

4

P[X = 1] = P[X = 1, Y = 2] + P[X = 1, Y = 3] =2 · 1 + 2

12+

2 · 1 + 3

12=

3

4

Var[X] = (1− 0)2 · 1

4· 3

4=

3

16

Or we could find E[X] and E[X2] directly without finding the marginal:

E[X] = 0 · P[X = 0, Y = 1] + 0 · P[X = 0, Y = 2]

+ 1 · P[X = 1, Y = 2] + 1 · P[X = 1, Y = 3]

= 0 + 0 +4

12+

5

12=

3

4E[X2] = 0 + 0 + 12 · P[X = 1, Y = 2] + 12 · P[X = 1, Y = 3]

= 0 + 0 +4

12+

5

12=

3

4

Var[X] =3

4−(

3

4

)2

=3

16

15. P[X = 0 | Y = 2] =P[X = 0, Y = 2]

P[Y = 2]

c©2012 The Infinite Actuary, LLC p. 5 C.6 Solutions

Page 155: The In nite Actuary Exam 1/P Online Seminar A.1 ... · The In nite Actuary Exam 1/P Online Seminar A.1 Fundamentals of Probability Solutions Last updated February 23, 2013 1. Let

=2/12

2/12 + 4/12=

1

3

P[X = 1 | Y = 2] =P[X = 1, Y = 2]

P[Y = 2]=

2

3

Var[X | Y = 2] =1

3· 2

3=

2

9

16. From before, we know E[X] = 3/4.

E[Y ] = 1 · P[X = 0, Y = 1] + 2 · P[X = 0, Y = 2] + 2 · P[X = 1, Y = 2] + 3 · P[X = 1, Y = 3]

=1

12+

2 · 212

+2 · 412

+3 · 512

=28

12E[XY ] = 0 · P[X = 0, Y = 1] + 0 · P[X = 0, Y = 2] + 2 · P[X = 1, Y = 2] + 3 · P[X = 1, Y = 3]

= 0 + 0 +2 · 412

+3 · 512

=23

12

Cov(X, Y ) =23

12− 3

4· 28

12=

1

6

17. Since you have to have basic coverage to purchase supplemental, Y < X, so 0 < y < x < 1.

x

y

1

1

1 =

∫ 1

0

∫ x

0

c(x + y) dy dx

=

∫ 1

0

c

2[(2x)2 − x2] dx =

∫ 1

0

3c

2x2 dx =

c

2

c = 2

E[W ] =

∫ 1

0

∫ x

0

√x + y · 2(x + y) dy dx =

∫ 1

0

∫ x

0

2(x + y)3/2 dy dx

=

∫ 1

0

2 · 25

[(2x)5/2 − x5/2]dx

=4

5· 2

7· [25/2 − 1] · 1 = 1.064

c©2012 The Infinite Actuary, LLC p. 6 C.6 Solutions

Page 156: The In nite Actuary Exam 1/P Online Seminar A.1 ... · The In nite Actuary Exam 1/P Online Seminar A.1 Fundamentals of Probability Solutions Last updated February 23, 2013 1. Let

18.

x

y

1

1

f(x, y) = fX(x) · fY (y) by independence

= 1 · 2y = 2y

P[Y < X] =

∫ 1

0

∫ x

0

2y dy dx

=

∫ 1

0

x2 dx =1

3

19. E[(X + 1)2(Y 2 − 1)] = E[X2 + 2X + 1] · E[Y 2 − 1] by independence

=(E[X2] + 2E[X] + 1

)·(E[Y 2]− 1

)=(3 + 12 + 2 · 1 + 1

)·(3 + 12 − 1

)= 21

20. P[X = x | Y = 2] =P[X = x, Y = 2]

P[Y = 2]

=(x + 2)/21

(1 + 2)/21 + (2 + 2)/21 + (3 + 2)/21

=x + 2

(1 + 2) + (2 + 2) + (3 + 2)=

x + 2

12

21.

x

y

1

1

Originally our distribution is over the largest triangle. We are given that X ≤ 1/2, so we are giventhat we are in one of the shaded regions, and we want the probability of being in the green shaded

c©2012 The Infinite Actuary, LLC p. 7 C.6 Solutions

Page 157: The In nite Actuary Exam 1/P Online Seminar A.1 ... · The In nite Actuary Exam 1/P Online Seminar A.1 Fundamentals of Probability Solutions Last updated February 23, 2013 1. Let

region.

P[X ≤ 1/2] =

∫ 1/2

0

∫ x

0

2(x + y) dy dx =

∫ 1/2

0

(2x)2 − x2 dx

=

∫ 1/2

0

3x2dx =

(1

2

)3

=1

8

P[blue region] =

∫ 1/2

3/8

∫ x

3/4−x2(x + y) dy dx

=

∫ 1/2

3/8

(x + x)2 − (x− (3/4− x))2 dx =

∫ 1/2

3/8

4x2 − 9/16 dx

=4

3

[1

8− 33

83

]− 9

16

(1

2− 3

8

)=

5

192

P[blue region | X ≤ 1/2] =5/192

1/8=

5

24

P[X + Y ≤ 3/4 | X ≤ 1/2] = 1− 5

24=

19

24

22.

x

y

y

1

1

We are given that we are in the blue triangle, and want the conditional density of y. If it werea discrete situation, we would want P[Y = y,X ≤ 1/2]/P [X ≤ 1/2]. In a joint continuous case,P[X ≤ 1/2] still makes sense, so we still want to use that. The numerator has to become a densitysince P[Y = y] = 0, and since we want X ≤ 1/2, we want to integrate f(x, y) along the green lineand divide by P[X ≤ 1/2]. From the previous problem, we know that P[X ≤ 1/2] = 1/8

fY |X(y | X ≤ 1/2) =

∫ 1/2

y2(x + y) dx

P[X ≤ 1/2]

=(1/2 + y)2 − (2y)2

1/8= 8 · 1

4+ 8 · 2 · 1

2y + 8y2 − 8 · 4y2

= 2 + 8y − 24y2

c©2012 The Infinite Actuary, LLC p. 8 C.6 Solutions

Page 158: The In nite Actuary Exam 1/P Online Seminar A.1 ... · The In nite Actuary Exam 1/P Online Seminar A.1 Fundamentals of Probability Solutions Last updated February 23, 2013 1. Let

23. Step 1: Find g−1, i.e., solve for X and Y in terms of U and W .

U = Y/X, W = X

U = Y/W, Y = UW

Step 2: Find J

J =

∣∣∣∣∣∣∣∣∂x

∂u

∂x

∂w∂y

∂u

∂y

∂w

∣∣∣∣∣∣∣∣ =

∣∣∣∣∣ 0 1

w u

∣∣∣∣∣= 0 · u− 1 · w = −w

Step 3: Plug everything into the formula

fU,V (u, v) = fX,Y (x = w, y = uw) · |J |

= 8w · uw · | − w| = 8uw3

24. Var(X + Y ) = 6 = Var(X) + 2Cov(X, Y ) + Var(Y )

Var(X − Y ) = 2 = Var(X)− 2Cov(X, Y ) + Var(Y )

Cov(X, Y ) = 1 by subtracting the second equation from the first

Cov(X, Y ) = 1 = E[XY ]− E[X] · E[Y ]

1 = E[XY ]− 2 · 2E[XY ] = 5

25.E[X] =

5∑x=2

2∑y=0

x · P[X = x, Y = y] = 2.85

E[Y ] =5∑

x=2

2∑y=0

y · P[X = x, Y = y] = 1

E[XY ] =5∑

x=2

2∑y=0

xy · P[X = x, Y = y] = 2.7

Cov(X, Y ) = 2.7− 2.85 · 1 = −0.15

26. Let X and Y be independent U(0, 1) random variables. We want

Var(XY ) = E[(XY )2]− (E[XY ])2

= E[X2Y 2]− (E[XY ])2 = E[X2]E[Y 2]− (E[X])2(E[Y ])2 by independence

=

[(1

2

)2

+1

12

]2−[(0.5)2

]2c©2012 The Infinite Actuary, LLC p. 9 C.6 Solutions

Page 159: The In nite Actuary Exam 1/P Online Seminar A.1 ... · The In nite Actuary Exam 1/P Online Seminar A.1 Fundamentals of Probability Solutions Last updated February 23, 2013 1. Let

=7

144

27.

max = X

max = Y

X

Y

40

40

E[X] =0 + 40

2= 40 · 1

2

E[max{X, Y }] =

∫ 40

0

∫ x

0

x

1600dy dx +

∫ 40

0

∫ 40

x

y

1600dy dx

=

∫ 40

0

x2

1600dx +

∫ 40

0

402 − x2

2 · 1600dx

=40

3+

40

2− 40

6= 40 · 2

3

E[X ·max{X, Y }] =

∫ 40

0

∫ x

0

x2

1600dy dx +

∫ 40

0

∫ 40

x

xy

1600dy dx

=

∫ 40

0

x3

1600+

∫ 40

0

402x− x3

2 · 1600dx

=402

4+

402

2 · 2− 402

8= 402 · 3

8

Cov(X,max{X, Y }) = 402 · 3

8− 40 · 1

2· 40 · 2

3

= 402 · 1

24= 66.7

c©2012 The Infinite Actuary, LLC p. 10 C.6 Solutions

Page 160: The In nite Actuary Exam 1/P Online Seminar A.1 ... · The In nite Actuary Exam 1/P Online Seminar A.1 Fundamentals of Probability Solutions Last updated February 23, 2013 1. Let

Discrete Distributions

Variable Key Properties P[X = x] E[X] Var[X] mgf

Uniform1

n

n+ 1

2

n2 − 1

12on {1, 2, . . . , n} 1 ≤ x ≤ n

Bernoulli 0 or 1 p, x = 1 p p(1− p) pet + q1− p, x = 0

Sum of Bernoullis

Binomial number of successes

(n

x

)px(1− p)n−x np np(1− p) (pet + q)

n

in n trials

Geometric Number of trials p(1− p)x−1 1

p

1− pp2

pet

1− qeton {1, 2, . . . } until first success

Geometric Number of failures p(1− p)x 1

p− 1

1− pp2

p

1− qeton {0, 1, . . . } before first success

Negative Sum of Geometric on 0, . . .

Binomial Number of failure

(x+ r − 1

r − 1

)pr(1− p)n−r r(1− p)

p

r(1− p)p2

(p

1− qet

)r

until r-th succss

Poisson Sum of Poissons e−λ · λn

n!λ λ eλ(e

t−1)

is Poisson

Bernoulli Variance Shortcut: If X is a random variable that can only take on 2 values, with P[X = a] = p and P[X = b] = 1 − p,then Var[X] = (b− a)2p(1− p)

Page 161: The In nite Actuary Exam 1/P Online Seminar A.1 ... · The In nite Actuary Exam 1/P Online Seminar A.1 Fundamentals of Probability Solutions Last updated February 23, 2013 1. Let

Continuous Distributions

One trick to remembering the Gamma distribution is to recall that if α is an integer, then a Gamma(α, θ) distribution is the sum ofα exponentials with mean θ.

Densities are only given for the region on which they are positive, and are 0 otherwise.

Distribution range density cdf EX VarX MGF

Uniform on (0, 1) 0 < x < 1 1 x for 0 < x < 11

2

1

12

et − 1

t

Uniform on (a, b) a < x < b1

b− ax− ab− a

a+ b

2

(b− a)2

12

etb − eta

t(b− a)Exponential

with “rate” λ 0 < x <∞ λe−λx =1

θe−x/θ 1− e−λx = 1− e−x/θ θ =

1

λθ2 =

1

λ21

1− θt=

λ

λ− tand mean θ =

1

λ

Gamma 0 < x <∞ xα−1

θα(α− 1)!e−x/θ see below αθ αθ2

(1

1− θt

Standard Normal −∞ < x <∞ 1√2πe−x

2/2 Φ(x) 0 1 et2/2

Normal(µ, σ2) −∞ < x <∞ 1

σ√

2πe−(x−µ)2/(2σ2) Φ

(x− µσ

)µ σ2 etµ+(t2σ2)/2

For α = 1, a Gamma is an exponential with mean θ and so has CDF P[X ≤ t] = 1− e−t/θ

For α = 2, a Gamma has cdf P[X ≤ t] = 1− e−t/θ −(t

θ

)e−t/θ

For α = 3, a Gamma has cdf P[X ≤ t] = 1− e−t/θ −(t

θ

)e−t/θ −

((t/θ)2

2

)e−t/θ

Note that each time we increase α by 1, we subtract another term from the cdf that looks that the probability distribution of aPoisson. In general, if α is an integer, the cdf of a Gamma(α, θ) is given by P[X ≤ t] = P[X < t] = 1 − P[N ≤ α − 1] = P[N ≥ α]where N is a Poisson random variable with mean λ = t/θ.

Page 162: The In nite Actuary Exam 1/P Online Seminar A.1 ... · The In nite Actuary Exam 1/P Online Seminar A.1 Fundamentals of Probability Solutions Last updated February 23, 2013 1. Let

Fundamentals and conditional probabilities

P[Ac] = 1− P[A] P[A ∪B] = P[A] + P[B]− P[AB]

P[A | B] =P[AB]

P[B]P[AB] = P[B]P[A | B]

A and B are mutually exclusive if P[AB] = 0.A and B are independent if P[AB] = P[A]P[B], which also implies that P[A | B] = P[A].Three events A,B and C are independent if every possible combination factors, e.g., P[ABC] = P[A]P[B]P[C],P[AcC] = P[Ac]P[C], etc.

If A1, A2, . . . , An are mutually exclusive and∑

P[Ai] = 1 (i.e., the Ai are a way of breaking things down into allpossible cases), then

P[B] =∑i

P[B | Ai]P[Ai] “Law of Total Probability”

P[Aj | B] =P[AjB]

P[B]=

P[B | Aj ]P[Aj ]∑i P[B | Ai]P[Ai]

Bayes’ Theorem

CDFs and densities:

F (x) = P[X ≤ x]

If F (x) is continuous, then f(x) =d

dxF (x) is the density and P[a ≤ X ≤ b] =

∫ b

a

f(x)dx

If F (x) has a jump at a, i.e. F (a) > limx↑a

F (x), then the size of the jump is the probability of X = a, i.e.,

P[X = a] = F (a)− F (a−).

Moments:

E[X] =∑x

x · P[X = x] +

∫ ∞−∞

xf(x)dx

(for discrete random variables, only the first part applies, for purely continuous, only the second, for mixed distri-butions use both)

If X ≥ 0, then E[X] =

∫ ∞0

P[X > x]dx.

If X ≥ 0 and only takes on integer values, E[X] =∑∞

k=0 P[X > k].

E[X2] =∑x

x2 · P[X = x] +

∫x2f(x)dx

E[g(X)] =∑x

g(x)P[X = x] +

∫g(x)f(x)dx

Var[X] = E[(X − µX)

2]

= E[X2]− (E[X])2

Var[aX] = a2Var[X]

SD[X] =√

Var[X]

Cov(X,Y ) = E[XY ]− E[X] · E[Y ]

Cov(X,X) = Var[X]

Page 163: The In nite Actuary Exam 1/P Online Seminar A.1 ... · The In nite Actuary Exam 1/P Online Seminar A.1 Fundamentals of Probability Solutions Last updated February 23, 2013 1. Let

Corr(X,Y ) =Cov(X,Y )

SD(X)SD(Y )

Coefficient of Variation of X =SD(X)

E(X)

Covariance is bilinear, meaning that it distributes as you might expect. This means that Cov(aX + bY, cZ) =acCov(X,Z) + bcCov(Y,Z),Var(aX + bY ) = a2VarX + 2abCov(X,Y ) + b2VarY .

Moment generating functions:

MX(t) = E[etX]

MX(0) = E[e0]

= 1

d

dtMX(t)

∣∣∣∣0

= E[X]

d(n)

dt(n)MX(t)

∣∣∣∣0

= E[Xn]

MaX+b(t) = E[eatX+bt] = ebtMX(at)

Normal, Gamma, and Binomial are the most common mgf’s used on the exam.If X and Y are independent, then MX+Y (t) = MX(t)MY (t).

Joint moment generating functions:

MX1,X2(t1, t2) = Eet1X1+t2X2

E[Xm1 X

n2 ] =

∂(m)

∂x(m)1

∂(n)

∂x(n)1

M(t1, t2)

∣∣∣∣∣(0,0)

MX1(t1) = Eet1X1+0X2 = MX1,X2(t1, 0)

Convolutions and transformations

If Z = X + Y , then fZ(z) =

∫ ∞−∞

fX,Y (x, z − x)dx =

∫ ∞−∞

fX(x)fY (z − x) The last equality (convolution formula)

only holds if X and Y are indepedent.You can also find fZ(z) by finding the CDF and differentiating.

If Y = g(X) and g is one-to-one, then fY (y) = fX [g−1(y)]

∣∣∣∣ ddy g−1(y)

∣∣∣∣ If g is two-to-one (or more), then you can

use this with each “inverse” and add them up.

If (Y1, Y2) = g(X1, X2), then fY1,Y2(y1, y2) = fX1,X2

[g−1(y1, y2)]∣∣J(g−1)

∣∣,where J(g−1) =

∣∣∣∣∣∣∣∣∂g−11

∂y1

∂g−11

∂y2

∂g−12

∂y1

∂g−12

∂y2

∣∣∣∣∣∣∣∣ and

∣∣∣∣ a bc d

∣∣∣∣ = ad− bc


Recommended